SlideShare a Scribd company logo
1 of 75
Tải tài liệu tại sividoc.com
Viết đề tài giá sinh viên – ZALO:0973.287.149-TEAMLUANVAN.COM
ĐẠI HỌC THÁI NGUYÊN
TRƯỜNG ĐẠI HỌC KHOA HỌC
NGUYỄN THẾ NGHĨA
SỬ DỤNG HÀNG ĐIỂM ĐIỀU HÒA
TRONG GIẢI TOÁN HÌNH HỌC PHẲNG
LUẬN VĂN THẠC SĨ TOÁN HỌC
THÁI NGUYÊN - 2016
Tải tài liệu tại sividoc.com
Viết đề tài giá sinh viên – ZALO:0973.287.149-TEAMLUANVAN.COM
ĐẠI HỌC THÁI NGUYÊN
TRƯỜNG ĐẠI HỌC KHOA HỌC
NGUYỄN THẾ NGHĨA
SỬ DỤNG HÀNG ĐIỂM ĐIỀU HÒA
TRONG GIẢI TOÁN HÌNH HỌC PHẲNG
LUẬN VĂN THẠC SĨ TOÁN HỌC
Chuyên ngành
Mã số
: Phương pháp Toán sơ cấp
:60460113
NGƯỜI HƯỚNG DẪN KHOA HỌC
TS. NGUYỄN DANH NAM
THÁI NGUYÊN - 2016
Tải tài liệu tại sividoc.com
Viết đề tài giá sinh viên – ZALO:0973.287.149-TEAMLUANVAN.COM
i
Mục lục
Trang
LỜI MỞ ĐẦU........................................................................................................... 1
Chương 1: KIẾN THỨC CHUẨN BỊ..................................................................... 2
1.1. Tỉ số đơn, tỉ số kép và hàng điểm điều hòa......................................................... 2
1.2. Chùm đường thẳng và tứ giác toàn phần ............................................................ 5
1.3. Đường tròn trực giao........................................................................................... 9
1.4. Cực và đường đối cực ......................................................................................... 9
1.5. Cách xác định cực và đường đối cực ................................................................ 16
Chương 2: SỬ DỤNG HÀNG ĐIỂM ĐIỀU HÒA TRONG GIẢI TOÁN HÌNH
HỌC PHẲNG ......................................................................................................... 19
2.1. Chứng minh hàng điểm điều hòa ...................................................................... 19
2.2. Chứng minh vuông góc..................................................................................... 25
2.3. Chứng minh song song...................................................................................... 31
2.4. Chứng minh thẳng hàng .................................................................................... 33
2.5. Chứng minh đồng quy....................................................................................... 40
2.6. Chứng minh điểm cố định................................................................................. 46
2.7. Chứng minh đẳng thức...................................................................................... 55
2.8. Một số bài toán khác ......................................................................................... 64
KẾT LUẬN ............................................................................................................. 71
TÀI LIỆU THAM KHẢO ..................................................................................... 72
Tải tài liệu tại sividoc.com
Viết đề tài giá sinh viên – ZALO:0973.287.149-TEAMLUANVAN.COM
1
LỜI MỞ ĐẦU
Hình học phẳng là một chủ đề hấp dẫn trong các kì thi học sinh giỏi. Một bài
toán hình học phẳng luôn có thể được giải bằng nhiều cách khác nhau, trong đó áp
dụng các khái niệm “hàng điểm điều hòa”, “cực và đường đối cực” được vận dụng
để giải các bài toán sẽ cho lời giải một cách ngắn gọn và đẹp mắt. Đây là những
công cụ mạnh và thú vị của hình học. Kiến thức về chùm đường thẳng, phép chiếu
xuyên tâm, đặc biệt là chùm đường thẳng điều hòa, tứ giác toàn phần cũng được sử
dụng để tìm kiếm các hàng điểm điều hòa. Khi xuất hiện các hàng điểm điều hòa,
chúng ta dễ dàng sử dụng các kết quả liên quan như hệ thức Đề-các, hệ thức Niu-
tơn và hệ thức Mácloranh trong giải bài toán hình học phẳng.
Với hướng khai thác các hàng điểm điều hòa đơn giản và các hàng điểm điều
hòa xuất hiện từ quan hệ giữa cực và đường đối cực của một điểm đối với một cặp
đường thẳng cắt nhau hoặc đối với một đường tròn nào đó để giải các dạng toán
hình học như: chứng minh thẳng hàng, chứng minh đồng quy, chứng minh song
song, chứng minh vuông góc, chứng minh điểm cố định, chứng minh đẳng thức, bài
toán quỹ tích và bài toán dựng hình. Trong luận văn này, chúng tôi quan tâm đến
các bài toán có liên quan đến hàng điểm điều hòa xuất hiện trong các cuộc thi học
sinh giỏi toán quốc gia và toán quốc tế. Các bài toán về hàng điểm điều hòa trong
luận văn đã được lựa chọn với lời giải của có tính độc đáo và thú vị hơn so với các
phương pháp thường gặp. Do vậy, có thể nói kết quả của luận văn cung cấp một
công cụ mới cho học sinh trong việc tiếp cận và giải các bài toán hình học phẳng,
đặc biệt là các bài toán xuất hiện trong các kì thi học sinh giỏi môn Toán.
Luận văn này được thực hiện tại Trường Đại học Khoa học - Đại học Thái
Nguyên và hoàn thành dưới sự hướng dẫn của TS. Nguyễn Danh Nam. Tác giả xin
bày tỏ lòng biết ơn sâu sắc tới thầy hướng dẫn đã tận tình giúp đỡ trong suốt quá
trình làm luận văn. Tác giả cũng xin chân thành cảm ơn các GS, PGS, TS và các
thầy cô giảng viên của Trường Đại học Khoa học - Đại học Thái Nguyên đã giảng
dạy và tạo mọi điều kiện thuận lợi trong quá trình tác giả học tập và nghiên cứu.
Tải tài liệu tại sividoc.com
Viết đề tài giá sinh viên – ZALO:0973.287.149-TEAMLUANVAN.COM
2
Chương 1
KIẾN THỨC CHUẨN BỊ
1.1. Tỉ số đơn, tỉ số kép và hàng điểm điều hòa
1.1.1. Độ dài đại số
Trên đường thẳng d chọn véctơ đơn vị e thì ta có trục d và hướng của e là
hướng của trục d.
Định nghĩa 1.1. [1] Trên trục d, cho hai điểm A, B. Độ dài đại số của AB là
một số có giá trị tuyệt đối bằng AB và số đó dương nếu AB cùng hướng với e và
số đó âm nếu AB ngược hướng với e . Kí hiệu: AB .
Các tính chất.
1) ABBA.
2) AB BC AC (A, B, C thẳng hàng).
3) A1 A2 A2 A3 ... An1 An A1 An (với mọi Ai , i1, n thẳng hàng).
1.1.2. Tỉ số đơn
Định nghĩa 1.2. [1] Cho ba điểm A, B, C thẳng hàng, tỉ số đơn của chúng lấy
theo thứ tự đó là tỉ số CB
CA
. Kí hiệu: (ABC).
Định lý 1.1. [1] Cho hai điểm A, B và một số thực k 1 thì tồn tại duy nhất
điểm C sao cho (ABC) = k.
Chứng minh.
Ta có (ABC) = k CB
CA
 k CA kCB CA kCA AB
 CA kAB AC CA k AC k AB AC k
k
1 AB (k1)
Suy ra, tồn tại duy nhất điểm C sao cho (ABC) = k.
Tải tài liệu tại sividoc.com
Viết đề tài giá sinh viên – ZALO:0973.287.149-TEAMLUANVAN.COM
3
1.1.3. Tỉ số kép
Định nghĩa 1.3. [1] Cho bốn điểm A, B, C, D thẳng hàng, tỉ số kép của
chúng lấy theo thứ tự đó là tỉ số CB
CA
: DB
DA
. Kí hiệu: (ABCD).
VậyABCD CB
CA
: DB
DA


ABD
ABC
.
Các tính chất.
1) Tỉ số kép của bốn điểm là không thay đổi trong các trường hợp sau: +
Nếu hoán vị cặp điểm đầu với cặp điểm cuối: (ABCD) = (CDAB).
+ Nếu đồng thời hoán vị hai điểm đầu và hai điểm cuối:
(ABCD) = (BADC)
+ Nếu viết chúng theo thứ tự ngược lại: (ABCD) = (DCBA).
2) Tỉ số kép của bốn điểm thay đổi trong các trường hợp:
+ Nếu hoán vị hai điểm đầu hoặc hai điểm cuối thì tỉ số kép của bốn điểm trở
thành số đảo ngược của nó:
(BACD) = (ABDC)
1
 ABCD
+ Nếu hoán vị hai điểm ở giữa hoặc hai điểm ở đầu và cuối thì tỉ số kép của
bốn điểm trở thành phần bù của 1:ABCD 1 ACBD 1DBCA.
1.1.4. Hàng điểm điều hoà
Định nghĩa 1.4. [1] Nếu (ABCD) = -1 thì ta nói bốn điểm A, B, C, D lập
thành một hàng điểm điều hoà hay A, B chia điều hoà C, D hay A, B liên hợp điều
hoà đối với C, D.
Các tính chất. Cho bốn điểm A, B, C, D thẳng hàng, ta có:
1) Hệ thức Đề-các:ABCD1 AB
2
 AC
1
 AD
1
.
2) Hệ thức Niu-tơn:ABCD1 IA2
 IC .ID (trong đó I là trung
điểm của đoạn thẳng AB).
Tải tài liệu tại sividoc.com
Viết đề tài giá sinh viên – ZALO:0973.287.149-TEAMLUANVAN.COM
4
3) Hệ thức Mácloranh: AC. AD AB.AJ (trong đó J là trung điểm của đoạn
thẳng CD).
Chứng minh. Trên đường thẳng AB, chọn O làm gốc toạ độ.
Đặt OA = a, OB = b, OC = c, OD = d, ta có:
CA OA OC = a – c ; CB OB OC = b - c
DA OD OA = d – a ; DB OD OB = d - b
Ta có ABCD1CA :DA a - c a - d
b - c b - d
CB DB
 (a - c)(b - d) - (a - d)(b - c)
 2(ab + cd) (a + b)(c + d) (1)
+ Chọn OA thì: OA = a = 0, AC = OC = c, AB = OB = b, AD = OD = d.
Từ (1) ta có 2cd = bc + bd b
2
 d
1

1
c AB
2
 AC
1
 AD
1
.
+ Chọn O I thì ta có OAOB hay a = - b.
Từ (1) ta có 2(- a2
+ cd) = 0 a2
= cd IA2
 IC.ID .
Chứng minh tương tự đối với hệ thức Mácloranh.
Định lý 1.2. [1] Nếu AD, AE lần lượt là phân giác trong, phân giác ngoài của
tam giác ABC (D, E thuộc đường thẳng BC) thì (BCDE) = - 1.
A
B D C E
Hình 1.1
Định lý 1.3. [1] Cho tam giác ABC và điểm O không thuộc các đường thẳng
chứa ba cạnh của tam giác. Các đường thẳng AO, BO, CO theo thứ tự cắt BC, CA,
AB tại M, N, P và BC cắt NP tại Q. Khi đó ta có (BCMQ) = - 1.
Tải tài liệu tại sividoc.com
Viết đề tài giá sinh viên – ZALO:0973.287.149-TEAMLUANVAN.COM
5
A
P N
O
B M C Q
Hình 1.2
Định lý 1.4. [1] Từ điểm S nằm ngoài đường tròn (O) kẻ các tiếp tuyến SA,
SB tới (O) (A, B là các tiếp điểm ). Một đường thẳng đi qua S và cắt (O) lần lượt tại
M, N, và AB cắt MN tại I. Khi đó (SIMN) = - 1.
Hình 1.3
1.2. Chùm đường thẳng và tứ giác toàn phần
1.2.1. Chùm đường thẳng
Định nghĩa 1.5. [1] Trong mặt phẳng, cho tập hợp các đường thẳng đồng
quy tại điểm S thì chúng lập nên một chùm đường thẳng và S được gọi là tâm của
chùm.
Tập hợp các đường thẳng nằm trong mặt phẳng và song song với nhau lập
nên một chùm đường thẳng và có tâm tại vô tận.
Định lý 1.5. [1] Một chùm bốn đường thẳng cắt một đường thẳng theo hàng
điểm có tỉ số kép không thay đổi.
Chứng minh.
* Trường hợp chùm đồng quy tại điểm S: Gọi l là đường thẳng cắt các đường
thẳng a, b, c, d lần lượt tại A, B, C, D và l’ là đường thẳng cắt các đường thẳng a, b,
Tải tài liệu tại sividoc.com
Viết đề tài giá sinh viên – ZALO:0973.287.149-TEAMLUANVAN.COM
6
c, d lần lượt tại A’, B’, C’, D’. Ta cần chứng minh đẳng thức (ABCD) = (A’B’C’D’)
(Hình 1.4).
S
N’
l’
M’ N
B’
M
C’ D’
A’
A B C D l
a b c d
Hình 1.4
Qua điểm B kẻ đường thẳng song song với đường thẳng a và cắt đường thẳng
c tại N, cắt đường thẳng d tại M.
Ta có:
CA SA và DA SA
 
CB MB DB NB
Từ đó suy ra:
CA DA SA SA NB (1)
ABCD :  : 
CB AB MB NB MB
Tương tự, từ điểm B’ kẻ đường thẳng song song với đường thẳng a và cắt
đường thẳng c, d lần lượt tại M’, N’.
Ta có A' B 'C ' D '
N 'B'
(2)
M 'B'
Mặt khác, ta có: NB N 'B' (3)

MB M 'B'
Từ (1), (2) và (3) ta có (ABCD) = (A’B’C’D’).
* Trường hợp chùm song song: Nếu a // b // c // d thì ta luôn có đẳng thức
(ABCD) = (A’B’C’D’).
Tải tài liệu tại sividoc.com
Viết đề tài giá sinh viên – ZALO:0973.287.149-TEAMLUANVAN.COM
7
Định nghĩa 1.6. [1] Trong mặt phẳng cho chùm bốn đường thẳng a, b, c, d.
Một đường thẳng l bất kì cắt chùm đó tại A, B, C, D thì (ABCD) được gọi là tỉ số
kép của chùm bốn đường thẳng a, b, c, d. Kí hiệu: (abcd) = (ABCD).
Nếu chùm đồng quy tại S thì ta kí hiệu:
S(abcd) = (ABCD).
S
l
Nếu (abcd) = - 1 thì ta có một chùm điều N
hoà, hay a, b liên hợp điều hoà với c, d hay a, b B
chia điều hoà c, d.
Định lý 1.6. [1] Trong mặt phẳng cho
M d
chùm bốn đường thẳng đồng quy. Điều kiện cần a c
b
và đủ để chùm đó lập thành một chùm điều hoà
Hình 1.5
là: Một đường thẳng bất kì song song với một
trong bốn đường thẳng đó bị ba đường thẳng còn lại chia thành hai đoạn thẳng
bằng nhau.
Chứng minh. Kẻ đường thẳng l song song với a và cắt b, c, d lần lượt tại M,
B, N.
Theo định lý trên, ta có:
ABCD(abcd) = MB
NB
và (abcd) = -1
 MB
NB
1 NBMB

 B là trung điểm của đoạn thẳng MN hay MB = NB (Hình 1.5).
Hệ quả 1. Trong một chùm điều hoà nếu có hai đường liên hợp vuông góc
với nhau thì hai đường đó là các đường phân giác của các góc tạo bởi hai đường còn
lại (Hình 1.6a).
Hệ quả 2. Hai đường phân giác của hai góc kề bù chia điều hoà hai cạnh của
góc đó (Hình 1.6b). Chùm đường thẳng gồm hai cạnh của một góc và hai đường
phân giác của góc đó được gọi là chùm phân giác.
Tải tài liệu tại sividoc.com
Viết đề tài giá sinh viên – ZALO:0973.287.149-TEAMLUANVAN.COM
8
S S
b
C
B
D A
c a d
a) b)
Hình 1.6
Trong mặt phẳng, tập hợp các đường thẳng đồng quy tại một điểm S, được
gọi là một chùm đường thẳng tâm S.
Cho chùm bốn đường thẳng a, b, c, d. Một đường thẳng bất kỳ cắt a, b, c, d
thứ tự tại A, B, C, D. Khi đó (ABCD) không phụ thuộc vào vị trí của .Giá trị
không đổi của tỉ số kép (ABCD) được gọi là tỉ số kép của chùm bốn đường thẳng a,
b, c, d, ký hiệu (abcd) hay S(abcd) khi cần quan tâm đến tâm của chùm.
1.2.2. Tứ giác toàn phần
Định nghĩa 1.7. [1] Trong mặt phẳng, cho bốn đường thẳng cắt nhau từng đôi
một và không có ba đường nào đồng quy thì chúng lập thành một tứ giác toàn phần.
- Các đường thẳng là các cạnh (có bốn cạnh).
- Giao của hai cạnh là đỉnh (có sáu đỉnh).
- Hai đỉnh không thuộc một cạnh là hai đỉnh đối diện (có ba cặp đỉnh đối diện).
- Đường thẳng nối hai đỉnh đối diện là đường chéo (có ba đường chéo).
Cho tứ giác toàn phần ABCA’B’C’. Khi đó, ta có cặp đỉnh đối diện là (A, A’),
(B, B’), (C, C’); ba đường chéo là AA’, BB’, CC’.
Định lý 1.7. [1] Trong một tứ giác toàn phần, cặp đỉnh đối diện chia điều hoà
hai giao điểm của đường chéo nối cặp đỉnh đối diện đó với hai đường chéo còn lại.
Chứng minh. Gọi P = AA’BB’, Q = AA’CC’, R = BB’CC’.
Ta chứng minh (AA’PQ) = (BB’PR) = (CC’QR) = - 1. Ta có:
B(AA’PQ) = B’(AA’PQ) = B’(CC’RQ) = B(CC’RQ) = B(A’APQ).
 (AA’PQ) = (A’APQ)AA' PQ
1
AA' PQ
2
1.
AA' PQ
Nếu (AA’PQ) = 1 thì ta có (AA’P) = (AA’Q) hay PQ (vô lý).
Tải tài liệu tại sividoc.com
Viết đề tài giá sinh viên – ZALO:0973.287.149-TEAMLUANVAN.COM
9
Vậy (AA’PQ) = - 1.
Các tỉ số kép khác được chứng minh một cách tương tự.
A
B
P
B’
A’
C
Q C’ R
Hình 1.7
1.3. Đường tròn trực giao
Định nghĩa 1.8. [3] Hai đường tròn gọi là trực giao với nhau tại một điểm
chung của chúng nếu tại điểm đó hai tiếp tuyến của hai đường tròn vuông góc với
nhau.
Từ định nghĩa, ta dễ dàng suy ra được các kết quả sau:
Định lý 1.8. [3] Điều kiện cần và đủ để hai đường tròn trực giao với nhau là
bình phương khoảng cách giữa hai tâm bằng tổng bình phương các bán kính của
chúng.
Định lý 1.9. [3] Điều kiện cần và đủ để hai đường tròn trực giao với nhau là
phương tích của tâm của một trong hai đường tròn đó đối với đường tròn thứ hai
bằng bình phương bán kính của đường tròn thứ nhất.
Định lý 1.10. [3] Điều kiện cần và đủ để hai đường tròn trực giao với nhau là có
một đường kính nào đó của một trong hai đường tròn bị đường tròn kia chia điều hoà.
Định nghĩa 1.9. [3] Người ta gọi chùm đường tròn là một tập hợp các đường
tròn kể từng đôi một, nhận một đường thẳng duy nhất làm trục đẳng phương.
Đường thẳng đó gọi là trục đẳng phương của chùm.
Từ định nghĩa trên ta thấy rằng, tâm các đường tròn của một chùm phải nằm
trên một đường thẳng gọi là đường chứa tâm của chùm và đường thẳng này vuông
góc với trục đẳng phương của chùm.
Tải tài liệu tại sividoc.com
Viết đề tài giá sinh viên – ZALO:0973.287.149-TEAMLUANVAN.COM
10
Từ định nghĩa của chùm đường tròn, ta suy ra hai định lý sau đây:
Định lý 1.11. [3] Điều kiện cần và đủ để một tập hợp các đường tròn lập
thành một chùm là có hai điểm mà mỗi điểm đều có cùng phương tích đối với tất cả
các đường tròn của tập hợp đó. Trục đẳng phương của chùm là đường thẳng nối hai
điểm nói trên.
Định lý 1.12. [3] Điều kiện cần và đủ để một tập hợp các đường tròn có tâm
thẳng hàng lập thành một chùm là có một điểm có cùng phương tích đối với tất cả
các đường tròn của tập hợp đó.
Trục đẳng phương của chùm là đường thẳng đi qua điểm nói trên và vuông
góc với đường chứa tâm.
1.4. Cực và đường đối cực
1.4.1. Đường đối cực của một điểm đối với hai đường thẳng cắt nhau
Định nghĩa 1.10. [3] Hai điểm M và N gọi là liên hợp với nhau đối với hai
đường thẳng đồng quy Ox, Oy nếu đường thẳng MN cắt hai đường thẳng đó tại hai
điểm A, B sao cho (MNAB) = -1.
Nếu (MNAB) = -1 thì ta cũng suy ra (ABMN) = -1 và khi đó hai điểm A và B
cũng liên hợp với nhau đối với hai đường thẳng đồng quy OM, ON.
Bài toán. Cho một điểm M không thuộc hai đường thẳng Ox, Oy. Hãy tìm tập
hợp các điểm N liên hợp với M đối với hai đường thẳng đã cho.
Lời giải. Qua M ta kẻ một đường thẳng lần lượt cắt Ox, Oy tại A, B. Ta lấy
trên đường thẳng đó một điểm N sao cho (MNAB) = -1 (Hình 1.8).
P
O
N’
B’
A’ N1
M B
A N
x y
Q
z
Hình 1.8
Tải tài liệu tại sividoc.com
Viết đề tài giá sinh viên – ZALO:0973.287.149-TEAMLUANVAN.COM
11
Nếu kẻ đường thẳng Oz đi qua O và N thì ta có chùm (OM, Oz, Ox, Oy) là
một chùm điều hoà. Do đó, mọi điểm của đường thẳng Oz (trừ hai điểm P và Q) đều
liên hợp với điểm M đối với hai đường thẳng đồng quy Ox, Oy (do hai điểm P và Q
thuộc Oz mà MP // Ox và MQ // Oy ta phải loại ra vì lúc đó các đường thẳng MP và
MQ đều không cắt cả hai đường thẳng Ox và Oy).
Ngược lại, nếu N1 là một điểm không thuộc đường thẳng Oz nói trên thì
không liên hợp với M vì khi đó nếu đường thẳng MN1 cắt Ox, Oy, Oz lần lượt tại A’,
B’, N’ thì ta có: (MN’A’B’) = -1 còn (MN1A’B’) (MN’A’B’) nên (MN1A’B’) -1.
Do đó, điểm N1 không liên hợp với M đối với hai đường thẳng Ox và Oy.
Vậy tập hợp các điểm N liên hợp với điểm M đối với hai đường thẳng Ox, Oy
là đường thẳng Oz loại trừ hai điểm P, Q nói trên.
Định nghĩa 1.11. [3] Đường thẳng Oz trong bài toán trên gọi là đường đối
cực của điểm M đối với hai đường thẳng Ox, Oy. Điểm M gọi là cực của đường
thẳng Oz đối với hai đường thẳng đó.
Nhận xét. Muốn dựng đường đối cực của một điểm M đối với hai đường
thẳng Ox, Oy cho trước, dựa vào tính chất của hình tứ giác toàn phần ta tìm hai
điểm P và Q phân biệt đều cùng liên hợp với M đối với Ox, Oy nói trên. Ta có PQ là
đường đối cực của điểm M đối với Ox, Oy và PQ luôn đi qua điểm O (Hình 1.9a).
O
M
P A O
N
M Q
B
x y
a) b)
Hình 1.9
Tải tài liệu tại sividoc.com
Viết đề tài giá sinh viên – ZALO:0973.287.149-TEAMLUANVAN.COM
12
1.4.2. Đường đối cực của một điểm đối với một đường tròn
Định nghĩa 1.12. [3] Hai điểm M và N gọi là liên hợp với nhau đối với đường
tròn (O), nếu đường tròn đường kính MN trực giao với đường tròn (O) (Hình 1.9b).
Nếu đường thẳng MN cắt đường tròn (O) tại hai điểm A và B thì điều kiện
cần và đủ để M và N liên hợp với nhau đối với đường tròn (O) cho trước là tỉ số kép
(MNAB) = -1. Hai điểm M, N có thể liên hợp với nhau đối với đường tròn (O) mà
đường thẳng MN không cắt đường tròn này.
Bài toán. Cho đường tròn (O) và một điểm M không trùng với tâm O của
đường tròn đó. Hãy tìm tập hợp những điểm N liên hợp của M đối với đường tròn
(O) đã cho.
Lời giải. Nếu N là điểm liên hợp của M đối với đường tròn (O) thì theo định
nghĩa, đường tròn đường kính MN trực giao với đường tròn (O). Khi đó, đường kính
AB đi qua M của đường tròn (O) bị đường tròn đường kính MN chia điều hoà. Gọi H
là giao điểm thứ hai của đường tròn đường kính MN với đường thẳng AB.
Ta có (ABMH) = -1 (Hình 1.10). Trong hàng điểm điều hoà A, B, M và H,
điểm H hoàn toàn được xác định vì ba điểm A, B, M đã được xác định. Mặt khác, do
MN là đường kính nên MH HN. Nói cách
khác, điểm N nằm trên đường thẳng m vuông
góc với đường thẳng MO tại H.
N
Ngược lại, nếu N’ là điểm bất kì của
đường thẳng m thì đường tròn đường kính
MN’ đi qua H và do (ABMH) = -1 nên đường M A HO B
tròn đường kính MN’ trực giao với đường
tròn (O). Vậy điểm N’ liên hợp với M đối
với đường tròn (O). Hình 1.10
Vậy tập hợp điểm N liên hợp với
điểm M đối với một đường tròn (O) cho trước là một đường thẳng m vuông góc với
đường thẳng MO tại H với (MHAB) = -1, trong đó A, B là giao điểm của đường
thẳng MO với đường tròn tâm O.
Tải tài liệu tại sividoc.com
Viết đề tài giá sinh viên – ZALO:0973.287.149-TEAMLUANVAN.COM
13
Định nghĩa 1.13. [3] Đường thẳng m trong bài toán trên gọi là đường đối cực
của điểm M đối với đường tròn (O). Điểm M gọi là cực của đường thẳng m đối với
đường tròn (O) nói trên.
Như vậy, mỗi điểm M không trùng với điểm O của đường tròn tâm O có một
đường đối cực xác định và ngược lại, mỗi đường thẳng không đi qua O có một điểm
cực xác định đối với một đường tròn tâm O cho trước.
Vì (ABMH) = -1 nên đường đối cực m của điểm M đối với đường tròn (O) sẽ
cắt, không cắt hay tiếp xúc với đường tròn tâm O (Hình 1.11a,b,c).
Muốn dựng đường đối cực của một điểm M đối với một đường tròn tâm O
cho trước, ta vẽ qua M hai cát tuyến MAB, MCD (Hình 1.12). Gọi P và Q lần lượt là
các điểm liên hợp với M nghĩa là (ABMP) = -1 và (CDMQ) = -1.
m m m
I R
M AHOB H AMOBHMO
A B
K S
a) b) c)
Hình 1.11
Ta suy ra PQ là đường đối cực của điểm M. Ta có thể dựa vào tính chất của
hình tứ giác toàn phần để tìm các điểm P và Q liên hợp với M đối với A, B và C, D.
Đặc biệt, khi các cát tuyến đó trở H
thành tiếp tuyến thì ba điểm P, A, B
trùng nhau và ba điểm C, Q, D cũng
trùng nhau.
Do đó, muốn dựng đường đối
cực của một điểm M ta thường làm
như sau: M
B
A P
O
C Q D
Hình 1.12
Tải tài liệu tại sividoc.com
Viết đề tài giá sinh viên – ZALO:0973.287.149-TEAMLUANVAN.COM
14
- Nếu điểm M nằm ngoài đường tròn (O) thì từ M ta vẽ hai đường tiếp tuyến
MI, MK với đường tròn, trong đó I và K là hai tiếp điểm. Khi đó, đường thẳng IK là
đường đối cực của điểm M cho trước (Hình 1.11a).
- Nếu điểm M nằm trong đường tròn thì ta vẽ đường thẳng vuông góc với
MO tại M. Đường thẳng này cắt đường tròn tại hai điểm R và S (Hình 1.11b). Các
tiếp tuyến của đường tròn tại R và S cắt nhau tại H. Đường thẳng m vuông góc với
đường thẳng MO tại H là đường đối cực của điểm M cho trước.
- Nếu điểm M nằm trên đường tròn thì tiếp tuyến tại M của đường tròn chính
là đường đối cực của điểm M cho trước (Hình 1.11c).
1.4.3. Các tính chất của cực và đường đối cực đối với một đường tròn
1) Đối với một đường tròn cho trước, nếu đường đối cực của điểm A đi
qua điểm B thì đường đối cực của điểm B đi qua điểm A.
Chứng minh. Nếu điểm B nằm trên đường đối cực a của điểm A thì A và B là
hai điểm liên hợp đối với đường tròn cho trước. Mặt khác ta biết rằng, tập hợp các
điểm liên hợp của điểm B là đường đối cực
b của điểm B đó (Hình 1.13). Vậy điểm A B
phải nằm trên đường đối cực b của điểm B
(vai trò của A và B là bình đẳng).
Ta có: B a A b.
Định nghĩa 1.14. [3] Hai đường b
A
thẳng a, b được gọi là liên hợp với nhau
đối với một đường tròn cho trước nếu
đường này đi qua cực của đường kia.
a
2) Đối với một đường tròn cho trước,
các đường đối cực của các điểm thẳng hàng Hình 1.13
thì đồng quy và các cực của các đường thẳng đồng quy thì thẳng hàng.
Chứng minh. Theo tính chất 1, giả sử các điểm A1, A2…, An nằm trên đường
thẳng b nghĩa là các điểm Ai b với i = 1, 2…, n thì điểm B thuộc các đường thẳng
b và ai là các đường đối cực của các điểm Ai. Vậy các đường đối cực của các điểm
Ai đều đồng quy tại B.
Phần còn lại chứng minh tương tự.
Tải tài liệu tại sividoc.com
Viết đề tài giá sinh viên – ZALO:0973.287.149-TEAMLUANVAN.COM
15
1.4.4. Phép đối cực
Trên mặt phẳng cho một đường tròn cơ sở (C). Giả sử có một hình H gồm
các điểm và các đường thẳng. Với mỗi điểm của hình H đều có các đường đối cực
của nó đối với đường tròn (C), với mỗi đường thẳng của hình H có các điểm là cực
của nó.
Hình H' là tập hợp các đường thẳng (gồm các đường đối cực của các điểm
thuộc hình H) và các điểm (gồm các cực của các đường thẳng thuộc hình H). Khi
đó, ta nói có một phép đối cực với đường tròn cơ sở (C) biến hình H thành hình H'.
Rõ ràng muốn chứng minh tính thẳng hàng của các điểm trên hình H ta chỉ việc
chứng minh tính đồng quy của các đường thẳng tương ứng trên hình H'.
Ví dụ 1.1. [3] (Định lý Bri-ăng-xông) Ba đường thẳng nối các cặp đỉnh đối
diện của một lục giác ngoại tiếp một đường tròn đồng quy tại một điểm.
Lời giải. Giả sử ABCDEF là lục giác ngoại tiếp đường tròn (O). Gọi M, N, P,
Q, K, I lần lượt là các tiếp điểm của các cạnh AB, BC, CD, DE, EF, FA với đường
tròn (C). Khi đó, theo định lý Pát-xcan:
B
MN KQ= M N
QP IM= , , thẳng hàng. A C
PN IK=
P
Hiển nhiên,là cực của BE,là cực của I
AD, là cực của CF. Vì , , thẳng hàng nên BE, F D
Q
K
AD, CF đồng quy tại một điểm. Ta có phép đối cực E
biến ba điểm , , thành ba đường thẳng BE, AD, Hình 1.14
CF (Hình 1.14).
Định lý 1.13. [3] Phép đối cực bảo tồn tỉ số kép, nghĩa là qua phép đối cực,
một chùm bốn đường thẳng (đồng quy) biến thành bốn điểm và tỉ số kép của bốn
điểm này bằng tỉ số kép của bốn đường thẳng đó.
Hệ quả. Phép đối cực biến một chùm đường thẳng điều hoà thành một hàng
điểm điều hoà và ngược lại.
Tải tài liệu tại sividoc.com
Viết đề tài giá sinh viên – ZALO:0973.287.149-TEAMLUANVAN.COM
16
Như vậy, phép đối cực là một công cụ tương đối hiệu quả trong việc chuyển
đổi hai dạng bài toán chứng minh đồng quy và chứng minh thẳng hàng, chuyển từ
chùm đường thẳng điều hòa sang hàng điểm điều hòa và ngược lại.
1.5. Cách xác định cực và đường đối cực
* Trường hợp 1: Khi cực S ở ngoài đường tròn (O). Ta có 2 cách dựng sau:
- Cách 1: Từ S kẻ tới (O) hai tiếp tuyến SA, SB (A, B là tiếp điểm). Khi đó
đường đối cực của S đối với (O) là AB.
A
S O .
B
Hình 1.15
- Cách 2: Từ S kẻ tới (O) hai cát tuyến SAB, SCD. Giả sử AD cắt BC ở F, AC
cắt BD ở E. Khi đó đường đối cực của điểm S đối với (O) là đường thẳng EF.
S
F
B
A
E
D
C
Hình 1.16
* Trường hợp 2: Khi điểm S nằm trong đường tròn (O). Ta có 2 cách dựng
sau đây:
Tải tài liệu tại sividoc.com
Viết đề tài giá sinh viên – ZALO:0973.287.149-TEAMLUANVAN.COM
17
- Cách 1: Qua điểm S dựng đường vuông góc với OS, đường này cắt (O) tại
hai điểm A, B. Tiếp tuyến của (O) tại A, B cắt nhau ở điểm P. Khi đó đường đối cực
của điểm S đối với đường tròn (O) là đường thẳng qua P vuông góc với OS.
A
P
S
O
B
Hình 1.17
- Cách 2: Qua điểm S dựng hai dây cung AB và CD. Giả sử AD cắt BC ở E,
AC cắt BD ở F. Khi đó đường đối cực của điểm S đối với (O) là EF.
E
C
A
S
.O
F
D B
Hình 1.18
* Trường hợp 3: Điểm S nằm trên đường tròn (O). Khi đó, tiếp tuyến của (O)
tại S chính là đường đối cực của S đối với (O).
Tải tài liệu tại sividoc.com
Viết đề tài giá sinh viên – ZALO:0973.287.149-TEAMLUANVAN.COM
18
O.
S
Hình 1.19
Chương 1 của luận văn trình bày các khái niệm cơ bản như hàng điểm điều
hòa, chùm đường thẳng, chùm đường thẳng điều hòa và tứ giác toàn phần. Đây là
những nội dung có liên quan đến hàng điểm điều hòa. Chúng ta có thể chứng minh
hàng điểm điều hòa dựa trên các tính chất của chùm đường thẳng điều hòa và tứ
giác toàn phần. Kiến thức về đường tròn trực giao, cực và đường đối cực đối với hai
đường thẳng đồng quy và đối với đường tròn cũng như cách dựng đường đối cực
của một điểm cho trước. Với cực và đường đối cực ta có thể đưa ra cách nhìn xuyên
suốt, nhất quán đối với một số dạng toán như chứng minh quan hệ vuông góc,
chứng minh các điểm thẳng hàng, chứng minh quan hệ đồng quy,... Các bài toán về
cực và đường đối cực thường gặp ở bậc trung học phổ thông là cực và đường đối
cực của một điểm đối với đường tròn hoặc đối với cặp đường thẳng cắt nhau. Đặc
biệt, phép đối cực được trình bày cho chúng ta một công cụ trong việc chuyển đổi
bài toán chứng minh thẳng hàng và bài toán chứng minh đồng quy. Trong chương 2
luận văn sẽ khai thác một số lớp bài toán sử dụng đến khái niệm cực và đường đối
cực để chứng minh các điểm thẳng hàng, chứng minh các đường thẳng đồng quy và
giải bài toán tìm điểm cố định.
Tải tài liệu tại sividoc.com
Viết đề tài giá sinh viên – ZALO:0973.287.149-TEAMLUANVAN.COM
19
Chương 2
SỬ DỤNG HÀNG ĐIỂM ĐIỀU HÒA TRONG GIẢI TOÁN
HÌNH HỌC PHẲNG
Để có thể sử dụng hàng điểm điều hòa trong giải toán, chúng ta cần phải
nhận ra các hàng điểm điều hòa trong bài toán, đặc biệt là vận dụng linh hoạt các
tính chất trong tứ giác toàn phần, tứ giác điều hòa, chùm phân giác,… Dưới đây là
một số minh họa cách tìm các hàng điểm điều hòa trong một bài toán cụ thể.
2.1. Chứng minh hàng điểm điều hòa
Để chứng minh bốn điểm lập thành hàng điểm điều hòa chúng ta có thể sử
dụng định nghĩa, nghĩa là chứng minh tỉ số kép của bốn điểm bằng -1. Các định lý
thường được áp dụng trong giải dạng toán này là định lý Xêva, định lý Mênêlauýt,
hệ thức Niu-tơn và hệ thức Đề-các về hàng điểm điều hòa.
Ví dụ 2.1. [4] Cho tam giác ABC. Lấy E trên BC, điểm F trên AC và điểm K
trên AB sao cho AE, BF, CK đồng quy tại một điểm. Gọi T là giao điểm của FK với
BC. Chứng minh rằng (TEBC) = -1.
Giải. Bài toán có giả thiết về các đường thẳng đồng quy trong tam giác, vì
vậy định lý Xêva, định lý Mênêlauýt được sử dụng trong bài toán này. Thật vậy,
trongABC, áp dụng định lý Xêva với ba đường đồng quy AE, BF, CK ta có:
EB
.
FC
.
KA
1 (1)
EC FA KB
Mặt khác, áp định lý Mênêlauýt với ba điểm thẳng hàng T, K, F ta lại có:
TC
.
KB
.
FA
1 (2)
TB KA FC
Hình 2.1
Tải tài liệu tại sividoc.com
Viết đề tài giá sinh viên – ZALO:0973.287.149-TEAMLUANVAN.COM
20
Nhân (1) và (2) vế theo vế suy ra: TC
TB
 EC
EB
hay (TEBC) = -1.
* Nhận xét: Nếu gọi I là điểm đồng quy của AE, BF, CK thì AIBC là một tứ
giác toàn phần với các đường chéo AI, FK và BC mà lời giải là một trong những
cách chứng minh cho định lý rất đẹp về hình tứ giác toàn phần: “Trong một hình tứ
giác toàn phần, một đường chéo bị hai đường chéo còn lại chia điều hòa”. Bài toán
đơn giản này cho ta sử dụng tính chất một hình tứ giác toàn phần hay hàng điểm
điều hòa cho một tam giác có ba đường thẳng đồng quy.
Ví dụ 2.2. [4] Cho tứ giác ABCD ngoại tiếp đường tròn tâm (O). Gọi M, N, P,
Q lần lượt là các tiếp điểm trên các cạnh AB, BC, CD, DA với đường tròn. Gọi K là
giao điểm của đường thẳng MQ với NP và I là giao điểm của đường thẳng MP với
QN. Chứng minh rằng (DBIK) = -1.
Giải. Bài toán có giả thiết về các tiếp điểm của đường tròn với các cạnh của
tam giác, vì vậy định lý Mênêlauýt được sử dụng, từ đó xuất hiện các tỉ số giữa các
đoạn thẳng và có thể được sử dụng để chứng minh hàng điểm điều hòa theo định
nghĩa. Áp dụng định lý Mênêlauýt cho tam giác ABD với 3 điểm thẳng hàng K, M,
KB QD MA
1
KB

MB
Q ta có: . . hay (vì QA = MA) (1)
KD QA MB KD QD
Mặt khác, ta có thể chứng minh được:
MB

IB
(2)
QD ID
A K
M
Q
B
O .
I N
D P C
Hình 2.2
Tải tài liệu tại sividoc.com
Viết đề tài giá sinh viên – ZALO:0973.287.149-TEAMLUANVAN.COM
21
Từ (1) và (2) suy ra KD
KB
 ID
IB
(Hình 2.2). Vì I nằm trong đoạn thẳng BD
và K nằm ngoài đoạn thẳng BD nên ta suy ra KD
KB
 ID
IB
. Vậy (DBIK) = -1.
Ví dụ 2.3. [2] ChoABC không cân tại A, phân giác trong AD, đường cao
AH. Gọi E, F là hình chiếu của D trên AB, AC. Kẻ đường thẳng EF cắt đường thẳng
BC tại điểm L. Chứng minh rằng (HLBC) = -1.
Giải. Tương tự ví dụ 2.2, bài toán này sử dụng định lý Mênêlauýt như sau:
A
E
F
B D H C L
Hình 2.3
XétABC, ta cần chứng minh: FC  EA  HB 1.
FA EB HC
Các tứ giác EAHD, FADH nội tiếp đường tròn
BE.BA BH.BD (1)

CD.CH CF.CA (2)
Từ (1) và (2) suy ra BH . BD BE . BA  BH  BE  HB .CF 1 (3)
CH
CH CD CF CA CF HC BE
Mà AD là phân giácBAC nên AE = AFAE 1 (4)
AF
Từ (3) và (4) ta có HB . CF . AE  1HB . FC . EA 1 (5)
HC BE AF HC FA EB
FC EA LB
XétABC với cát tuyến EFL, ta có . . 1 (6)
FA EB LC
(áp dụng định lý Mênêlauýt trong mặt phẳng).
Tải tài liệu tại sividoc.com
Viết đề tài giá sinh viên – ZALO:0973.287.149-TEAMLUANVAN.COM
22
Từ5 và6 ta có HC
HB
. LC
LB
1 hay (HLBC) = -1.
Ví dụ 2.4. [4] Cho điểm A nằm ngoài đường tròn (O). Từ điểm A kẻ hai tiếp
tuyến AB, AC đến đường tròn (O) (với B, C là hai tiếp điểm). Đường thẳng AO cắt
đường tròn (O) tại E, F và cắt cạnh BC tại điểm K. Chứng minh rằng (AKEF) = -1.
Giải. Trong bài toán này, chúng ta nhận thấy xuất hiện các tam giác vuông.
Do đó, ta có thể sử dụng hệ thức lượng trong tam giác vuông. Các hệ thức này có
quan hệ với hệ thức Niu-tơn về hàng điểm điều hòa. Đó cũng là một ý tưởng để
chứng minh hàng điểm điều hòa.
Ta có OB2
= OK.OA (hệ thức lượng tam giác vuông) (1)
Mặt khác ta lại có: OB2
= OE2
= OF2
(2)
Từ (1) và (2) ta suy ra: OE2
= OF2
= OK.OA. Từ đó suy ra điều phải chứng minh
(Hình 2.4) .
B
F
O K E
A
C
Hình 2.4
Ví dụ 2.5. [2] Cho hình vuông và một đường tròn tâm O nội tiếp hình vuông.
Một tiếp tuyến bất kỳ của đường tròn cắt các cặp cạnh đối của hình vuông tại A, B
và C, D. Chứng minh rằng (ABCD) = - 1.
Giải. Bài toán xuất hiện các đường phân giác của một góc. Điều này gợi ý
cho việc sử dụng các chùm phân giác trong chứng minh hàng điểm điều hòa.
Tải tài liệu tại sividoc.com
Viết đề tài giá sinh viên – ZALO:0973.287.149-TEAMLUANVAN.COM
23
* Cách 1: Ta có OD là phân giác củaGOF, OC là phân giác củaFOI mà
ta lại cóGOF +FOI = 180o
nên OD OC (1)
Ta có OA là phân giác củaEOF, OD là phân giác củaFOG. Từ đó suy ra
rằngAOD =AOF +FOD =
1
2EOF +
1
2FOG = 450
.
Từ điều trên và (1) suy ra OA là phân giác củaCOD.
Tương tự, ta chứng minh được OA OB (Hình 2.5).
Như vậy: OA, OB, OC, OD là một chùm đường thẳng điều hòa.
Từ đó suy ra (ABCD) = - 1.
C
M A E
F
D
G
O
B
Q H
N
I
P
Hình 2.5
* Cách 2: Xét chùm đường thẳng FE, FI, FH, FG.
Ta có: số đo cung EI = số đo cung IH FI là phân giácEFH
số đo cung IH = số đo cung HG FH là phân giácIFG Suy
ra FE, FI, FH, FG là chùm đường thẳng điều hòa.
Mặt khác: FE OA, FI OC, FH OB, FG OD. Từ đó suy ra các đường
thẳng OA, OB, OC, OD cũng là chùm phân giác nên nó là chùm đường thẳng điều
hòa, suy ra (ABCD) = - 1.
Tải tài liệu tại sividoc.com
Viết đề tài giá sinh viên – ZALO:0973.287.149-TEAMLUANVAN.COM
24
Ví dụ 2.6. [2] Cho đường tròn tâm O, điểm M nằm ngoài đường tròn. Gọi
MA, MB là hai tiếp tuyến với đường tròn (A, B là các tiếp điểm) và cát truyến MCD
với đường tròn (C, D thuộc đường tròn tâm O). Chứng minh rằng AM, AB, AC, AD
là chùm đường thẳng điều hòa.
Giải. GọiI AB MO , OM cắt (O) tại C’, D’ (Hình 2.6).
Ta có MO AB và cung AC’ = cung BC’.
GọiH ABCD . Ta có (NIC’D’) = - 1 D(NIC’D’) = - 1. Từ đó suy
ra DC’ là phân giác củaMDI (vì C’D DD’). Vậy cung CC’ = cung C’K (với
K DI(O) ). Như vậy, các điểm C, A đối xứng với K, B qua đường thẳng MO. Vì
thếCIA =KIB, màKIB =AID (hai góc đối đỉnh) nênCIA =AID hay IA là
phân giác củaCID. Mặt khác IM IA. Vậy IM, IA, IC, ID là chùm phân giác nên nó
là chùm đường thẳng điều hòa. Từ đó suy ra (MHCD) = -1 hay AM, AB, AC, AD là
chùm đường thẳng điều hòa.
A
H
D
C
I
.
O
M C’ D’
K
B
Hình 2.6
* Nhận xét: Từ bài toán trên, ta có kết quả sau đây: “Với mỗi cát tuyến MCD
cắt đường nối hai tiếp điểm A, B của đường tròn tại điểm H thì ta có hàng điểm điều
hòa: (MHCD) = - 1”.
Ví dụ 2.7. [2] ChoABCnội tiếp đường tròn (O), tiếp tuyến tại A và B của
(O) cắt nhau tại I. Một đường thẳng d đi qua I cắt AC, BC lần lượt tại M và N, cắt
đường tròn (O) tại P, Q. Chứng minh rằng (MNPQ) = -1.
Tải tài liệu tại sividoc.com
Viết đề tài giá sinh viên – ZALO:0973.287.149-TEAMLUANVAN.COM
25
Giải. Bài toán xuất hiện các tiếp tuyến từ một điểm đến đường tròn, do đó sẽ
xuất hiện các đường đối cực của một điểm nào đó đối với đường tròn. Chúng ta có
thể khai thác yếu tố “cực” và “đường đối cực” trong bài toán này.
A
D
I Q
O
. C
T
N P
M
B E
Hình 2.7
Giải. Dựng các tiếp tuyến MD, ME. Giả sử N ' DE BC . Áp dụng định lý
Briăng-xông cho lục giác AEDDBC có: AE DB T , ED BC N ', DD CA M .
Khi đó M, N’, T thẳng hàng, từ đó suy ra PQ, BC, DE đồng quy.
Mà PQ BC N N N ' . Do đó N ED suy ra (MNPQ) = -1 (tính chất
“cát tuyến cắt đường nối hai tiếp điểm” với MD, ME là hai tiếp tuyến). Từ đó ta có
điều phải chứng minh.
2.2. Chứng minh vuông góc
Cực và đường đối cực là một công cụ hiệu quả trong chứng minh quan hệ
vuông góc trong mặt phẳng. Dưới đây chúng tôi minh họa một số ví dụ về khai thác
các tính chất của cực và đường đối cực trong giải bài toán chứng minh vuông góc.
Ví dụ 2.8. [4] Giả sử đường tròn (O) với tâm O và bán kính R. Qua điểm M
nằm trong đường tròn (M khác điểm O) vẽ hai dây cung CD và EF không đi qua
tâm O. Hai tiếp tuyến tại C, D của (O) cắt nhau tại điểm A, hai tiếp tuyến tại E, F
của (O) cắt nhau tại điểm B. Chứng minh rằng OM và AB vuông góc với nhau.
Tải tài liệu tại sividoc.com
Viết đề tài giá sinh viên – ZALO:0973.287.149-TEAMLUANVAN.COM
26
Giải. Bài toán có hai tiếp tuyến
với đường tròn với yêu cầu chứng minh
vuông góc. Điều này giúp ta liên tưởng
đến đường đối cực của một điểm đối
với một đường tròn. Ta thấy đường đối
cực của điểm A là đườngthẳng CD đi
qua M nên đường đối cực của điểm M
sẽ đi qua điểm A (Hình 2.8). Tương
Hình 2.8
tự, đường đối cực của điểm M đi qua
điểm B. Vậy, đường thẳng AB chính là đường đối cực của điểm M. Do đó, AB
vuông góc với OM.
Ví dụ 2.9. [4] Cho tam giác ABC cân tại A. Hai đường thẳng d1, d2 bất kì qua
điểm A. Các đường thẳng đi qua B, C tương ứng vuông góc với d1, d2 cắt nhau tại
D. Đường thẳng đi qua B vuông góc với AB cắt d1 tại E, đường thẳng đi qua C
vuông góc với AC cắt d2 tại F. Chứng
minh rằng AD vuông góc với EF.
Giải. Bài toán này không xuất
hiện đường tròn nhưng ta để ý thấy yếu
tố “cân” trong tam giác ABC. Vậy, có
đường tròn tâm A, bán kính AB đi qua B
và C (Hình 2.9).
Dễ nhận thấy BE, CF lần lượt là
các tiếp tuyến của đường tròn (A; AB).
Đường đối cực của điểm E sẽ đi qua Hình 2.9 điểm B và vuông
góc với AE hay d3.
Tương tự, đường đối cực của điểm F sẽ đi qua điểm C và vuông góc với CF hay d4.
Vậy, cực của đường thẳng EF đối với đường tròn (A; AB) chính là điểm D. từ đó
suy ra AD vuông góc với EF.
Tải tài liệu tại sividoc.com
Viết đề tài giá sinh viên – ZALO:0973.287.149-TEAMLUANVAN.COM
27
Ví dụ 2.10. [4] Cho tam giác ABC với các đường cao BB’, CC’. Gọi E, F lần
lượt là trung điểm của AC, AB. Đường thẳng EF cắt đường thẳng B’C’ tại điểm K.
Chứng minh rằng AK vuông góc với đường thẳng Ơle của tam giác ABC.
Giải. Ta xét cực và đường đối cực đối với đường tròn Ơle của tam giác ABC
(đường tròn tâm O9). Gọi I là giao điểm của FB’ và EC’, G là giao điểm của CF và
BE, H là giao điểm của BB’ và CC’. Áp dụng định lý Pa-puýt cho hai bộ ba điểm
(F, C’, B) và (E, B’, C) ta suy ra ba điểm H, G, I thẳng hàng (Hình 2.10). Do đó, O9I
là đường thẳng Ơle của tam giác ABC (1)
Hình 2.10
Mặt khác, chú ý E, F, B’, C’ cùng nằm trên đường tròn (O9) nên suy ra AK chính là
đường đối cực của điểm I. Vậy, O9I vuông góc với AK (2) Từ (1) và (2) ta suy ra
điều phải chứng minh.
Ví dụ 2.11. [4] Cho tam giác ABC nội tiếp đường tròn (O; R). Các đường
phân giác trong BE, CF của các góc B, góc C cắt lại (O) lần lượt tại M, N. Đường
thẳng qua điểm M vuông góc với BM cắt đường thẳng đi qua N vuông góc với CN
tại điểm S. Chứng minh rằng SO vuông góc với EF.
Giải. Trước hết ta tìm đường đối cực của điểm S đối với đường tròn (O) và
chứng minh rằng nó song song với EF. Các đường thẳng SN, SM cắt lại (O) lần lượt
tại L, G. Khi đó, ta dễ thấy C, O, G thẳng hàng và B, O, L thẳng hàng. Tiếp tuyến tại
Tải tài liệu tại sividoc.com
Viết đề tài giá sinh viên – ZALO:0973.287.149-TEAMLUANVAN.COM
28
G và N của (O) cắt nhau tại điểm Q, tiếp tuyến của tại L và M của (O) cắt nhau tại
điểm P. Đường thẳng OP cắt LM tại điểm H, đường thẳng OQ cắt NG tại điểm K.
Ta thấy, đường đối cực của điểm Q là đường thẳng GN đi qua S nên đường
đối cực của điểm S đi qua điểm Q. Tương tự, đường đối cực của điểm S cũng đi qua
điểm P. Do đó, đường đối cực của điểm S là PQ (Hình 2.11).
Ta chứng minh PQ // EF. Thật vậy, ta thấy IE // OP, IF // OQ nên để chứng
minh PQ // EF ta chỉ ra góc lượng giácFI , FE (QO, QP ) k 2 .
Hình 2.11
Mặt khác, ta nhận thấy OK .OQ OG 2
 OL2
 OH .OP .Từ đó suy ra 5 điểm Q,
K, H, P cùng nằm trên đường tròn vàQO, QP ( HK , HO) k 2 .Từ đó suy ra
ta cần chỉ raFI , FE ( HK , HO) k 2 (1)
Kẻ ID, IV lần lượt vuông góc với AC, AB và chú ý rằng:
ID
sinIFV
IE 
sinIED (vì ID = IV)
IF sin IED
IV
sinIFV
Tải tài liệu tại sividoc.com
Viết đề tài giá sinh viên – ZALO:0973.287.149-TEAMLUANVAN.COM
29
 C
sin A 
CM OK
 2 sinNAC
  (định lý hàm số sin) (2)
 Bsin MAB BM OH
sin A 
 2
Ta lại có IE // OH, IF // OK nênFI , FE ( HK , HO) k 2 (3)
Từ (2) và (3) suy ra tam giác IEF đồng dạng với tam giác OKH. Do đó, (1)
đúng nên suy ra điều phải chứng minh.
Ví dụ 2.12. [4] Cho tứ giác ABCD ngoại tiếp đường tròn (I) và nội tiếp
đường tròn (O). Tiếp điểm của đường tròn (I) trên các cạnh AB, BC, CD, DA lần
lượt là M, N, P, Q. Chứng minh rằng MP vuông góc với NQ.
Giải. Trường hợp tứ giác ABCD có ít nhất một cặp cạnh song song thì đơn
giản. Ta sẽ giải bài toán trong trường hợp còn lại.
Hình 2.12
Xét cực và đường đối cực đối với đường tròn (I) (Hình 2.12). Đường thẳng
AB cắt đường thẳng CD tại điểm E, đường thẳng AD cắt đường thẳng BC tại điểm
F. Ta thấy cực của đường thẳng MP là điểm E, cực của đường thẳng NQ là điểm F.
Để giải bài toán ta chỉ cần chứng minh IE và IF vuông góc với nhau. Thật vậy, IE,
IF lần lượt là phân giác của các gócAED,AFB. Gọi giao điểm của IF với AB và
CD lần lượt là S, V thì ta cần chứng minh tam giác ESV cân tại điểm E.
Tải tài liệu tại sividoc.com
Viết đề tài giá sinh viên – ZALO:0973.287.149-TEAMLUANVAN.COM
30
Ví dụ 2.13. [4] Cho tứ giác ABCD ngoại tiếp đường tròn tâm (O). Gọi M, N,
P, Q lần lượt là các tiếp điểm trên các cạnh AB, BC, CD, DA với đường tròn. Gọi K
là giao điểm của MQ với NP. Chứng minh rằng OK vuông góc với AC.
Giải. Bài toán xuất hiện các tiếp tuyến từ một điểm đến đường tròn, từ đó ta
dễ dàng nhận thấy đường thẳng AC là đường đối cực của điểm K và đường thẳng
QK là đường đối cực của điểm A. Do đó, gọi E và F là hai giao điểm của AC với
đường tròn (O). Hai tiếp tuyến qua E và F với đường tròn (O) cắt nhau tại K’. Dễ
dàng chứng minh được rằng các điểm K’, N, P thẳng hàng và K’, M, Q thẳng hàng
(Hình 2.13). Từ đó suy ra K’ là giao điểm của MQ với NP hay K’ K. Vậy KE, KF
là hai tiếp tuyến kẻ từ K với đường tròn (O). Từ đó suy ra KO EF hay KO AC.
K
A
E
Q
M
B
O . N
F
D P C
Hình 2.13
Ví dụ 2.14. [2] Cho tứ giác MNPQ nội tiếp đường tròn (O). Đặt K = QM PN,
L = MN QP, I = MP QN. Chứng minh rằng I là trực tâm của tam giác KOL.
Giải. Phân tích tương tự ví dụ 2.13 ta thấy xuất hiện các cực và đường đối
cực trong bài toán này, do đó sẽ có các điểm cùng nằm trên một đường thẳng.
Tải tài liệu tại sividoc.com
Viết đề tài giá sinh viên – ZALO:0973.287.149-TEAMLUANVAN.COM
31
K
A M
B
N
O.I C
Q
P L
D
Hình 2.14
Kẻ bốn tiếp tuyến đi qua M, N, P, Q với đường tròn (O). Các tiếp tuyến này
cắt nhau tại bốn điểm là A, B, C, D. Dễ thấy I là giao điểm của AC với BD.
Mặt khác, ta thấy BD OL nên suy ra D, B, K thẳng hàng. Suy ra KI OL
và LI KO hay I là trực tâm củaKOL (Hình 2.14).
2.3. Chứng minh song song
Ví dụ 2.15. [4] Cho tam giác ABC có đường tròn nội tiếp là (I). Tiếp điểm
của (I) trên các cạnh BC, CA, AB lần lượt là D, E, F. Đường thẳng AD cắt lại đường
tròn (I) tại điểm M. Đường thẳng đi qua M vuông góc với đường thẳng AD cắt EF
tại điểm N. Chứng minh rằng AN song song với BC.
Giải. Xét cực và đường đối cực đối với đường tròn (I). Gọi P là giao điểm
thứ hai của MN với (I). Dễ thấy D, P, I thẳng hàng. Đường thẳng EF cắt IP, IA lần
lượt tại điểm J, G.
Ta thấy AM . AD AE 2
 AG. AI (Hình 2.15). Ta suy ra các điểm M, G, I, D
cùng nằm trên đường tròn. Do đó: góc lượng giác
Tải tài liệu tại sividoc.com
Viết đề tài giá sinh viên – ZALO:0973.287.149-TEAMLUANVAN.COM
32
(GM , GF ) (GA, GF ) (GA, GM ) k
2 ( DI , DM )
 ( MD, MP) ( DI , DM ) k ( PM , PD) k
Từ đó, suy ra tứ giác MGJP nội tiếp. Ta có: NJ . NG NP.NM NE.NF .
Hình 2.15
Chú ý rằng G là trung điểm của FE nên ta suy ra (NJEF) = -1 hay N thuộc đường
đối cực của điểm J (1)
Mặt khác, đường đối cực của điểm A là EF đi qua J nên đường đối cực của điểm J
đi qua A (2)
Từ (1) và (2) suy ra đường đối cực của điểm J là đường thẳng AN. Vậy IJ vuông
góc với AN, mà IJ vuông góc với BC từ đó suy ra điều phải chứng minh.
Ví dụ 2.16. [4] Cho hai đường thẳng a và a’ cắt nhau tại A và giả sử trên a ta
có bốn điểm A, B, C, D sao cho (ABCD) = -1 và trên a’ có bốn điểm A, B’, C’, D’
sao cho (AB’C’D’) = -1. Chứng minh rằng các đường thẳng BB’, CC’, DD’ hoặc
song song với nhau hoặc đồng quy.
Giải. Bài toán sử dụng đến các chùm đường thẳng song song hoặc chùm các
đường thẳng đồng quy. Tỉ số kép không đổi của một chùm đường thẳng được khai
thác trong ví dụ này.
Tải tài liệu tại sividoc.com
Viết đề tài giá sinh viên – ZALO:0973.287.149-TEAMLUANVAN.COM
33
* Nếu BB’ và CC’ cắt nhau tại O, giả sử tia OD cắt đường thẳng a’ tại D’’.
Vì (ABCD) = -1 O(ABCD) = -1 O(AB’C’D’) = -1 (AB’C’D’’) = -1.
O
a’
D’
C’ B’
A
C
B
D a
Hình 2.16
Mặt khác, theo giả thiết (AB’C’D’) = -1 nên D’ trùng với D’’.
Vậy các đường thẳng BB’, CC’, DD’ đồng quy tại O.
* Nếu BB’ và CC’ song song, từ A và D ta vẽ các đường thẳng song song với
CC’ và BB’. Đường thẳng song song đi qua D cắt AB’ tại D’’, ta chứng minh D’’
trùng với D’. Từ đó, ta suy ra các đường thẳng BB’, CC’, DD’ song song.
2.4. Chứng minh thẳng hàng
Cực và đường đối cực là công cụ hữu hiệu trong chứng minh thẳng hàng.
Thật vậy, chúng ta có thể sử dụng tính chất “cực của các đường thẳng đồng quy thì
thẳng hàng” hoặc quỹ tích các điểm liên hợp với một điểm cho trước để chỉ ra
chúng cùng nằm trên đường đối cực của điểm đó.
Ví dụ 2.17. [4] Cho một điểm A cố định và một đường thẳng d cố định không
đi qua A. Gọi O là hình chiếu vuông góc của A trên d và I là trung điểm của đoạn
thẳng AO. Trên đường thẳng d ta lấy hai điểm thay đổi P và Q không trùng với O.
Dựng các đường thẳng Px và Qy vuông góc với d. Đường thẳng QI cắt AP và Px lần
lượt tại M và N. Đường thẳng PI cắt AQ và Qy lần lượt tại M’ và N’.
a) Chứng minh (QMIN) = -1, (PM’IN’) = -1.
b) Chứng minh ba điểm N, A, N’ thẳng hàng.
Tải tài liệu tại sividoc.com
Viết đề tài giá sinh viên – ZALO:0973.287.149-TEAMLUANVAN.COM
34
Giải. Trong bài toán này, chúng ta cần chỉ ra các điểm N, A, N’ cùng nằm
trên đường đối cực của điểm I đối với hai đường thẳng đồng quy AP, AQ.
a) Ta có chùm (PQ, PM, PI, PN) là một chùm điều hoà vì có cát tuyến AIO
song song với PN và AI = IO. Do đó (QMIN) = -1.
Tương tự ta có chùm (QP, QM’, QI, QN’) là chùm điều hoà. Suy ra, ta có
(PM’IN’) = -1 (Hình 2.17).
y
N’
d
A
M’ I
M
x
N
Q O P
Hình 2.17
b) Vì (QMIN) = -1 nên AN là đường đối cực của điểm I đối với hai đường
thẳng cắt nhau AP và AQ.Tương tự (PM’IN’) = -1 nên AN’ là đường đối cực của
điểm I đối với hai đường thẳng cắt nhau AP và AQ, từ đó suy ra các điểm N, A, N’
cùng thuộc đường đối cực của điểm I đối với hai đường thẳng AP và AQ hay ba
điểm N, A, N’ thẳng hàng.
Ví dụ 2.18. [4] Cho tam giác ABC ngoại tiếp đường tròn (I). Tiếp điểm của
đường tròn (I) trên các cạnh BC, CA, AB lần lượt là D, E, F. Gọi M, N, P lần lượt là
điểm chung của các cặp đường thẳng (EF; BC), (DF; CA), (DE; AB). Chứng minh
rằng các điểm M, N, P thẳng hàng.
Giải. Bài toán xuất hiện các đường tròn và các tiếp tuyến đến đường tròn
này. Do vậy, tính chất của cực và các đường đối cực của một điểm đối với một
đường tròn sẽ được sử dụng trong chứng minh thẳng hàng. Thật vậy, đường đối cực
Tải tài liệu tại sividoc.com
Viết đề tài giá sinh viên – ZALO:0973.287.149-TEAMLUANVAN.COM
35
của điểm A đối với đường tròn (I) là EF đi qua điểm M nên đường đối cực của điểm
M đi qua điểm A (Hình 2.18).
Dễ thấy, đường đối cực của điểm M đi qua điểm D nên suy ra đường đối cực
của điểm M đối với đường tròn (I) là đường thẳng AD. Tương tự, ta có đường đối
cực của điểm N là đường thẳng BE, đường đối cực của điểm P là đường thẳng CF.
Áp dụng định lý Xêva ta chứng minh được các đường thẳng AD, BE, CF đồng quy
nên ba điểm M, N, P thẳng hàng.
Hình 2.18
Ví dụ 2.19. [4] Cho tam giác ABC và một điểm O. Các đường thẳng đi qua O
và vuông góc với OA, OB, OC theo thứ tự cắt BC, CA, AB tại M, N, P. Chứng minh
rằng M, N, P thẳng hàng.
Giải. Phân tích tương tự ví dụ 3.18, gọi A’, B’, C’ lần lượt là cực của các
đường thẳng BC, CA, AB đối với đường tròn (O; R) với R > 0. Do BC, CA, AB
không đồng quy nên A’, B’, C’ không thẳng hàng.
Tải tài liệu tại sividoc.com
Viết đề tài giá sinh viên – ZALO:0973.287.149-TEAMLUANVAN.COM
36
Hình 2.19
Vì đường đối cực của B’ đi qua điểm A nên B’ thuộc đường đối cực của điểm
A đối với (O). Tương tự đường đối cực của C’ đi qua A nên đường đối cực của A đi
qua C’. Từ đó, suy ra đường đối cực của điểm A đối với đường tròn (O) chính là
B’C’. Tương tự ta có C’A’, A’B’ tương ứng là đường đối cực của hai điểm B, C đối
với đường tròn (O) (Hình 2.19).
Vì đường đối cực của M vuông góc với OM, OA OM nên đường đối cực
của M song song với AO. Mà AO vuông góc với đường đối cực của điểm A nên
đường đối cực của điểm M vuông góc với B’C’ (1). Vì M BC là đường đối cực
của điểm A’ nên điểm A’ thuộc đường đối cực của điểm M (2). Từ (1) và (2) suy ra
đường đối cực của điểm M là đường cao trong tam giác A’B’C’. Tương tự, các
đường đối cực của điểm N, P đối với (O) cũng là đường cao trong tam giác A’B’C’
suy ra chúng đồng quy (Hình 2.19). Vậy các điểm M, N, P thẳng hàng.
Ví dụ 2.20. [4] Cho tam giác ABC có (I) là đường tròn nội tiếp. Gọi D, E, F
lần lượt là các tiếp điểm của (I) trên các cạnh BC, CA, AB. Gọi D’, E’, F’ lần lượt là
các giao điểm của các đường thẳng EF với BC, FD với CA, DE với AB. Chứng
minh rằng D’, E’, F’ thẳng hàng.
Giải. Ta thấy EF là đường đối cực của A đối với (I) mà D’ EF nên điểm A
thuộc đường đối cực của điểm D’ đối với (I). Do D’D là tiếp tuyến với (I) nên AD là
Tải tài liệu tại sividoc.com
Viết đề tài giá sinh viên – ZALO:0973.287.149-TEAMLUANVAN.COM
37
đường đối cực của điểm D’ đối với (I). Tương tự, ta có BE, CF cũng là đường đối
cực của các điểm E’, F’ đối với (I).
Hình 2.20
Ta biết AD, BE, CF đồng quy tại điểm Giéc-gôn, gọi là K. Khi đó, D’, E’, F’
phải nằm trên đường đối cực của điểm K đối với (I) (Hình 2.20). Từ đó suy ra D’,
E’, F’ thẳng hàng và đường thẳng D’E’F’ vuông góc với IK.
Ví dụ 2.21. [4] Cho tam giác ABC không cân. Các đường phân giác ngoài của
các góc A, B, C cắt các cạnh đối diện lần lượt tại A', B', C'. Gọi O, I lần lượt là tâm
đường tròn ngoại tiếp và nội tiếpABC. Chứng minh rằng các điểm A', B', C'
thẳng hàng và đường thẳng A'B'C' vuông góc với OI.
Giải. Bài toán không cho các đường vuông góc, nhưng với các giả thiết về
đường tròn nội tiếp, đường tròn ngoại tiếp tam giác gợi ý cho chúng ta có thể sử
dụng khái niệm cực và đường đối cực trong chứng minh bằng toán vuông góc. Gọi
tiếp điểm của đường tròn (I) nội tiếp tam giác trên BC, CA, AB lần lượt là D, E, F.
Gọi M, N, P lần lượt là trung điểm của các cạnh FE, FD, DE. Xét cực và đường đối
cực đối với đường tròn (I). Ta thấy AA' là đường đối cực của M nên A' thuộc đường
đối cực của M. Mà A' thuộc BC là đường đối cực của D nên ta có đường đối cực của
A' chính là đường thẳng DM (1)
Tương tự, đường đối cực của B', C' lần lượt là các đường thẳng EN, FP (2)
(Hình 2.21).
Tải tài liệu tại sividoc.com
Viết đề tài giá sinh viên – ZALO:0973.287.149-TEAMLUANVAN.COM
38
Chú ý rằng các đường thẳng DM, EN, FP đồng quy tại trọng tâm G của tam giác
DEF (3) Từ (1), (2), (3) ta có A', B', C' thẳng hàng và đường thẳng A'B'C' vuông góc
với IG (đường thẳng Ơle của tam giác DEF).
Hình 2.21
Ví dụ 2.22. [4] Cho tứ giác ABCD ngoại tiếp đường tròn (O) và M, N, P, Q
lần lượt là các tiếp điểm trên các cạnh AB, BC, CD, DA của tứ giác. Đặt K =
ADBC,L=ABDC,E=QMPN,F=QPMN.
Chứng minh bốn điểm K, L, E, F thẳng hàng.
Giải. Bài toán xuất hiện các cực và đường đối cực của điểm đối với đường
tròn. Do đó, gọi I là giao điểm của BD với AC, E’ là giao điểm của DB với KL, T là
giao điểm của CE’ với DK (Hình 2.22).
Dễ thấy (TAKD) = -1 suy ra (CT, CA, CK, CD) = -1. Do đó (E’IBD) = -1.
Mặt khác, (EIBD) = -1 nên suy ra E’ E. Từ đó suy ra E, K, L thẳng hàng (1)
Lập luận tương tự cũng có F, K, L thẳng hàng (2)
Từ (1) và (2) ta suy ra điều phải chứng minh.
Tải tài liệu tại sividoc.com
Viết đề tài giá sinh viên – ZALO:0973.287.149-TEAMLUANVAN.COM
39
F
T
K
A E
M
B
Q
D
O. I
P
N
L
C
Hình 2.22
Ví dụ 2.23.[2] Cho tứ giác ABCD nội tiếp đường tròn (O). Gọi

S

 ABCD,

F

 ADBC,

E ACBD
. Kẻ tiếp tuyến SM, SN với (O).

Chứng minh rằng bốn điểm E, F, M, N thẳng hàng.
Giải. Bài toán có các tiếp tuyến đối với đường tròn, do vậy nó có liên quan
đến bài toán dựng đường đối cực của một điểm đối với đường tròn và từ đó làm
xuất hiện các tứ giác toàn phần.
Giả sử
K

CDEF
,
K ' CDMN.
Khi đó, theo tính chất của tứ giác toàn

phần FEAB ta có (SKDC) = -1 (1)
Mặt khác theo tính chất “cát tuyến”, ta có (SK’DC) = -1 (2)
Tải tài liệu tại sividoc.com
Viết đề tài giá sinh viên – ZALO:0973.287.149-TEAMLUANVAN.COM
40
F
.
.
M B
A E
.
O
S D K
C
N
Hình 2.23
Từ (1) và (2) ta có K K’.
Tương tự, ta cũng có L L’ với
LEFAB
,
L'MNAB
.
 
Từ đó EF và MN có hai điểm chung nên hai đường thẳng này phải trùng
nhau. Vậy ta có M, N, E, F thẳng hàng.
2.5. Chứng minh đồng quy
Bài toán chứng minh đồng quy có thể coi là bài toán “đối ngẫu” của bài toán
chứng minh thẳng hàng. Phép đối cực chính là phương tiện để chuyển đổi hai dạng
bài toán. Do vậy, cực và đường đối cực sẽ được khai thác triệt để trong giải bài toán
dạng này.
Ví dụ 2.24. [4] Cho tam giác ABC. Đường tròn nội tiếp tam giác ABC tiếp
xúc với các cạnh BC, CA, AB lần lượt tại D, E, F. Đường tròn nội tiếp tam giác DEF
tiếp xúc với EF, FD, DE lần lượt tại M, P, N. Chứng minh rằng các đường thẳng
AM, BP, CN đồng quy.
Giải. Gọi I, O lần lượt là tâm của đường tròn nội tiếp tam giác DEF và tam
giác ABC. Gọi H, K, L lần lượt là giao điểm của các cặp đường thẳng (MP; EF),
(MN; FD), (MP; DE). Ta dễ thấy H, K, L thẳng hàng (1)
Tải tài liệu tại sividoc.com
Viết đề tài giá sinh viên – ZALO:0973.287.149-TEAMLUANVAN.COM
41
Hình 2.24
Chú ý rằng DM, FN, EP đồng quy nên (HMFE) = -1. Do đó, M thuộc đường đối cực
của điểm H đối với đường tròn (O). Mặt khác, điểm A thuộc đường đối cực của
điểm H đối với (O) nên ta có AM là đường đối cực của điểm H đối với (O) (2)
Tương tự, ta có BP là đường đối cực của điểm K đối với (O) và Cn là đường đối cực
của điểm L đối với (O) (3)
Từ (1), (2) và (3) suy ra điều phải chứng minh.
Ví dụ 2.25. [4] Cho tứ giác ABCD nội tiếp đường tròn (O). Gọi M, N lần lượt
là trung điểm của AB, CD. Đường tròn (ABN) cắt lại cạnh CD tại điểm P, đường
tròn (CDM) cắt lại cạnh AB tại điểm Q. Chứng minh rằng các đường thẳng AC, PQ,
BD đồng quy.
Giải. Trong bài toán này, chúng ta sẽ tìm các đường đối cực của các điểm đối
với đường tròn (O). Từ đó khai thác tính chất của các đường đối cực trong chứng
minh đồng quy. Thật vậy, khi AB // CD thì bài toán đơn giản. Ta đi xét trường hợp
còn lại.
Tải tài liệu tại sividoc.com
Viết đề tài giá sinh viên – ZALO:0973.287.149-TEAMLUANVAN.COM
42
Gọi S là giao điểm của đường thẳng AB và CD (Hình 2.25). Gọi d là đường đối cực
của điểm S đối với (O). Gọi I là giao điểm của AC và BD thì dễ thấy điểm I thuộc
đường thẳng d. Ta thấy SM .SQ SC.SD SA.SB . Chú ý rằng M là trung điểm của AB
nên ta có (SQAB) = -1. Do đó, điểm Q thuộc đường thẳng d. Tương tự ta có điểm P
cũng thuộc đường thẳng d. Từ đó suy ra điều phải chứng minh.
Hình 2.25
Ví dụ 2.26. [4] Trong tam giác ABC kẻ các đường cao AA’, BB’, CC’. Gọi H
là trực tâm của tam giác ABC. Gọi J là một giao điểm của AA’ với đường tròn (I)
đường kính BC. Chứng minh rằng BC, B’C’ và tiếp tuyến tại điểm J của đường tròn
(I) đồng quy.
Giải. Tương tự ví dụ 2.25, trong bài toán này, chúng ta cũng sẽ đi tìm các
đường đối cực của một điểm nào đó đối với đường tròn (I). Gọi giao điểm của AH
với đường tròn (I) là J1, J2. Vậy điểm J sẽ là J2 hoặc J1. Ta chứng minh BC, B’C’ và
tiếp tuyến tại J1 của đường tròn (I) đồng quy.
Xét cực và đường đối cực đối với đường tròn (I). Gọi giao điểm của BC và
B’C’ là điểm S (Hình 2.26). Ta thấy AH là đường đối cực của điểm S, AH đi qua
điểm J1 nên đường đối cực của J1 sẽ đi qua điểm S hay tiếp tuyến tại J1 đi qua điểm
S. Vậy ta có điều phải chứng minh.
Tải tài liệu tại sividoc.com
Viết đề tài giá sinh viên – ZALO:0973.287.149-TEAMLUANVAN.COM
43
Hình 2.26
Ví dụ 2.27. [4] Gọi O là tâm đường tròn nội tiếp tứ giác ABCD. Qua A, B, C,
D lần lượt vẽ các đường thẳng dA, dB, dC và dD tương ứng vuông góc với OA, OB,
OC và OD. Các cặp đường thẳng dA và dB, dB và dC, dC và dD, dD và dA tương ứng
cắt nhau tại K, L, M, N. Chứng minh rằng KM và LN cắt nhau tại O.
Giải. Gọi I, J, P, Q lần lượt là tiếp điểm của đường tròn (O) trên AB, BC, CD,
DA (Hình 2.27).
Hình 2.27
Tải tài liệu tại sividoc.com
Viết đề tài giá sinh viên – ZALO:0973.287.149-TEAMLUANVAN.COM
44
Gọi E, F, G, H lần lượt là giao điểm của các cặp đường thẳng (OA; IQ), (OB;
IJ), (OC; JP), (OD; PQ).
Ta sẽ chứng minh K, O, M thẳng hàng (Hình 2.27). Theo giả thiết ta sẽ có dA
là đường đối cực của điểm E đối với đường tròn (O). Tương tự, dB là đường đối cực
của điểm F. Từ đó suy ra EF là đường đối cực của điểm K, GH là đường đối cực
của điểm M đối với đường tròn (O). Mặt khác, dễ thấy EF // GH. Từ đó suy ra điều
phải chứng minh.
Ví dụ 2.28. [4] Cho tứ giác ABCD ngoại tiếp đường tròn (O). Tiếp điểm
thuộc các cạnh AB, BC, CD, DA lần lượt là M, N, P, Q. Các đường thẳng AN, AP cắt
đường tròn (O) tại E, F. Chứng minh rằng:
a) Các đường thẳng MP, NQ, AC, BD đồng quy.
b) Các đường thẳng ME, QF, AC đồng quy.
Giải.
.I
.
J
Hình 2.28
a) Hạ CJ MP. Ta có:OMPOPMBMPCPM CJ CP .
GọiI AC MP
IA

AM

AM
(1)
IC JC PC
Tương tự gọi
I' ACNQ I ' A AQ
(2)
 I ' C NC
Tải tài liệu tại sividoc.com
Viết đề tài giá sinh viên – ZALO:0973.287.149-TEAMLUANVAN.COM
45
Vì AM = AQ và PC = PN nên từ (1) và (2) suy ra I I ' . Ta suy ra các đường thẳng
MP, NQ, AC đồng quy tại I (3)
Tương tự, ta cũng có MP, NQ, BD đồng quy tại I (4)
Kết hợp (3) và (4) ta có điều phải chứng minh.
b) Gọi K là cực của đường thẳng AC đối với đường tròn (O). Xét tứ giác nội
tiếp MNPQ. Theo tính chất cực và đường đối cực của tứ giác nội tiếp ta có MQ và
NP cắt nhau tại điểm K. Tương tự, tứ giác EFPN nội tiếp cũng có EF và NP cắt
nhau tại điểm K suy ra MQ và EF cắt nhau tại điểm K . Vì ME và QF cắt nhau tại
một điểm thuộc đường đối cực của K đối với (O) tức thuộc đường thẳng AC. Do đó,
ba đường thẳng ME, QF, AC đồng quy.
Ví dụ 2.29.(Định lý Đờ-giác) [3] Cho ΔABC và ΔA’B’C’. Ta gọi giao điểm
của cạnh BC và B’C’, cạnh CA và C’A’, cạnh AB và A’B’ lần lượt là X, Y, Z. Chứng
minh rằng các điểm X, Y, Z thẳng hàng khi và chỉ khi các đường thẳng AA’, BB’,
CC’ đồng quy.
Giải. Định lý này có thể chứng minh dựa vào định lý Xêva kết hợp với định
lý Mênêlauýt. Tuy nhiên, các cách chứng minh đó không thể cho ta ngay chiều
ngược lại của bài toán. Phương pháp sử dụng tỉ số kép của chùm đường thẳng hay
phép chiếu xuyên tâm dưới đây sẽ khắc phục điều này.
B
D
C
A Y
Z
X A’
C’
B’
D’
Hình 2.29
Tải tài liệu tại sividoc.com
Viết đề tài giá sinh viên – ZALO:0973.287.149-TEAMLUANVAN.COM
46
Thật vậy, gọi AA’BY = D, AA’B’Y = D’. Ta có ba điểm X, Y, Z thẳng
hàng A(XYZA’) = A’(XYZA) (YCDB) = (C’YB’D’) (YCDB) = (C’YB’D’) =
(YC’D’B) AA’, BB’, CC’ đồng quy (Hình 2.29).
2.6. Chứng minh điểm cố định
Đường đối cực của một điểm đối với hai đường thẳng cắt nhau hoặc đối với
đường tròn cho trước là tập hợp các điểm liên hợp điều hòa với điểm đó. Do vậy,
chúng ta có thể khai thác tính chất của cực và đường đối cực để giải quyết các bài
toán tìm điểm cố định qua các ví dụ cụ thể dưới đây.
Ví dụ 2.30. [4] Cho tam giác ABC nội tiếp trong đường tròn (O). Gọi D và D’
là chân hai đường phân giác trong và ngoài của góc A. Gọi P là giao điểm của hai
tiếp tuyến của (O) tại B và C. Chứng minh rằng cực của đường thẳng AP đối với (O)
là trung điểm của DD’.
Giải. Gọi E là trung điểm của DD’. Ta có đường đối cực của điểm P là
đường thẳng BC đi qua E (Hình 2.30). Do vậy E và P là hai điểm liên hợp với nhau
đối với đường tròn (O) (1)
Mặt khác, do AD là đường phân giác của gócBAC và AD A D’ nên ta có
(D’DBC) = -1 (chùm tâm A). Vì E là trung điểm của DD’ nên theo hệ thức Niu-tơn
ta có: ED '2
 ED 2
EB .EC (2)
Hình 2.30
Tải tài liệu tại sividoc.com
Viết đề tài giá sinh viên – ZALO:0973.287.149-TEAMLUANVAN.COM
47
XétADD’ với AE là đường trung tuyến. Ta có AE = ED = ED’ (3)
Từ (2) và (3) ta suy ra EA2
 . . Ta suy ra AE là tiếp tuyến của (O) tại
EB EC
điểm A. Do đó, E và A liên hợp với nhau đối với đường tròn (O) (4)
Từ (1) và (4) suy ra đường đối cực của điểm E đối với đường tròn (O) chính
là đường thẳng AP.
Ví dụ 2.31. [4] Cho đường tròn (O) đường kính AB và đường thẳng d vuông
góc với AB tại điểm I ở ngoài đường tròn. Điểm M thay đổi trên (O), các đường
thẳng MA, MB cắt d lần lượt tại P và Q. Đường thẳng QA cắt đường tròn (O) tại
điểm N. Chứng minh rằng đường thẳng MN đi qua một điểm cố định.
Giải. Giả thiết bài toán có đường tròn và đường thẳng d cố định, điều đó làm
xuất hiện suy nghĩ rằng điểm cố định cần tìm có thể là cực của một đường thẳng cố
định đối với một đường tròn cho trước. Do đó, khai thác tính chất của cực và đường
đối cực đối với đường tròn cũng là một hướng tiếp cận trong bài toán này.
Gọi E = AO MN. XétBQP ta có AM QB, AI PQ nên suy ra A là trực
tâm của tam giác BQP. Từ đó ta có QA BP hay QN BP (Hình 2.31).
Mà BN QN nên suy ra P, N, B thẳng hàng. Mặt khác, đường thẳng QE là
đường đối cực của điểm P đối với đường tròn (O) nên hai điểm E và P liên hợp với
nhau đối với (O). Mà PQ OE nên ta suy ra E là cực của đường thẳng PQ đối với
đường tròn (O). Vậy E là điểm cố định (vì PQ cố định).
Hình 2.31
Tải tài liệu tại sividoc.com
Viết đề tài giá sinh viên – ZALO:0973.287.149-TEAMLUANVAN.COM
48
Ví dụ 2.32. [4] Từ điểm P nằm ngoài đường tròn (O) ta vẽ các tiếp tuyến PA
và PB tới đường tròn (O). Từ điểm B hạ đường vuông góc BD với đường kính AC.
Chứng minh rằng PC đi qua trung điểm BD.
Giải. Bài toán xuất hiện các đường tiếp tuyến đối với đường tròn, do đó
chúng ta có thể xác định các cặp điểm liên hợp điều hòa trong bài toán này.
Hình 2.32
Gọi I là giao điểm của PC và BD (Hình 2.32). Kéo dài PB cắt AC tại điểm E.
Ta có hai điểm B và E liên hợp với nhau đối với đường tròn (O). Mà BD CE nên
đường đối cực của điểm E đối với đường tròn (O) là đường thẳng BD. Vậy hai điểm
E và D liên hợp với nhau đối với (O) nên suy ra (EDCA) = -1 hay P(EDCA) = -1.
Mà ta có PA // BD suy ra IB = ID. Vậy PC đi qua trung điểm của BD.
Ví dụ 2.33. [4] Cho đường tròn (O) và dây cung AB. Từ trung điểm I của dây
cung AB kẻ hai dây cung MN và PQ. Các đường thẳng MP và NQ cắt dây cung AB
lần lượt tại J và K. Chứng minh rằng I cũng là trung điểm của JK.
Giải. Đây là bài toán con bướm quen thuộc, chúng ta có thể chứng minh
thông qua các tam giác đồng dạng và các góc bằng nhau. Tuy nhiên, sử dụng khái
niệm đường đối cực giúp lời giải trở nên thú vị hơn.
Gọi D là giao điểm của hai tiếp tuyến tại A và B của đường tròn (O). Khi đó,
ta có AB chính là đường đối cực của điểm D đối với đường tròn (O) suy ra I và D là
hai điểm liên hợp với nhau đối với đường tròn (O) (Hình 2.33). Kẻ Dx OI suy ra
Dx chính là đường đối cực của điểm I đối với (O). Mặt khác, I và C là hai điểm liên
hợp với nhau đối với (O) nên C Dx. Gọi E là giao điểm của PQ và Dx. Khi đó hai
Tải tài liệu tại sividoc.com
Viết đề tài giá sinh viên – ZALO:0973.287.149-TEAMLUANVAN.COM
49
điểm E và I liên hợp với nhau đối với (O), ta suy ra (PQIE) = -1 hay C(PQIE) = -1.
Mà ta có JK // Cx suy ra IJ = JK. Vậy I là trung điểm của đoạn thẳng JK.
Hình 2.33
Ví dụ 2.34.[4] Cho đường tròn (O), điểm M nằm ngoài (O) và điểm I nằm
trong (O). Một đường thẳng thay đổi qua I cắt (O) tại A, A’. Các đường thẳng MA,
MA’ lần lượt cắt (O) tại các điểm thứ hai B, B’. Chứng minh rằng đường thẳng BB’
đi qua một điểm cố định.
Giải.
* Trường hợp 1: Gọi N = AB’ A’B và P = AA’ BB’. Ta có NP chính là
đường đối cực của điểm M đối với (O).
Hình 2.34
Tải tài liệu tại sividoc.com
Viết đề tài giá sinh viên – ZALO:0973.287.149-TEAMLUANVAN.COM
50
Gọi Q = NP MI, R = BB’ MI, S = NP MA’ suy ra P(MSB’A’) = -1. Từ
đó ta có P(MQRI) = -1 suy ra (MQRI) = -1. Do M, Q, I cố định nên R cũng cố định.
Vậy BB’ đi qua điểm cố định R MI thỏa mãn (MORI) = -1 với Q là giao điểm của
đường đối cực của điểm M đối với (O) với đường thẳng MI (Hình 2.34).
* Trường hợp 2: AB’ // A’B.
Gọi J = AA’ BB’. Ta có MO chính là đường trung trực của A’B nên cũng đi
qua J. Gọi C, D lần lượt là giao điểm
của MO với (O). Khi đó, do CD là
đường kính nên gócDAC = 900
. Mặt
khácA’AC =CAB nên suy ra chùm
A(DCJM) là chùm phân giác suy ra
(DCJM) = -1. Vậy điểm J thuộc đường
đối cực của điểm M đối với (O) suy ra
J(B’A’SM) = -1 hay (MQRI) = -1.
Vậy điểm R cố định (Hình 2.35).
Hình 2.35
* Trường hợp 3: AA’ // BB’.
Ta chứng minh giao của DR và CI nằm trên đường đối cực của điểm M đối với
đường tròn (O). Khi đó (MQRI) = (MJCD) = -1 (Hình 2.36). Vậy R là điểm cố định.
Hình 2.36
Tải tài liệu tại sividoc.com
Viết đề tài giá sinh viên – ZALO:0973.287.149-TEAMLUANVAN.COM
51
Ví dụ 2.35. [4] Cho đường tròn (O)
và một đường thẳng d nằm ngoài (O). Một
điểm S di chuyển trên đường thẳng d. Từ S
kẻ hai tiếp tuyến SA, SB tới (O). Chứng
minh rằng khi S di chuyển trên d thì AB
luôn đi qua một điểm cố định.
Giải. Gọi I là cực của đường
thẳng d đối với đường tròn (O). Vì d cố
định nên điểm I cố định (Hình 2.37). Vì
điểm S thuộc đường thẳng d nên suy ra
I
Hình 2.37
đường đối cực của điểm S sẽ đi qua cực của đường thẳng d hay đường thẳng AB
luôn đi qua điểm I cố định.
Ví dụ 2.36. [2] Cho đường tròn (O), trên (O) có hai điểm B, C cố định và
điểm A thay đổi. Đường kính DE vuông góc với BC và cắt AB, AC lần lượt tại M, N.
Lấy P, Q sao cho (ABMP) = (ACNQ) = -1. Chứng minh rằng PQ đi qua một điểm
cố định.
Giải.
* Cách 1: Gọi I là trung điểm của đoạn thẳng BC, ta chứng minh PQ đi qua I
GọiF BQ DE. Vì (ACNQ) = -1 (QM, BI, BN, BF) = -1 (MINF) = -1 (1)
Gọi
 
(Hình 2.38).
I ' PQ BC . Ta chứng minh I ' I
Do (ABMP) = -1 (QN, QF, QM, QI’) = -1  (NFMI’) = -1 (2)
Từ (1) và (2) suy ra I ' I . Vậy đường thẳng PQ đi qua điểm I cố định.
* Nhận xét: Nếu PQ đi qua điểm I thì từ (ABMP) = -1 Q(ABMP) = -1. Ta
có BC cắt QA, QB, QP lần lượt tại B, I, C mà IB = IC nên ta nghĩ đến MQ // BC. Từ
đó ta có cách giải khác như sau:
* Cách 2: Giả sử IQ ABP ', ta sẽ chứng minh P ' P . Qua điểm A kẻ
đường thẳng vuông góc với DE và cắt DE, IQ lần lượt tại H, K.
Tải tài liệu tại sividoc.com
Viết đề tài giá sinh viên – ZALO:0973.287.149-TEAMLUANVAN.COM
52
MA

HA

HA

NA

QA
(do AH//BC,  và tỉ số kép
Ta có IB IC
MB IB IC NC QC
(ACNQ) = -1)) MQ //BC.
N
P
D
A
M Q
F
B I C
E
Hình 2.38
P ' A KA
Khi đó, ta có  (do AK//IB) P 'A KA QA MA .
P ' B IB P ' B IC QC MB
Hay MA :P ' A 1, từ đó suy ra (ABMP’) = -1.
MB P ' B
Mặt khác, theo giả thiết (ABMP) = -1. Do đó suy ra P ' P .
* Nhận xét: Nếu sử dụng định lý Mênêlauýt, ta có thể suy ra cách giải thứ ba
dưới đây:
* Cách 3: Gọi I là trung điểm đoạn thẳng BC.
Áp dụng định lý Mênêlauýt choABC , ta cần chứng minh IB
.
QC .PA 1.
IC PB
QA
MA
:
PA
1
PA

MA
Vì (ABMP) = -1 nên hay .
MB PB PB MB
Tải tài liệu tại sividoc.com
Viết đề tài giá sinh viên – ZALO:0973.287.149-TEAMLUANVAN.COM
53
Tương tự, ta có (ACNQ) = -1 nên NA : QA 1 hay QC NC .
NC QC QA NA
  
IB QC PA

IB

NC

MA
1 (áp dụng định lý Mênêlauýt
Từ đó . . .
  .
 
IC QA PB IC NA  MB
với MNI là cát tuyến trongABC).
Ví dụ 2.37. [2] Cho hai đường thẳng cố định Ox, Oy và điểm A không nằm
trên Ox, Oy và phân giácxOy. Hai đường thẳng di động qua A, đối xứng qua OA,
một đường cắt Ox tại M, đường kia cắt Oy tại N. Chứng minh rằng đường thẳng MN
đi qua một điểm cố định.
Giải. Theo đề bài A và O là hai điểm cố định. Các đường thẳng AM, AN lại
đối xứng qua OA, nghĩa là phân giác củaMAN, từ đó ta nghĩ đến chùm phân giác.
Do đó, ta kẻ đường thẳng d qua điểm A và vuông góc OA để tạo ra chùm phân giác.
d
K
M
O I
A
N
Hình 2.39
Gọi d là đường thẳng qua A và vuông góc với OA, K là giao điểm của MN và
d, I là giao điểm của OA và MK.
Do AM và AN đối xứng với nhau qua OA nên AO là phân giác củaMAN.
Mà AI AK, do đó AK, AI, AM, AN là chùm phân giác nên là chùm đường
thẳng điều hòa A(KIMN) = -1 (KIMN) = -1 O(KIMN) = -1.
Tải tài liệu tại sividoc.com
Viết đề tài giá sinh viên – ZALO:0973.287.149-TEAMLUANVAN.COM
54
Mà các đường thẳng OM, OI, ON cố định nên đường thẳng OK cố định.
Mặt khác,K OK d điểm K cố định. Vậy đường thẳng MN luôn đi
qua điểm K cố định (Hình 2.39).
Ví dụ 2.38. [2] Cho tam giác ABC không cân (B, C cố định, A thay đổi), với
ba đường cao AD, BE, CF. Đường thẳng d đi qua D, song song với EF, cắt các
đường thẳng AB, AC lần lượt tại M, N, các đường thẳng EF, BC cắt nhau tại P.
Chứng minh rằng đường tròn ngoại tiếp tam giác MNP đi qua một điểm cố định.
Giải. Giả thiết của bài toán có các đường phân giác nên ta nghĩ đến chùm
phân giác và I là trung điểm của DE nên nghĩ đến hệ thức Mácloranh trong bài toán.
A
d
E
F
N
P B
. D Q
C
M
Hình 2.40
Theo tính chất của tứ giác toàn phần, ta có (BCDP) = -1. Khi đó, nếu gọi Q là trung
điểm của cạnh BC, theo hệ thức Mácloranh ta có: DQ .DP DB . DC (1)
Mặt khác, do tứ giác BCEF nội tiếp (đường tròn đường kính BC) và MN // EF nên
(NC, NM) = (EC, EF) = (BC, BF) = (BC, BM). Từ đó, suy ra bốn điểm B, C, M, N
cùng nằm trên một đường tròn. Do đó: DB .DC DM .DN (2)
Từ (1) và (2) suy ra DQ.DP DM .DN , tức là bốn điểm P, Q, M, N cùng nằm
trên đường tròn. Vậy đường tròn ngoại tiếp tam giác MNP đi qua điểm Q cố định.
Ví dụ 2.39. [2] Cho đường tròn (O). Hai điểm B, C cố định trên đường tròn,
BC không phải đường kính. Lấy A là điểm trên đường tròn không trùng với B,
C. Các đường thẳng AD, AE lần lượt là đường phân giác trong và ngoài của
Tải tài liệu tại sividoc.com
Viết đề tài giá sinh viên – ZALO:0973.287.149-TEAMLUANVAN.COM
55
BAC. Điểm I là trung điểm của DE. Qua trực tâm của tam giác ABC kẻ đường
thẳng vuông góc với AI cắt AD, AE lần lượt tại M, N. Chứng minh rằng MN luôn đi
qua một điểm cố định.
Giải.
Gọi 2 là độ lớn cung nhỏ BC. Khi đóBAC bằng hoặc 180o
- .
Gọi J là điểm đối xứng của O qua BC, suy ra J cố định.
Ta có OJ = 2d(O,BC) = 2Rcos = AH, OJ // AH (vì cùng vuông góc với BC)
nên AOJH là hình bình hành. Ta suy ra AO // HJ (1)
A
N
O.
H
B
D
I
E
C
M
J
Hình 2.41
Lại có (CBDE) = -1 nên theo hệ thức Niutơn ta có ID 2
 . , mà IA = ID
IB IC
(tam giác ADE vuông tại A), suy ra IA2
 . .
IB IC
Do đó, IA tiếp xúc (O) hay IA OA (2)
Từ (1) và (2) suy ra JH AI, mà MN đi qua H và vuông góc với AI nên M, N,
J thẳng hàng. Vậy MN đi qua điểm J cố định.
2.7. Chứng minh đẳng thức
Hệ thức Đề-các, hệ thức Niutơn và hệ thức Mácloranh đối với hàng điểm
điều hòa là những công cụ hiệu quả khi giải bài toán chứng minh đẳng thức, đặc biệt
là các bài toán đẳng thức chứa các tỷ lệ thức. Chúng ta cần nhận dạng những đẳng
thức có liên quan đến hàng điểm điều hòa như các ví dụ dưới đây.
Tải tài liệu tại sividoc.com
Viết đề tài giá sinh viên – ZALO:0973.287.149-TEAMLUANVAN.COM
56
Ví dụ 2.40. [4] ChoABC và AH = h là chiều cao xuất phát từ A. Gọi r và r’
lần lượt là bán kính của đường tròn nội tiếp và bàng tiếp đối với góc A của tam giác
ABC. Chứng minh rằng: h
2

1
r r
1
' .
Giải. Đẳng thức cần chứng minh chính là hệ thức Đề-các đối với hàng điểm
điều hòa, vì vậy ta phải tìm cách xác định được hàng điểm điều hòa đó. Bài toán cho
h là chiều cao xuất phát từ A, còn r, r’ là bán kính của đường tròn nội tiếp và bàng
tiếp đối với góc A của tam giác ABC, điều này làm ta nghĩ đến sử dụng chùm phân
giác để suy ra hàng điểm điều hòa trong bài toán này.
Gọi I là tâm đường tròn nội tiếp tam giác ABC và K là tâm đường tròn bàng
tiếp góc A của tam giác đó. Các đường thẳng BI, BK là phân giác của góc B. Ta suy
ra các tia BA, BC, BI, BK tạo nên một chùm điều hoà. Gọi D là giao điểm của AI và
BC ta có (ADIK) = -1.
Chiếu vuông góc các điểm A,
I, D, K xuống đường cao AH ta
được lần lượt các điểm A, I', H, K'.
Trên tia HA chọn H làm gốc
toạ độ, ta có:
HA h, HI ' r , HK 'r '
Ta có (AHI'K') = -1
 HA
2
 HI
1
'HK
1
'
(theo hệ thức Đề-các đối với hàng
điểm điều hòa).
2 1 1
Hình 2.42
hay h r r ' .
Ví dụ 2.41. [4] Cho tam giác ABC cóBCA = 900
. Gọi D là chân đường cao
hạ từ C, X là điểm nằm trên đoạn thẳng CD. Gọi K là điểm thuộc đoạn thẳng AX sao
cho BK = BC. Tương tự L là điểm trên đoạn thẳng BX sao cho AL = AC. Gọi M là
giao điểm của AL và BK. Chứng minh rằng MK = ML.
Tải tài liệu tại sividoc.com
Viết đề tài giá sinh viên – ZALO:0973.287.149-TEAMLUANVAN.COM
57
Giải.
Dễ thấy các đường tròn (A, AC) và (B, BC) cắt nhau tại điểm E khác C thì E
đối xứng với C qua đường thẳng AB. Khi đó, dễ thấy AC, AE cùng tiếp xúc với
đường tròn (B, BC). Gọi Q = AK (B, BC) (Q khác K). Do AC, AE cùng tiếp xúc
với đường tròn (B, BC) nên tứ giác CQEK là tứ giác điều hòa. Do đó, tiếp tuyến tại
K và Q của (B, BC) cắt nhau tại điểm P thuộc CE. Hơn nữa ta có (PXCE) = -1.
Vậy tương tự, nếu gọi N = BL (A, AC) thì tiếp tuyến tại L và N cắt nhau tại
P’ thuộc CE và (P’XCE) = -1. Do đó P P’. Từ đó, chú ý CE là trục đẳng phương
của (A, AC) và (B, BC) nên PL = PK. Từ đó ta dễ thấy hai tam giác vuôngPML
=PMK suy ra MK = ML (Hình 2.43).
Hình 2.43
Ví dụ 2.42. [4] Cho hai đường tròn (O1) và (O2) cắt nhau tại A, B. Gọi C, D
thuộc đường thẳng O1O2 sao cho AC vuông góc với O1A và AD vuông góc với O2A.
Gọi P là điểm thuộc đoạn thẳng AB. Đường thẳng CP giao với (O1) tại L sao cho C,
L khác phía với AB. Đường thẳng DP giao với (O2) tại K sao cho D, K khác phía với
AB, LO1 cắt KO2 tại M. Chứng minh rằng MK = ML.
Tải tài liệu tại sividoc.com
Viết đề tài giá sinh viên – ZALO:0973.287.149-TEAMLUANVAN.COM
58
Giải.
Gọi DK giao (O2) tại R khác K. Ta dễ thấy DA, DB tiếp xúc với (O2) do đó tứ
giác ARBK điều hòa. Vậy tiếp tuyến tại K và R của (O2) cắt nhau tại Q thuộc AB và
(ABPQ) = -1 (Hình 2.44).
Hình 2.44
Tương tự, gọi CL giao với (O1) tại S khác L thì tiếp tuyến tại S và L của (O1)
cắt nhau tại Q’ thuộc AB và (ABPQ’) = -1 do đó Q Q’. Từ đó QL, QK lần lượt tiếp
xúc (O1), (O2) mà AB là trục đẳng phương của (O1), (O2) do đó QL = QK. Từ đó, ta
dễ thấy hai tam giác vuôngQML =QMK suy ra MK = ML.
Ví dụ 2.43. [2] Cho hình bình hành ABCD và d là đường thẳng thay đổi qua
A và cắt BD, BC, CD lần lượt tại E, F, G. Chứng minh rằng AE
1
 AF
1
 AG
1
.
Giải. Đẳng thức cần chứng minh có dạng giống như hệ thức Đề-các, do đó ta
cần dựng một điểm K sao cho AK
2
 AF
1
 AG
1
, nghĩa là AK 2AE , cần chứng
minh (AKFG) = -1.
Tải tài liệu tại sividoc.com
Viết đề tài giá sinh viên – ZALO:0973.287.149-TEAMLUANVAN.COM
59
A D
O
E
B
F
C
K
L
G
Hình 2.45
* Cách 1: Gọi K là điểm thuộc d sao cho AK 2AE hay E là trung điểm của
AK. Khi đó EO // KC (tính chất đường trung bình tam giác), suy ra BD//KC. Xét
chùm CA, CK,CF, CG. Ta có BD // CK chắn CA, CF, CG lần lượt tại O, B, D; mà
OB = OD (do ABCD là hình bình hành) nên theo định lý cát tuyến song song thì
C(AKFG) = -1 suy ra (AKFG) = -1.
Theo hệ thức Đề-các: AK
2
 AF
1
 AG
1
 AE
1
 AF
1
 AG
1
(do AK 2AE ).
* Cách 2: Gọi L là trung điểm của CG và K là giao điểm của BL và AG.
Do AB // LG nên KB
KL

LG
BA (định lý Ta-lét) mà
LG
BA CD
CL
,
suy ra KB
KL
 CD
CL
Theo định lý Ta-lét đảo, ta có CK // BD.
Xét tam giác ACK, ta có O là trung điểm AC và OE // CK, suy ra E là trung
điểm của AK. Xét chùm BA, BK, BF, BG, ta có GD là cát tuyến song song với BA,
chắn trên ba tia còn lại hai đoạn bằng nhau CL = LG.
Theo định lý cát tuyến song song, BA, BK, BF, BG là chùm điều hòa nên suy ra
(AKFG) = -1. Theo hệ thức Đề-các: AK
2
 AF
1
 AG
1
 AE
1
 AF
1
 AG
1
.
Ví dụ 2.44. [2] Cho tam giác ABC có trọng tâm G. Một đường thẳng d thay
đổi đi qua G cắt BC, CA, AB lần lượt tại M, N, P. Chứng minh rằng:
Tải tài liệu tại sividoc.com
Viết đề tài giá sinh viên – ZALO:0973.287.149-TEAMLUANVAN.COM
60
GM
1
GN
1
GP
1
0
Giải.
A Q x
N
G
B
I
C
M
d
P
Hình 2.46
Dựng Ax // BC. Gọi I là là trung điểm của BC.
Xét chùm Ax, AG, AB, AC có BC là cát tuyến song song với Ax, chắn trên ba
tia còn lại hai đoạn bằng nhau IB = IC. Theo định lý cát tuyến song song, Ax, AG,
AB, AC là chùm điều hòa suy ra (Ax, AG, AB,AC) = -1. Gọi Q là giao điểm của Ax
và d thì (QGPN) = -1 (Hình 2.46).
Theo hệ thức Đề-các: 2  1  1 2GM  GM  GM (1)
GQ GQ
GP GN GP GN
MàGIMGAQ ( g g) GI  GM 1
GA GQ 2
Vậy từ (1) ta suy ra1
GM
GP
GM
GN GM
1
 GP
1
 GN
1
 0 .
* Nhận xét: Từ bài toán này ta có thể mở rộng thêm: Qua A kẻ các đường
thẳng song song với GB, GC và cắt d tại X, Y. Chứng minh
1

1

1
 0 .
GM GX GY
Giải.
Tải tài liệu tại sividoc.com
Viết đề tài giá sinh viên – ZALO:0973.287.149-TEAMLUANVAN.COM
61
X
A
Y
x G
R B M I C S
d
Hình 2.47
Giả sử các đường thẳng đi qua A, song song GB, GC và cắt BC lần lượt tại R, S.
Ta chứng minh được G là trọng tâmARS (vì IB = IC và IR
IB

IC
IS
IG
IA
1
3 nên
IR = IS, mặt khác lại có
IG
IA
1
3 ) rồi áp dụng bài toán trên cho ba điểm M, X, Y
lần lượt nằm trên RS, AR, SA ta có được điều phải chứng minh.
Ví dụ 2.45. [2] Cho đường tròn đường kính CD tâm O. Trên CD lấy điểm
A1, A2 sao cho ( A1A2CD ) = -1. Qua lần lượt kẻ các đường t hẳng d1, d2 vuông
góc với CD. Một tiếp tuyến thay đổi của (O) cắt d1, d2 lần lượt tại M1, M2. Chứng
minh rằng
OM
1 const .
OM 2
Giải. Việc chứng minh tỉ số
OM1
 const làm ta nghĩ đến tính chất về tỷ lệ
OM 2
đường phân giác trong tam giác. Vậy ta cần tạo ra chùm phân giác có hai tia OM1,
OM2. Theo gải thiết, lại xuất hiện tiếp tuyến làm ta liên tưởng đến việc tạo ra sự
vuông góc (chùm phân giác). Như vậy ta cần tạo ra các tiếp tuyến nữa để sử dụng
tính chất hai tiếp tuyến cắt nhau. Ở đây, tiếp tuyến dựng từ C và D là điều tối ưu
nhất ta có thể dùng. Bằng cách cộng các góc, ta có đượcC ' OD ' 90o
từ đó chùm
tia cần tìm là O(M1M2C’D’).
A1, A2
Sử Dụng Hàng Điểm Điều Hòa Trong Giải Toán Hình Học Phẳng.doc
Sử Dụng Hàng Điểm Điều Hòa Trong Giải Toán Hình Học Phẳng.doc
Sử Dụng Hàng Điểm Điều Hòa Trong Giải Toán Hình Học Phẳng.doc
Sử Dụng Hàng Điểm Điều Hòa Trong Giải Toán Hình Học Phẳng.doc
Sử Dụng Hàng Điểm Điều Hòa Trong Giải Toán Hình Học Phẳng.doc
Sử Dụng Hàng Điểm Điều Hòa Trong Giải Toán Hình Học Phẳng.doc
Sử Dụng Hàng Điểm Điều Hòa Trong Giải Toán Hình Học Phẳng.doc
Sử Dụng Hàng Điểm Điều Hòa Trong Giải Toán Hình Học Phẳng.doc
Sử Dụng Hàng Điểm Điều Hòa Trong Giải Toán Hình Học Phẳng.doc
Sử Dụng Hàng Điểm Điều Hòa Trong Giải Toán Hình Học Phẳng.doc
Sử Dụng Hàng Điểm Điều Hòa Trong Giải Toán Hình Học Phẳng.doc

More Related Content

What's hot

BĐT Côsi ngược dấu
BĐT Côsi ngược dấuBĐT Côsi ngược dấu
BĐT Côsi ngược dấunhankhangvt
 
Một số ứng dụng của định lí pascal và định lí brianchon trong hình học sơ cấp
Một số ứng dụng của định lí pascal và định lí brianchon trong hình học sơ cấpMột số ứng dụng của định lí pascal và định lí brianchon trong hình học sơ cấp
Một số ứng dụng của định lí pascal và định lí brianchon trong hình học sơ cấpnataliej4
 
Hệ Hoán Vị Vòng Quanh
Hệ Hoán Vị Vòng QuanhHệ Hoán Vị Vòng Quanh
Hệ Hoán Vị Vòng QuanhNhập Vân Long
 
Ứng dụng phương tích và trục đẳng phương vào bài toán hình học phẳng
Ứng dụng phương tích và trục đẳng phương vào bài toán hình học phẳng Ứng dụng phương tích và trục đẳng phương vào bài toán hình học phẳng
Ứng dụng phương tích và trục đẳng phương vào bài toán hình học phẳng Bui Loi
 
Một số dạng toán về đa thức qua các kỳ thi Olympic
Một số dạng toán về đa thức qua các kỳ thi OlympicMột số dạng toán về đa thức qua các kỳ thi Olympic
Một số dạng toán về đa thức qua các kỳ thi OlympicMan_Ebook
 
Cđ một số ứng dụng định lí mê nê la uýt và xê va
Cđ một số ứng dụng định lí mê nê la uýt và xê vaCđ một số ứng dụng định lí mê nê la uýt và xê va
Cđ một số ứng dụng định lí mê nê la uýt và xê vaCảnh
 
Bài toán số học liên quan tới lũy thữa
Bài toán số học liên quan tới lũy thữaBài toán số học liên quan tới lũy thữa
Bài toán số học liên quan tới lũy thữaThế Giới Tinh Hoa
 
Bất đẳng thức
Bất đẳng thứcBất đẳng thức
Bất đẳng thứca123b234
 
Phương Tích - Trục Đẳng Phương
Phương Tích - Trục Đẳng PhươngPhương Tích - Trục Đẳng Phương
Phương Tích - Trục Đẳng PhươngNhập Vân Long
 
Bất đẳng thức suy luận và khám phá phạm văn thuận lê vĩ
Bất đẳng thức suy luận và khám phá   phạm văn thuận lê vĩBất đẳng thức suy luận và khám phá   phạm văn thuận lê vĩ
Bất đẳng thức suy luận và khám phá phạm văn thuận lê vĩThế Giới Tinh Hoa
 
Cđ một số kỹ thuật sd bđt cauchy
Cđ một số kỹ thuật sd bđt cauchyCđ một số kỹ thuật sd bđt cauchy
Cđ một số kỹ thuật sd bđt cauchyCảnh
 
Cđ đồng dư thức trong toán 7
Cđ đồng dư thức trong toán 7Cđ đồng dư thức trong toán 7
Cđ đồng dư thức trong toán 7Cảnh
 
Phep nghich dao __
Phep nghich dao  __Phep nghich dao  __
Phep nghich dao __Duc Tung
 
Giải một số phương trình nghiệm nguyên trong đề thi toán 9
Giải một số phương trình nghiệm nguyên trong đề thi toán 9Giải một số phương trình nghiệm nguyên trong đề thi toán 9
Giải một số phương trình nghiệm nguyên trong đề thi toán 9Nhập Vân Long
 
Một số bất đẳng thức hình học luận văn của thầy hoàng ngọc quang
Một số bất đẳng thức hình học   luận văn của thầy hoàng ngọc quangMột số bất đẳng thức hình học   luận văn của thầy hoàng ngọc quang
Một số bất đẳng thức hình học luận văn của thầy hoàng ngọc quangThế Giới Tinh Hoa
 

What's hot (20)

BĐT Côsi ngược dấu
BĐT Côsi ngược dấuBĐT Côsi ngược dấu
BĐT Côsi ngược dấu
 
Một số ứng dụng của định lí pascal và định lí brianchon trong hình học sơ cấp
Một số ứng dụng của định lí pascal và định lí brianchon trong hình học sơ cấpMột số ứng dụng của định lí pascal và định lí brianchon trong hình học sơ cấp
Một số ứng dụng của định lí pascal và định lí brianchon trong hình học sơ cấp
 
Scp mod p
Scp mod pScp mod p
Scp mod p
 
Hệ Hoán Vị Vòng Quanh
Hệ Hoán Vị Vòng QuanhHệ Hoán Vị Vòng Quanh
Hệ Hoán Vị Vòng Quanh
 
Ứng dụng phương tích và trục đẳng phương vào bài toán hình học phẳng
Ứng dụng phương tích và trục đẳng phương vào bài toán hình học phẳng Ứng dụng phương tích và trục đẳng phương vào bài toán hình học phẳng
Ứng dụng phương tích và trục đẳng phương vào bài toán hình học phẳng
 
Một số dạng toán về đa thức qua các kỳ thi Olympic
Một số dạng toán về đa thức qua các kỳ thi OlympicMột số dạng toán về đa thức qua các kỳ thi Olympic
Một số dạng toán về đa thức qua các kỳ thi Olympic
 
Cđ một số ứng dụng định lí mê nê la uýt và xê va
Cđ một số ứng dụng định lí mê nê la uýt và xê vaCđ một số ứng dụng định lí mê nê la uýt và xê va
Cđ một số ứng dụng định lí mê nê la uýt và xê va
 
Bài toán số học liên quan tới lũy thữa
Bài toán số học liên quan tới lũy thữaBài toán số học liên quan tới lũy thữa
Bài toán số học liên quan tới lũy thữa
 
Bất đẳng thức
Bất đẳng thứcBất đẳng thức
Bất đẳng thức
 
Bdt schur
Bdt schurBdt schur
Bdt schur
 
Phương Tích - Trục Đẳng Phương
Phương Tích - Trục Đẳng PhươngPhương Tích - Trục Đẳng Phương
Phương Tích - Trục Đẳng Phương
 
Tính bất khả quy Của đa thức với hệ số nguyên.docx
Tính bất khả quy Của đa thức với hệ số nguyên.docxTính bất khả quy Của đa thức với hệ số nguyên.docx
Tính bất khả quy Của đa thức với hệ số nguyên.docx
 
Bất đẳng thức suy luận và khám phá phạm văn thuận lê vĩ
Bất đẳng thức suy luận và khám phá   phạm văn thuận lê vĩBất đẳng thức suy luận và khám phá   phạm văn thuận lê vĩ
Bất đẳng thức suy luận và khám phá phạm văn thuận lê vĩ
 
Cđ một số kỹ thuật sd bđt cauchy
Cđ một số kỹ thuật sd bđt cauchyCđ một số kỹ thuật sd bđt cauchy
Cđ một số kỹ thuật sd bđt cauchy
 
Bất đẳng thức hình học
Bất đẳng thức hình họcBất đẳng thức hình học
Bất đẳng thức hình học
 
Luận văn: Bất đẳng thức trong lớp hàm siêu việt, HAY, 9đ
Luận văn: Bất đẳng thức trong lớp hàm siêu việt, HAY, 9đLuận văn: Bất đẳng thức trong lớp hàm siêu việt, HAY, 9đ
Luận văn: Bất đẳng thức trong lớp hàm siêu việt, HAY, 9đ
 
Cđ đồng dư thức trong toán 7
Cđ đồng dư thức trong toán 7Cđ đồng dư thức trong toán 7
Cđ đồng dư thức trong toán 7
 
Phep nghich dao __
Phep nghich dao  __Phep nghich dao  __
Phep nghich dao __
 
Giải một số phương trình nghiệm nguyên trong đề thi toán 9
Giải một số phương trình nghiệm nguyên trong đề thi toán 9Giải một số phương trình nghiệm nguyên trong đề thi toán 9
Giải một số phương trình nghiệm nguyên trong đề thi toán 9
 
Một số bất đẳng thức hình học luận văn của thầy hoàng ngọc quang
Một số bất đẳng thức hình học   luận văn của thầy hoàng ngọc quangMột số bất đẳng thức hình học   luận văn của thầy hoàng ngọc quang
Một số bất đẳng thức hình học luận văn của thầy hoàng ngọc quang
 

Similar to Sử Dụng Hàng Điểm Điều Hòa Trong Giải Toán Hình Học Phẳng.doc

VECTƠ VÀ CÁC ỨNG DỤNG
VECTƠ VÀ CÁC ỨNG DỤNGVECTƠ VÀ CÁC ỨNG DỤNG
VECTƠ VÀ CÁC ỨNG DỤNGDANAMATH
 
8 chuyen de hinh hoc phang
8 chuyen de hinh hoc phang8 chuyen de hinh hoc phang
8 chuyen de hinh hoc phangonthi360
 
Chuyendequytichbacthcs6155 141225015434-conversion-gate02
Chuyendequytichbacthcs6155 141225015434-conversion-gate02Chuyendequytichbacthcs6155 141225015434-conversion-gate02
Chuyendequytichbacthcs6155 141225015434-conversion-gate02Đông Nam Nguyễn
 
Sáng kiến kinh nghiệm: Giải một bài toán quỹ tích như thế nào - Trường THCS V...
Sáng kiến kinh nghiệm: Giải một bài toán quỹ tích như thế nào - Trường THCS V...Sáng kiến kinh nghiệm: Giải một bài toán quỹ tích như thế nào - Trường THCS V...
Sáng kiến kinh nghiệm: Giải một bài toán quỹ tích như thế nào - Trường THCS V...Học Tập Long An
 
De thi vao truong le hong phong (hcm) truonghocso.com
De thi vao truong le hong phong (hcm)   truonghocso.comDe thi vao truong le hong phong (hcm)   truonghocso.com
De thi vao truong le hong phong (hcm) truonghocso.comThế Giới Tinh Hoa
 
Phuong phap-tim-cac-loai-khoang-cach-trong-hinh-hoc-khong-gian
Phuong phap-tim-cac-loai-khoang-cach-trong-hinh-hoc-khong-gianPhuong phap-tim-cac-loai-khoang-cach-trong-hinh-hoc-khong-gian
Phuong phap-tim-cac-loai-khoang-cach-trong-hinh-hoc-khong-gianĐức Mạnh Ngô
 
The tich-khoi-da-dien-hv phien-www.mathvn.com
The tich-khoi-da-dien-hv phien-www.mathvn.comThe tich-khoi-da-dien-hv phien-www.mathvn.com
The tich-khoi-da-dien-hv phien-www.mathvn.comsmile258
 
Lý thuyết và các dạng toán liên quan tới Vectơ.pdf
Lý thuyết và các dạng toán liên quan tới Vectơ.pdfLý thuyết và các dạng toán liên quan tới Vectơ.pdf
Lý thuyết và các dạng toán liên quan tới Vectơ.pdfcuongTa20
 
[Toanmath.com] 50 câu trắc nghiệm mặt cầu, mặt trụ, mặt nón - trần công diêu
[Toanmath.com]   50 câu trắc nghiệm mặt cầu, mặt trụ, mặt nón - trần công diêu[Toanmath.com]   50 câu trắc nghiệm mặt cầu, mặt trụ, mặt nón - trần công diêu
[Toanmath.com] 50 câu trắc nghiệm mặt cầu, mặt trụ, mặt nón - trần công diêuMười Nguyễn
 
Cac dinh-ly-dong-quy copy
Cac dinh-ly-dong-quy copyCac dinh-ly-dong-quy copy
Cac dinh-ly-dong-quy copyThai An Nguyen
 
On tap toan 7 hoc ki ii va de thi hoc ki ii
On tap toan 7 hoc ki ii va de thi hoc ki iiOn tap toan 7 hoc ki ii va de thi hoc ki ii
On tap toan 7 hoc ki ii va de thi hoc ki iiTu Em
 
03 bai toan giai tam giac p1_bg
03 bai toan giai tam giac p1_bg03 bai toan giai tam giac p1_bg
03 bai toan giai tam giac p1_bgNgoc Diep Ngocdiep
 
03 bai toan giai tam giac p1_bg
03 bai toan giai tam giac p1_bg03 bai toan giai tam giac p1_bg
03 bai toan giai tam giac p1_bgNgoc Diep Ngocdiep
 
Ly thuyet-va-bai-tap-trac-nghiem-chuyen-de-to-hop-xac-suat (1)
Ly thuyet-va-bai-tap-trac-nghiem-chuyen-de-to-hop-xac-suat (1)Ly thuyet-va-bai-tap-trac-nghiem-chuyen-de-to-hop-xac-suat (1)
Ly thuyet-va-bai-tap-trac-nghiem-chuyen-de-to-hop-xac-suat (1)o0onhuquynh
 
100 câu hỏi trắc nghiệm thể tích khối đa diện có đáp án - iHoc.me
100 câu hỏi trắc nghiệm thể tích khối đa diện có đáp án - iHoc.me100 câu hỏi trắc nghiệm thể tích khối đa diện có đáp án - iHoc.me
100 câu hỏi trắc nghiệm thể tích khối đa diện có đáp án - iHoc.mehaic2hv.net
 

Similar to Sử Dụng Hàng Điểm Điều Hòa Trong Giải Toán Hình Học Phẳng.doc (20)

Sử dụng hàng điểm điều hòa trong giải toán hình học phẳng.doc
Sử dụng hàng điểm điều hòa trong giải toán hình học phẳng.docSử dụng hàng điểm điều hòa trong giải toán hình học phẳng.doc
Sử dụng hàng điểm điều hòa trong giải toán hình học phẳng.doc
 
Phương Pháp Diện Tích Và Thể Tích Trong Hình Học Sơ Cấp.doc
Phương Pháp Diện Tích Và Thể Tích Trong Hình Học Sơ Cấp.docPhương Pháp Diện Tích Và Thể Tích Trong Hình Học Sơ Cấp.doc
Phương Pháp Diện Tích Và Thể Tích Trong Hình Học Sơ Cấp.doc
 
VECTƠ VÀ CÁC ỨNG DỤNG
VECTƠ VÀ CÁC ỨNG DỤNGVECTƠ VÀ CÁC ỨNG DỤNG
VECTƠ VÀ CÁC ỨNG DỤNG
 
8 chuyen de hinh hoc phang
8 chuyen de hinh hoc phang8 chuyen de hinh hoc phang
8 chuyen de hinh hoc phang
 
Pp tinh the tich
Pp tinh the tichPp tinh the tich
Pp tinh the tich
 
Chuyendequytichbacthcs6155 141225015434-conversion-gate02
Chuyendequytichbacthcs6155 141225015434-conversion-gate02Chuyendequytichbacthcs6155 141225015434-conversion-gate02
Chuyendequytichbacthcs6155 141225015434-conversion-gate02
 
Sáng kiến kinh nghiệm: Giải một bài toán quỹ tích như thế nào - Trường THCS V...
Sáng kiến kinh nghiệm: Giải một bài toán quỹ tích như thế nào - Trường THCS V...Sáng kiến kinh nghiệm: Giải một bài toán quỹ tích như thế nào - Trường THCS V...
Sáng kiến kinh nghiệm: Giải một bài toán quỹ tích như thế nào - Trường THCS V...
 
40 0975
40 097540 0975
40 0975
 
De thi vao truong le hong phong (hcm) truonghocso.com
De thi vao truong le hong phong (hcm)   truonghocso.comDe thi vao truong le hong phong (hcm)   truonghocso.com
De thi vao truong le hong phong (hcm) truonghocso.com
 
Phuong phap-tim-cac-loai-khoang-cach-trong-hinh-hoc-khong-gian
Phuong phap-tim-cac-loai-khoang-cach-trong-hinh-hoc-khong-gianPhuong phap-tim-cac-loai-khoang-cach-trong-hinh-hoc-khong-gian
Phuong phap-tim-cac-loai-khoang-cach-trong-hinh-hoc-khong-gian
 
The tich-khoi-da-dien-hv phien-www.mathvn.com
The tich-khoi-da-dien-hv phien-www.mathvn.comThe tich-khoi-da-dien-hv phien-www.mathvn.com
The tich-khoi-da-dien-hv phien-www.mathvn.com
 
Lý thuyết và các dạng toán liên quan tới Vectơ.pdf
Lý thuyết và các dạng toán liên quan tới Vectơ.pdfLý thuyết và các dạng toán liên quan tới Vectơ.pdf
Lý thuyết và các dạng toán liên quan tới Vectơ.pdf
 
[Toanmath.com] 50 câu trắc nghiệm mặt cầu, mặt trụ, mặt nón - trần công diêu
[Toanmath.com]   50 câu trắc nghiệm mặt cầu, mặt trụ, mặt nón - trần công diêu[Toanmath.com]   50 câu trắc nghiệm mặt cầu, mặt trụ, mặt nón - trần công diêu
[Toanmath.com] 50 câu trắc nghiệm mặt cầu, mặt trụ, mặt nón - trần công diêu
 
Cac dinh-ly-dong-quy copy
Cac dinh-ly-dong-quy copyCac dinh-ly-dong-quy copy
Cac dinh-ly-dong-quy copy
 
On tap toan 7 hoc ki ii va de thi hoc ki ii
On tap toan 7 hoc ki ii va de thi hoc ki iiOn tap toan 7 hoc ki ii va de thi hoc ki ii
On tap toan 7 hoc ki ii va de thi hoc ki ii
 
03 bai toan giai tam giac p1_bg
03 bai toan giai tam giac p1_bg03 bai toan giai tam giac p1_bg
03 bai toan giai tam giac p1_bg
 
03 bai toan giai tam giac p1_bg
03 bai toan giai tam giac p1_bg03 bai toan giai tam giac p1_bg
03 bai toan giai tam giac p1_bg
 
Ly thuyet-va-bai-tap-trac-nghiem-chuyen-de-to-hop-xac-suat (1)
Ly thuyet-va-bai-tap-trac-nghiem-chuyen-de-to-hop-xac-suat (1)Ly thuyet-va-bai-tap-trac-nghiem-chuyen-de-to-hop-xac-suat (1)
Ly thuyet-va-bai-tap-trac-nghiem-chuyen-de-to-hop-xac-suat (1)
 
40 de-thi-toan-vao-10-chon-loc
40 de-thi-toan-vao-10-chon-loc40 de-thi-toan-vao-10-chon-loc
40 de-thi-toan-vao-10-chon-loc
 
100 câu hỏi trắc nghiệm thể tích khối đa diện có đáp án - iHoc.me
100 câu hỏi trắc nghiệm thể tích khối đa diện có đáp án - iHoc.me100 câu hỏi trắc nghiệm thể tích khối đa diện có đáp án - iHoc.me
100 câu hỏi trắc nghiệm thể tích khối đa diện có đáp án - iHoc.me
 

More from DV Viết Luận văn luanvanmaster.com ZALO 0973287149

More from DV Viết Luận văn luanvanmaster.com ZALO 0973287149 (20)

Ảnh Hưởng Của Marketing Quan Hệ Đến Lòng Trung Thành Của Khách Hàng.Tình Huốn...
Ảnh Hưởng Của Marketing Quan Hệ Đến Lòng Trung Thành Của Khách Hàng.Tình Huốn...Ảnh Hưởng Của Marketing Quan Hệ Đến Lòng Trung Thành Của Khách Hàng.Tình Huốn...
Ảnh Hưởng Của Marketing Quan Hệ Đến Lòng Trung Thành Của Khách Hàng.Tình Huốn...
 
Phát triển nguồn nhân lực tại Uỷ ban nhân dân huyện Trà Bồng, tỉnh Quảng Ngãi...
Phát triển nguồn nhân lực tại Uỷ ban nhân dân huyện Trà Bồng, tỉnh Quảng Ngãi...Phát triển nguồn nhân lực tại Uỷ ban nhân dân huyện Trà Bồng, tỉnh Quảng Ngãi...
Phát triển nguồn nhân lực tại Uỷ ban nhân dân huyện Trà Bồng, tỉnh Quảng Ngãi...
 
Báo cáo tốt Nghiệp tài chính hợp nhất tại tổng công ty Indochina gol...
Báo cáo tốt Nghiệp  tài chính hợp nhất tại tổng công ty Indochina gol...Báo cáo tốt Nghiệp  tài chính hợp nhất tại tổng công ty Indochina gol...
Báo cáo tốt Nghiệp tài chính hợp nhất tại tổng công ty Indochina gol...
 
Tạo động lực thúc đẩy nhân viên làm việc tại ngân hàng TMCP Ngoại Thương Việt...
Tạo động lực thúc đẩy nhân viên làm việc tại ngân hàng TMCP Ngoại Thương Việt...Tạo động lực thúc đẩy nhân viên làm việc tại ngân hàng TMCP Ngoại Thương Việt...
Tạo động lực thúc đẩy nhân viên làm việc tại ngân hàng TMCP Ngoại Thương Việt...
 
Phát triển công nghiệp trên địa bàn Thành phố Tam Kỳ, Tỉnh Quảng Na...
Phát triển công nghiệp trên địa bàn Thành phố Tam Kỳ, Tỉnh Quảng Na...Phát triển công nghiệp trên địa bàn Thành phố Tam Kỳ, Tỉnh Quảng Na...
Phát triển công nghiệp trên địa bàn Thành phố Tam Kỳ, Tỉnh Quảng Na...
 
Giải pháp phát triển cho vay xuất nhập khẩu tại ngân hàng NN&PTNN ch...
Giải pháp phát triển cho vay xuất nhập khẩu tại ngân hàng NN&PTNN ch...Giải pháp phát triển cho vay xuất nhập khẩu tại ngân hàng NN&PTNN ch...
Giải pháp phát triển cho vay xuất nhập khẩu tại ngân hàng NN&PTNN ch...
 
Hoàn thiện công tác lập báo cáo tài chính hợp nhất tại tổng công ...
Hoàn thiện công tác lập báo cáo tài chính hợp nhất tại tổng công ...Hoàn thiện công tác lập báo cáo tài chính hợp nhất tại tổng công ...
Hoàn thiện công tác lập báo cáo tài chính hợp nhất tại tổng công ...
 
Luận Văn Thạc Sĩ Quản trị thành tích nhân viên tại Cục Hải quan TP Đà Nẵng.doc
Luận Văn Thạc Sĩ  Quản trị thành tích nhân viên tại Cục Hải quan TP Đà Nẵng.docLuận Văn Thạc Sĩ  Quản trị thành tích nhân viên tại Cục Hải quan TP Đà Nẵng.doc
Luận Văn Thạc Sĩ Quản trị thành tích nhân viên tại Cục Hải quan TP Đà Nẵng.doc
 
Hoàn thiện công tác quản lý thuế thu nhập cá nhân tại cục thuế Tỉ...
Hoàn thiện công tác quản lý thuế thu nhập cá nhân tại cục thuế Tỉ...Hoàn thiện công tác quản lý thuế thu nhập cá nhân tại cục thuế Tỉ...
Hoàn thiện công tác quản lý thuế thu nhập cá nhân tại cục thuế Tỉ...
 
Đề Tài Phát triển bền vững nông nghiệp Huyện Ba Tơ, Tỉnh Quảng Ngãi....
Đề Tài Phát triển bền vững nông nghiệp Huyện Ba Tơ, Tỉnh Quảng Ngãi....Đề Tài Phát triển bền vững nông nghiệp Huyện Ba Tơ, Tỉnh Quảng Ngãi....
Đề Tài Phát triển bền vững nông nghiệp Huyện Ba Tơ, Tỉnh Quảng Ngãi....
 
Hoàn thiện công tác bảo trợ xã hội trên địa bàn huyện Phong Điền, tỉnh Thừa T...
Hoàn thiện công tác bảo trợ xã hội trên địa bàn huyện Phong Điền, tỉnh Thừa T...Hoàn thiện công tác bảo trợ xã hội trên địa bàn huyện Phong Điền, tỉnh Thừa T...
Hoàn thiện công tác bảo trợ xã hội trên địa bàn huyện Phong Điền, tỉnh Thừa T...
 
Đề Tài Luận VănPhát triển sản phẩm du lịch tại thành phố Đà Nẵng.doc
Đề Tài Luận VănPhát triển sản phẩm du lịch tại thành phố Đà Nẵng.docĐề Tài Luận VănPhát triển sản phẩm du lịch tại thành phố Đà Nẵng.doc
Đề Tài Luận VănPhát triển sản phẩm du lịch tại thành phố Đà Nẵng.doc
 
Đào tạo nghề cho lao động thuộc diện thu hồi đất trên địa bàn Thàn...
Đào tạo nghề cho lao động thuộc diện thu hồi đất trên địa bàn Thàn...Đào tạo nghề cho lao động thuộc diện thu hồi đất trên địa bàn Thàn...
Đào tạo nghề cho lao động thuộc diện thu hồi đất trên địa bàn Thàn...
 
Tóm Tắt Luận Văn Thạc Sĩ Quản Trị Kinh Doanh Xây dựng chính sách Marketing tạ...
Tóm Tắt Luận Văn Thạc Sĩ Quản Trị Kinh Doanh Xây dựng chính sách Marketing tạ...Tóm Tắt Luận Văn Thạc Sĩ Quản Trị Kinh Doanh Xây dựng chính sách Marketing tạ...
Tóm Tắt Luận Văn Thạc Sĩ Quản Trị Kinh Doanh Xây dựng chính sách Marketing tạ...
 
Đề Tài Nghiên cứu rủi ro cảm nhận đối với mua hàng thời trang trực tuyến.docx
Đề Tài Nghiên cứu rủi ro cảm nhận đối với mua hàng thời trang trực tuyến.docxĐề Tài Nghiên cứu rủi ro cảm nhận đối với mua hàng thời trang trực tuyến.docx
Đề Tài Nghiên cứu rủi ro cảm nhận đối với mua hàng thời trang trực tuyến.docx
 
Giải pháp nâng cao động lực thúc đẩy người lao động tại công ty khai...
Giải pháp nâng cao động lực thúc đẩy người lao động tại công ty khai...Giải pháp nâng cao động lực thúc đẩy người lao động tại công ty khai...
Giải pháp nâng cao động lực thúc đẩy người lao động tại công ty khai...
 
Giải pháp phát triển dịch vụ ngân hàng điện tử tại ngân hàng đầu ...
Giải pháp phát triển dịch vụ ngân hàng điện tử tại ngân hàng đầu ...Giải pháp phát triển dịch vụ ngân hàng điện tử tại ngân hàng đầu ...
Giải pháp phát triển dịch vụ ngân hàng điện tử tại ngân hàng đầu ...
 
Giải pháp phát triển dịch vụ ngân hàng điện tử tại ngân hàng đầu ...
Giải pháp phát triển dịch vụ ngân hàng điện tử tại ngân hàng đầu ...Giải pháp phát triển dịch vụ ngân hàng điện tử tại ngân hàng đầu ...
Giải pháp phát triển dịch vụ ngân hàng điện tử tại ngân hàng đầu ...
 
Quản trị quan hệ khách hàng tại Chi nhánh Viettel Đà Nẵng – Tập đoàn Viễn thô...
Quản trị quan hệ khách hàng tại Chi nhánh Viettel Đà Nẵng – Tập đoàn Viễn thô...Quản trị quan hệ khách hàng tại Chi nhánh Viettel Đà Nẵng – Tập đoàn Viễn thô...
Quản trị quan hệ khách hàng tại Chi nhánh Viettel Đà Nẵng – Tập đoàn Viễn thô...
 
Đề Tài Đánh giá thành tích đội ngũ giảng viên trường Đại Học Phạm ...
Đề Tài Đánh giá thành tích đội ngũ giảng viên trường Đại Học Phạm ...Đề Tài Đánh giá thành tích đội ngũ giảng viên trường Đại Học Phạm ...
Đề Tài Đánh giá thành tích đội ngũ giảng viên trường Đại Học Phạm ...
 

Recently uploaded

30 ĐỀ PHÁT TRIỂN THEO CẤU TRÚC ĐỀ MINH HỌA BGD NGÀY 22-3-2024 KỲ THI TỐT NGHI...
30 ĐỀ PHÁT TRIỂN THEO CẤU TRÚC ĐỀ MINH HỌA BGD NGÀY 22-3-2024 KỲ THI TỐT NGHI...30 ĐỀ PHÁT TRIỂN THEO CẤU TRÚC ĐỀ MINH HỌA BGD NGÀY 22-3-2024 KỲ THI TỐT NGHI...
30 ĐỀ PHÁT TRIỂN THEO CẤU TRÚC ĐỀ MINH HỌA BGD NGÀY 22-3-2024 KỲ THI TỐT NGHI...Nguyen Thanh Tu Collection
 
Thong bao 337-DHPY (24.4.2024) thi sat hach Ngoai ngu dap ung Chuan dau ra do...
Thong bao 337-DHPY (24.4.2024) thi sat hach Ngoai ngu dap ung Chuan dau ra do...Thong bao 337-DHPY (24.4.2024) thi sat hach Ngoai ngu dap ung Chuan dau ra do...
Thong bao 337-DHPY (24.4.2024) thi sat hach Ngoai ngu dap ung Chuan dau ra do...hoangtuansinh1
 
QUẢN LÝ HOẠT ĐỘNG GIÁO DỤC KỸ NĂNG SỐNG CHO HỌC SINH CÁC TRƯỜNG TRUNG HỌC CƠ ...
QUẢN LÝ HOẠT ĐỘNG GIÁO DỤC KỸ NĂNG SỐNG CHO HỌC SINH CÁC TRƯỜNG TRUNG HỌC CƠ ...QUẢN LÝ HOẠT ĐỘNG GIÁO DỤC KỸ NĂNG SỐNG CHO HỌC SINH CÁC TRƯỜNG TRUNG HỌC CƠ ...
QUẢN LÝ HOẠT ĐỘNG GIÁO DỤC KỸ NĂNG SỐNG CHO HỌC SINH CÁC TRƯỜNG TRUNG HỌC CƠ ...ThunTrn734461
 
BỘ ĐỀ PHÁT TRIỂN THEO CẤU TRÚC ĐỀ MINH HỌA BGD NGÀY 22-3-2024 KỲ THI TỐT NGHI...
BỘ ĐỀ PHÁT TRIỂN THEO CẤU TRÚC ĐỀ MINH HỌA BGD NGÀY 22-3-2024 KỲ THI TỐT NGHI...BỘ ĐỀ PHÁT TRIỂN THEO CẤU TRÚC ĐỀ MINH HỌA BGD NGÀY 22-3-2024 KỲ THI TỐT NGHI...
BỘ ĐỀ PHÁT TRIỂN THEO CẤU TRÚC ĐỀ MINH HỌA BGD NGÀY 22-3-2024 KỲ THI TỐT NGHI...Nguyen Thanh Tu Collection
 
Sơ đồ tư duy môn sinh học bậc THPT.pdf
Sơ đồ tư duy môn sinh học bậc THPT.pdfSơ đồ tư duy môn sinh học bậc THPT.pdf
Sơ đồ tư duy môn sinh học bậc THPT.pdftohoanggiabao81
 
ôn tập lịch sử hhhhhhhhhhhhhhhhhhhhhhhhhh
ôn tập lịch sử hhhhhhhhhhhhhhhhhhhhhhhhhhôn tập lịch sử hhhhhhhhhhhhhhhhhhhhhhhhhh
ôn tập lịch sử hhhhhhhhhhhhhhhhhhhhhhhhhhvanhathvc
 
NQA Lợi ích Từ ISO và ESG Tăng Trưởng và Bền Vững ver01.pdf
NQA Lợi ích Từ ISO và ESG Tăng Trưởng và Bền Vững ver01.pdfNQA Lợi ích Từ ISO và ESG Tăng Trưởng và Bền Vững ver01.pdf
NQA Lợi ích Từ ISO và ESG Tăng Trưởng và Bền Vững ver01.pdfNguyễn Đăng Quang
 
Trích dẫn trắc nghiệm tư tưởng HCM5.docx
Trích dẫn trắc nghiệm tư tưởng HCM5.docxTrích dẫn trắc nghiệm tư tưởng HCM5.docx
Trích dẫn trắc nghiệm tư tưởng HCM5.docxnhungdt08102004
 
TỔNG HỢP ĐỀ THI CHÍNH THỨC KỲ THI TUYỂN SINH VÀO LỚP 10 THPT MÔN NGỮ VĂN NĂM ...
TỔNG HỢP ĐỀ THI CHÍNH THỨC KỲ THI TUYỂN SINH VÀO LỚP 10 THPT MÔN NGỮ VĂN NĂM ...TỔNG HỢP ĐỀ THI CHÍNH THỨC KỲ THI TUYỂN SINH VÀO LỚP 10 THPT MÔN NGỮ VĂN NĂM ...
TỔNG HỢP ĐỀ THI CHÍNH THỨC KỲ THI TUYỂN SINH VÀO LỚP 10 THPT MÔN NGỮ VĂN NĂM ...Nguyen Thanh Tu Collection
 
10 ĐỀ KIỂM TRA + 6 ĐỀ ÔN TẬP CUỐI KÌ 2 VẬT LÝ 11 - KẾT NỐI TRI THỨC - THEO C...
10 ĐỀ KIỂM TRA + 6 ĐỀ ÔN TẬP CUỐI KÌ 2 VẬT LÝ 11 - KẾT NỐI TRI THỨC - THEO C...10 ĐỀ KIỂM TRA + 6 ĐỀ ÔN TẬP CUỐI KÌ 2 VẬT LÝ 11 - KẾT NỐI TRI THỨC - THEO C...
10 ĐỀ KIỂM TRA + 6 ĐỀ ÔN TẬP CUỐI KÌ 2 VẬT LÝ 11 - KẾT NỐI TRI THỨC - THEO C...Nguyen Thanh Tu Collection
 
Kiểm tra chạy trạm lí thuyết giữa kì giải phẫu sinh lí
Kiểm tra chạy trạm lí thuyết giữa kì giải phẫu sinh líKiểm tra chạy trạm lí thuyết giữa kì giải phẫu sinh lí
Kiểm tra chạy trạm lí thuyết giữa kì giải phẫu sinh líDr K-OGN
 
Sáng kiến “Sử dụng ứng dụng Quizizz nhằm nâng cao chất lượng ôn thi tốt nghiệ...
Sáng kiến “Sử dụng ứng dụng Quizizz nhằm nâng cao chất lượng ôn thi tốt nghiệ...Sáng kiến “Sử dụng ứng dụng Quizizz nhằm nâng cao chất lượng ôn thi tốt nghiệ...
Sáng kiến “Sử dụng ứng dụng Quizizz nhằm nâng cao chất lượng ôn thi tốt nghiệ...Nguyen Thanh Tu Collection
 
Chàm - Bệnh án (da liễu - bvdlct ctump) .pptx
Chàm - Bệnh án (da liễu - bvdlct ctump) .pptxChàm - Bệnh án (da liễu - bvdlct ctump) .pptx
Chàm - Bệnh án (da liễu - bvdlct ctump) .pptxendkay31
 
BỘ ĐỀ KIỂM TRA CUỐI KÌ 2 VẬT LÝ 11 - KẾT NỐI TRI THỨC - THEO CẤU TRÚC ĐỀ MIN...
BỘ ĐỀ KIỂM TRA CUỐI KÌ 2 VẬT LÝ 11 - KẾT NỐI TRI THỨC - THEO CẤU TRÚC ĐỀ MIN...BỘ ĐỀ KIỂM TRA CUỐI KÌ 2 VẬT LÝ 11 - KẾT NỐI TRI THỨC - THEO CẤU TRÚC ĐỀ MIN...
BỘ ĐỀ KIỂM TRA CUỐI KÌ 2 VẬT LÝ 11 - KẾT NỐI TRI THỨC - THEO CẤU TRÚC ĐỀ MIN...Nguyen Thanh Tu Collection
 
Chuong trinh dao tao Su pham Khoa hoc tu nhien, ma nganh - 7140247.pdf
Chuong trinh dao tao Su pham Khoa hoc tu nhien, ma nganh - 7140247.pdfChuong trinh dao tao Su pham Khoa hoc tu nhien, ma nganh - 7140247.pdf
Chuong trinh dao tao Su pham Khoa hoc tu nhien, ma nganh - 7140247.pdfhoangtuansinh1
 
SÁNG KIẾN “THIẾT KẾ VÀ SỬ DỤNG INFOGRAPHIC TRONG DẠY HỌC ĐỊA LÍ 11 (BỘ SÁCH K...
SÁNG KIẾN “THIẾT KẾ VÀ SỬ DỤNG INFOGRAPHIC TRONG DẠY HỌC ĐỊA LÍ 11 (BỘ SÁCH K...SÁNG KIẾN “THIẾT KẾ VÀ SỬ DỤNG INFOGRAPHIC TRONG DẠY HỌC ĐỊA LÍ 11 (BỘ SÁCH K...
SÁNG KIẾN “THIẾT KẾ VÀ SỬ DỤNG INFOGRAPHIC TRONG DẠY HỌC ĐỊA LÍ 11 (BỘ SÁCH K...Nguyen Thanh Tu Collection
 
30 ĐỀ PHÁT TRIỂN THEO CẤU TRÚC ĐỀ MINH HỌA BGD NGÀY 22-3-2024 KỲ THI TỐT NGHI...
30 ĐỀ PHÁT TRIỂN THEO CẤU TRÚC ĐỀ MINH HỌA BGD NGÀY 22-3-2024 KỲ THI TỐT NGHI...30 ĐỀ PHÁT TRIỂN THEO CẤU TRÚC ĐỀ MINH HỌA BGD NGÀY 22-3-2024 KỲ THI TỐT NGHI...
30 ĐỀ PHÁT TRIỂN THEO CẤU TRÚC ĐỀ MINH HỌA BGD NGÀY 22-3-2024 KỲ THI TỐT NGHI...Nguyen Thanh Tu Collection
 
bài 5.1.docx Sinh học di truyền đại cương năm nhất của học sinh y đa khoa
bài 5.1.docx Sinh học di truyền đại cương năm nhất của học sinh y đa khoabài 5.1.docx Sinh học di truyền đại cương năm nhất của học sinh y đa khoa
bài 5.1.docx Sinh học di truyền đại cương năm nhất của học sinh y đa khoa2353020138
 
Sáng kiến Dạy học theo định hướng STEM một số chủ đề phần “vật sống”, Khoa họ...
Sáng kiến Dạy học theo định hướng STEM một số chủ đề phần “vật sống”, Khoa họ...Sáng kiến Dạy học theo định hướng STEM một số chủ đề phần “vật sống”, Khoa họ...
Sáng kiến Dạy học theo định hướng STEM một số chủ đề phần “vật sống”, Khoa họ...Nguyen Thanh Tu Collection
 

Recently uploaded (19)

30 ĐỀ PHÁT TRIỂN THEO CẤU TRÚC ĐỀ MINH HỌA BGD NGÀY 22-3-2024 KỲ THI TỐT NGHI...
30 ĐỀ PHÁT TRIỂN THEO CẤU TRÚC ĐỀ MINH HỌA BGD NGÀY 22-3-2024 KỲ THI TỐT NGHI...30 ĐỀ PHÁT TRIỂN THEO CẤU TRÚC ĐỀ MINH HỌA BGD NGÀY 22-3-2024 KỲ THI TỐT NGHI...
30 ĐỀ PHÁT TRIỂN THEO CẤU TRÚC ĐỀ MINH HỌA BGD NGÀY 22-3-2024 KỲ THI TỐT NGHI...
 
Thong bao 337-DHPY (24.4.2024) thi sat hach Ngoai ngu dap ung Chuan dau ra do...
Thong bao 337-DHPY (24.4.2024) thi sat hach Ngoai ngu dap ung Chuan dau ra do...Thong bao 337-DHPY (24.4.2024) thi sat hach Ngoai ngu dap ung Chuan dau ra do...
Thong bao 337-DHPY (24.4.2024) thi sat hach Ngoai ngu dap ung Chuan dau ra do...
 
QUẢN LÝ HOẠT ĐỘNG GIÁO DỤC KỸ NĂNG SỐNG CHO HỌC SINH CÁC TRƯỜNG TRUNG HỌC CƠ ...
QUẢN LÝ HOẠT ĐỘNG GIÁO DỤC KỸ NĂNG SỐNG CHO HỌC SINH CÁC TRƯỜNG TRUNG HỌC CƠ ...QUẢN LÝ HOẠT ĐỘNG GIÁO DỤC KỸ NĂNG SỐNG CHO HỌC SINH CÁC TRƯỜNG TRUNG HỌC CƠ ...
QUẢN LÝ HOẠT ĐỘNG GIÁO DỤC KỸ NĂNG SỐNG CHO HỌC SINH CÁC TRƯỜNG TRUNG HỌC CƠ ...
 
BỘ ĐỀ PHÁT TRIỂN THEO CẤU TRÚC ĐỀ MINH HỌA BGD NGÀY 22-3-2024 KỲ THI TỐT NGHI...
BỘ ĐỀ PHÁT TRIỂN THEO CẤU TRÚC ĐỀ MINH HỌA BGD NGÀY 22-3-2024 KỲ THI TỐT NGHI...BỘ ĐỀ PHÁT TRIỂN THEO CẤU TRÚC ĐỀ MINH HỌA BGD NGÀY 22-3-2024 KỲ THI TỐT NGHI...
BỘ ĐỀ PHÁT TRIỂN THEO CẤU TRÚC ĐỀ MINH HỌA BGD NGÀY 22-3-2024 KỲ THI TỐT NGHI...
 
Sơ đồ tư duy môn sinh học bậc THPT.pdf
Sơ đồ tư duy môn sinh học bậc THPT.pdfSơ đồ tư duy môn sinh học bậc THPT.pdf
Sơ đồ tư duy môn sinh học bậc THPT.pdf
 
ôn tập lịch sử hhhhhhhhhhhhhhhhhhhhhhhhhh
ôn tập lịch sử hhhhhhhhhhhhhhhhhhhhhhhhhhôn tập lịch sử hhhhhhhhhhhhhhhhhhhhhhhhhh
ôn tập lịch sử hhhhhhhhhhhhhhhhhhhhhhhhhh
 
NQA Lợi ích Từ ISO và ESG Tăng Trưởng và Bền Vững ver01.pdf
NQA Lợi ích Từ ISO và ESG Tăng Trưởng và Bền Vững ver01.pdfNQA Lợi ích Từ ISO và ESG Tăng Trưởng và Bền Vững ver01.pdf
NQA Lợi ích Từ ISO và ESG Tăng Trưởng và Bền Vững ver01.pdf
 
Trích dẫn trắc nghiệm tư tưởng HCM5.docx
Trích dẫn trắc nghiệm tư tưởng HCM5.docxTrích dẫn trắc nghiệm tư tưởng HCM5.docx
Trích dẫn trắc nghiệm tư tưởng HCM5.docx
 
TỔNG HỢP ĐỀ THI CHÍNH THỨC KỲ THI TUYỂN SINH VÀO LỚP 10 THPT MÔN NGỮ VĂN NĂM ...
TỔNG HỢP ĐỀ THI CHÍNH THỨC KỲ THI TUYỂN SINH VÀO LỚP 10 THPT MÔN NGỮ VĂN NĂM ...TỔNG HỢP ĐỀ THI CHÍNH THỨC KỲ THI TUYỂN SINH VÀO LỚP 10 THPT MÔN NGỮ VĂN NĂM ...
TỔNG HỢP ĐỀ THI CHÍNH THỨC KỲ THI TUYỂN SINH VÀO LỚP 10 THPT MÔN NGỮ VĂN NĂM ...
 
10 ĐỀ KIỂM TRA + 6 ĐỀ ÔN TẬP CUỐI KÌ 2 VẬT LÝ 11 - KẾT NỐI TRI THỨC - THEO C...
10 ĐỀ KIỂM TRA + 6 ĐỀ ÔN TẬP CUỐI KÌ 2 VẬT LÝ 11 - KẾT NỐI TRI THỨC - THEO C...10 ĐỀ KIỂM TRA + 6 ĐỀ ÔN TẬP CUỐI KÌ 2 VẬT LÝ 11 - KẾT NỐI TRI THỨC - THEO C...
10 ĐỀ KIỂM TRA + 6 ĐỀ ÔN TẬP CUỐI KÌ 2 VẬT LÝ 11 - KẾT NỐI TRI THỨC - THEO C...
 
Kiểm tra chạy trạm lí thuyết giữa kì giải phẫu sinh lí
Kiểm tra chạy trạm lí thuyết giữa kì giải phẫu sinh líKiểm tra chạy trạm lí thuyết giữa kì giải phẫu sinh lí
Kiểm tra chạy trạm lí thuyết giữa kì giải phẫu sinh lí
 
Sáng kiến “Sử dụng ứng dụng Quizizz nhằm nâng cao chất lượng ôn thi tốt nghiệ...
Sáng kiến “Sử dụng ứng dụng Quizizz nhằm nâng cao chất lượng ôn thi tốt nghiệ...Sáng kiến “Sử dụng ứng dụng Quizizz nhằm nâng cao chất lượng ôn thi tốt nghiệ...
Sáng kiến “Sử dụng ứng dụng Quizizz nhằm nâng cao chất lượng ôn thi tốt nghiệ...
 
Chàm - Bệnh án (da liễu - bvdlct ctump) .pptx
Chàm - Bệnh án (da liễu - bvdlct ctump) .pptxChàm - Bệnh án (da liễu - bvdlct ctump) .pptx
Chàm - Bệnh án (da liễu - bvdlct ctump) .pptx
 
BỘ ĐỀ KIỂM TRA CUỐI KÌ 2 VẬT LÝ 11 - KẾT NỐI TRI THỨC - THEO CẤU TRÚC ĐỀ MIN...
BỘ ĐỀ KIỂM TRA CUỐI KÌ 2 VẬT LÝ 11 - KẾT NỐI TRI THỨC - THEO CẤU TRÚC ĐỀ MIN...BỘ ĐỀ KIỂM TRA CUỐI KÌ 2 VẬT LÝ 11 - KẾT NỐI TRI THỨC - THEO CẤU TRÚC ĐỀ MIN...
BỘ ĐỀ KIỂM TRA CUỐI KÌ 2 VẬT LÝ 11 - KẾT NỐI TRI THỨC - THEO CẤU TRÚC ĐỀ MIN...
 
Chuong trinh dao tao Su pham Khoa hoc tu nhien, ma nganh - 7140247.pdf
Chuong trinh dao tao Su pham Khoa hoc tu nhien, ma nganh - 7140247.pdfChuong trinh dao tao Su pham Khoa hoc tu nhien, ma nganh - 7140247.pdf
Chuong trinh dao tao Su pham Khoa hoc tu nhien, ma nganh - 7140247.pdf
 
SÁNG KIẾN “THIẾT KẾ VÀ SỬ DỤNG INFOGRAPHIC TRONG DẠY HỌC ĐỊA LÍ 11 (BỘ SÁCH K...
SÁNG KIẾN “THIẾT KẾ VÀ SỬ DỤNG INFOGRAPHIC TRONG DẠY HỌC ĐỊA LÍ 11 (BỘ SÁCH K...SÁNG KIẾN “THIẾT KẾ VÀ SỬ DỤNG INFOGRAPHIC TRONG DẠY HỌC ĐỊA LÍ 11 (BỘ SÁCH K...
SÁNG KIẾN “THIẾT KẾ VÀ SỬ DỤNG INFOGRAPHIC TRONG DẠY HỌC ĐỊA LÍ 11 (BỘ SÁCH K...
 
30 ĐỀ PHÁT TRIỂN THEO CẤU TRÚC ĐỀ MINH HỌA BGD NGÀY 22-3-2024 KỲ THI TỐT NGHI...
30 ĐỀ PHÁT TRIỂN THEO CẤU TRÚC ĐỀ MINH HỌA BGD NGÀY 22-3-2024 KỲ THI TỐT NGHI...30 ĐỀ PHÁT TRIỂN THEO CẤU TRÚC ĐỀ MINH HỌA BGD NGÀY 22-3-2024 KỲ THI TỐT NGHI...
30 ĐỀ PHÁT TRIỂN THEO CẤU TRÚC ĐỀ MINH HỌA BGD NGÀY 22-3-2024 KỲ THI TỐT NGHI...
 
bài 5.1.docx Sinh học di truyền đại cương năm nhất của học sinh y đa khoa
bài 5.1.docx Sinh học di truyền đại cương năm nhất của học sinh y đa khoabài 5.1.docx Sinh học di truyền đại cương năm nhất của học sinh y đa khoa
bài 5.1.docx Sinh học di truyền đại cương năm nhất của học sinh y đa khoa
 
Sáng kiến Dạy học theo định hướng STEM một số chủ đề phần “vật sống”, Khoa họ...
Sáng kiến Dạy học theo định hướng STEM một số chủ đề phần “vật sống”, Khoa họ...Sáng kiến Dạy học theo định hướng STEM một số chủ đề phần “vật sống”, Khoa họ...
Sáng kiến Dạy học theo định hướng STEM một số chủ đề phần “vật sống”, Khoa họ...
 

Sử Dụng Hàng Điểm Điều Hòa Trong Giải Toán Hình Học Phẳng.doc

  • 1. Tải tài liệu tại sividoc.com Viết đề tài giá sinh viên – ZALO:0973.287.149-TEAMLUANVAN.COM ĐẠI HỌC THÁI NGUYÊN TRƯỜNG ĐẠI HỌC KHOA HỌC NGUYỄN THẾ NGHĨA SỬ DỤNG HÀNG ĐIỂM ĐIỀU HÒA TRONG GIẢI TOÁN HÌNH HỌC PHẲNG LUẬN VĂN THẠC SĨ TOÁN HỌC THÁI NGUYÊN - 2016
  • 2. Tải tài liệu tại sividoc.com Viết đề tài giá sinh viên – ZALO:0973.287.149-TEAMLUANVAN.COM ĐẠI HỌC THÁI NGUYÊN TRƯỜNG ĐẠI HỌC KHOA HỌC NGUYỄN THẾ NGHĨA SỬ DỤNG HÀNG ĐIỂM ĐIỀU HÒA TRONG GIẢI TOÁN HÌNH HỌC PHẲNG LUẬN VĂN THẠC SĨ TOÁN HỌC Chuyên ngành Mã số : Phương pháp Toán sơ cấp :60460113 NGƯỜI HƯỚNG DẪN KHOA HỌC TS. NGUYỄN DANH NAM THÁI NGUYÊN - 2016
  • 3. Tải tài liệu tại sividoc.com Viết đề tài giá sinh viên – ZALO:0973.287.149-TEAMLUANVAN.COM i Mục lục Trang LỜI MỞ ĐẦU........................................................................................................... 1 Chương 1: KIẾN THỨC CHUẨN BỊ..................................................................... 2 1.1. Tỉ số đơn, tỉ số kép và hàng điểm điều hòa......................................................... 2 1.2. Chùm đường thẳng và tứ giác toàn phần ............................................................ 5 1.3. Đường tròn trực giao........................................................................................... 9 1.4. Cực và đường đối cực ......................................................................................... 9 1.5. Cách xác định cực và đường đối cực ................................................................ 16 Chương 2: SỬ DỤNG HÀNG ĐIỂM ĐIỀU HÒA TRONG GIẢI TOÁN HÌNH HỌC PHẲNG ......................................................................................................... 19 2.1. Chứng minh hàng điểm điều hòa ...................................................................... 19 2.2. Chứng minh vuông góc..................................................................................... 25 2.3. Chứng minh song song...................................................................................... 31 2.4. Chứng minh thẳng hàng .................................................................................... 33 2.5. Chứng minh đồng quy....................................................................................... 40 2.6. Chứng minh điểm cố định................................................................................. 46 2.7. Chứng minh đẳng thức...................................................................................... 55 2.8. Một số bài toán khác ......................................................................................... 64 KẾT LUẬN ............................................................................................................. 71 TÀI LIỆU THAM KHẢO ..................................................................................... 72
  • 4. Tải tài liệu tại sividoc.com Viết đề tài giá sinh viên – ZALO:0973.287.149-TEAMLUANVAN.COM 1 LỜI MỞ ĐẦU Hình học phẳng là một chủ đề hấp dẫn trong các kì thi học sinh giỏi. Một bài toán hình học phẳng luôn có thể được giải bằng nhiều cách khác nhau, trong đó áp dụng các khái niệm “hàng điểm điều hòa”, “cực và đường đối cực” được vận dụng để giải các bài toán sẽ cho lời giải một cách ngắn gọn và đẹp mắt. Đây là những công cụ mạnh và thú vị của hình học. Kiến thức về chùm đường thẳng, phép chiếu xuyên tâm, đặc biệt là chùm đường thẳng điều hòa, tứ giác toàn phần cũng được sử dụng để tìm kiếm các hàng điểm điều hòa. Khi xuất hiện các hàng điểm điều hòa, chúng ta dễ dàng sử dụng các kết quả liên quan như hệ thức Đề-các, hệ thức Niu- tơn và hệ thức Mácloranh trong giải bài toán hình học phẳng. Với hướng khai thác các hàng điểm điều hòa đơn giản và các hàng điểm điều hòa xuất hiện từ quan hệ giữa cực và đường đối cực của một điểm đối với một cặp đường thẳng cắt nhau hoặc đối với một đường tròn nào đó để giải các dạng toán hình học như: chứng minh thẳng hàng, chứng minh đồng quy, chứng minh song song, chứng minh vuông góc, chứng minh điểm cố định, chứng minh đẳng thức, bài toán quỹ tích và bài toán dựng hình. Trong luận văn này, chúng tôi quan tâm đến các bài toán có liên quan đến hàng điểm điều hòa xuất hiện trong các cuộc thi học sinh giỏi toán quốc gia và toán quốc tế. Các bài toán về hàng điểm điều hòa trong luận văn đã được lựa chọn với lời giải của có tính độc đáo và thú vị hơn so với các phương pháp thường gặp. Do vậy, có thể nói kết quả của luận văn cung cấp một công cụ mới cho học sinh trong việc tiếp cận và giải các bài toán hình học phẳng, đặc biệt là các bài toán xuất hiện trong các kì thi học sinh giỏi môn Toán. Luận văn này được thực hiện tại Trường Đại học Khoa học - Đại học Thái Nguyên và hoàn thành dưới sự hướng dẫn của TS. Nguyễn Danh Nam. Tác giả xin bày tỏ lòng biết ơn sâu sắc tới thầy hướng dẫn đã tận tình giúp đỡ trong suốt quá trình làm luận văn. Tác giả cũng xin chân thành cảm ơn các GS, PGS, TS và các thầy cô giảng viên của Trường Đại học Khoa học - Đại học Thái Nguyên đã giảng dạy và tạo mọi điều kiện thuận lợi trong quá trình tác giả học tập và nghiên cứu.
  • 5. Tải tài liệu tại sividoc.com Viết đề tài giá sinh viên – ZALO:0973.287.149-TEAMLUANVAN.COM 2 Chương 1 KIẾN THỨC CHUẨN BỊ 1.1. Tỉ số đơn, tỉ số kép và hàng điểm điều hòa 1.1.1. Độ dài đại số Trên đường thẳng d chọn véctơ đơn vị e thì ta có trục d và hướng của e là hướng của trục d. Định nghĩa 1.1. [1] Trên trục d, cho hai điểm A, B. Độ dài đại số của AB là một số có giá trị tuyệt đối bằng AB và số đó dương nếu AB cùng hướng với e và số đó âm nếu AB ngược hướng với e . Kí hiệu: AB . Các tính chất. 1) ABBA. 2) AB BC AC (A, B, C thẳng hàng). 3) A1 A2 A2 A3 ... An1 An A1 An (với mọi Ai , i1, n thẳng hàng). 1.1.2. Tỉ số đơn Định nghĩa 1.2. [1] Cho ba điểm A, B, C thẳng hàng, tỉ số đơn của chúng lấy theo thứ tự đó là tỉ số CB CA . Kí hiệu: (ABC). Định lý 1.1. [1] Cho hai điểm A, B và một số thực k 1 thì tồn tại duy nhất điểm C sao cho (ABC) = k. Chứng minh. Ta có (ABC) = k CB CA  k CA kCB CA kCA AB  CA kAB AC CA k AC k AB AC k k 1 AB (k1) Suy ra, tồn tại duy nhất điểm C sao cho (ABC) = k.
  • 6. Tải tài liệu tại sividoc.com Viết đề tài giá sinh viên – ZALO:0973.287.149-TEAMLUANVAN.COM 3 1.1.3. Tỉ số kép Định nghĩa 1.3. [1] Cho bốn điểm A, B, C, D thẳng hàng, tỉ số kép của chúng lấy theo thứ tự đó là tỉ số CB CA : DB DA . Kí hiệu: (ABCD). VậyABCD CB CA : DB DA   ABD ABC . Các tính chất. 1) Tỉ số kép của bốn điểm là không thay đổi trong các trường hợp sau: + Nếu hoán vị cặp điểm đầu với cặp điểm cuối: (ABCD) = (CDAB). + Nếu đồng thời hoán vị hai điểm đầu và hai điểm cuối: (ABCD) = (BADC) + Nếu viết chúng theo thứ tự ngược lại: (ABCD) = (DCBA). 2) Tỉ số kép của bốn điểm thay đổi trong các trường hợp: + Nếu hoán vị hai điểm đầu hoặc hai điểm cuối thì tỉ số kép của bốn điểm trở thành số đảo ngược của nó: (BACD) = (ABDC) 1  ABCD + Nếu hoán vị hai điểm ở giữa hoặc hai điểm ở đầu và cuối thì tỉ số kép của bốn điểm trở thành phần bù của 1:ABCD 1 ACBD 1DBCA. 1.1.4. Hàng điểm điều hoà Định nghĩa 1.4. [1] Nếu (ABCD) = -1 thì ta nói bốn điểm A, B, C, D lập thành một hàng điểm điều hoà hay A, B chia điều hoà C, D hay A, B liên hợp điều hoà đối với C, D. Các tính chất. Cho bốn điểm A, B, C, D thẳng hàng, ta có: 1) Hệ thức Đề-các:ABCD1 AB 2  AC 1  AD 1 . 2) Hệ thức Niu-tơn:ABCD1 IA2  IC .ID (trong đó I là trung điểm của đoạn thẳng AB).
  • 7. Tải tài liệu tại sividoc.com Viết đề tài giá sinh viên – ZALO:0973.287.149-TEAMLUANVAN.COM 4 3) Hệ thức Mácloranh: AC. AD AB.AJ (trong đó J là trung điểm của đoạn thẳng CD). Chứng minh. Trên đường thẳng AB, chọn O làm gốc toạ độ. Đặt OA = a, OB = b, OC = c, OD = d, ta có: CA OA OC = a – c ; CB OB OC = b - c DA OD OA = d – a ; DB OD OB = d - b Ta có ABCD1CA :DA a - c a - d b - c b - d CB DB  (a - c)(b - d) - (a - d)(b - c)  2(ab + cd) (a + b)(c + d) (1) + Chọn OA thì: OA = a = 0, AC = OC = c, AB = OB = b, AD = OD = d. Từ (1) ta có 2cd = bc + bd b 2  d 1  1 c AB 2  AC 1  AD 1 . + Chọn O I thì ta có OAOB hay a = - b. Từ (1) ta có 2(- a2 + cd) = 0 a2 = cd IA2  IC.ID . Chứng minh tương tự đối với hệ thức Mácloranh. Định lý 1.2. [1] Nếu AD, AE lần lượt là phân giác trong, phân giác ngoài của tam giác ABC (D, E thuộc đường thẳng BC) thì (BCDE) = - 1. A B D C E Hình 1.1 Định lý 1.3. [1] Cho tam giác ABC và điểm O không thuộc các đường thẳng chứa ba cạnh của tam giác. Các đường thẳng AO, BO, CO theo thứ tự cắt BC, CA, AB tại M, N, P và BC cắt NP tại Q. Khi đó ta có (BCMQ) = - 1.
  • 8. Tải tài liệu tại sividoc.com Viết đề tài giá sinh viên – ZALO:0973.287.149-TEAMLUANVAN.COM 5 A P N O B M C Q Hình 1.2 Định lý 1.4. [1] Từ điểm S nằm ngoài đường tròn (O) kẻ các tiếp tuyến SA, SB tới (O) (A, B là các tiếp điểm ). Một đường thẳng đi qua S và cắt (O) lần lượt tại M, N, và AB cắt MN tại I. Khi đó (SIMN) = - 1. Hình 1.3 1.2. Chùm đường thẳng và tứ giác toàn phần 1.2.1. Chùm đường thẳng Định nghĩa 1.5. [1] Trong mặt phẳng, cho tập hợp các đường thẳng đồng quy tại điểm S thì chúng lập nên một chùm đường thẳng và S được gọi là tâm của chùm. Tập hợp các đường thẳng nằm trong mặt phẳng và song song với nhau lập nên một chùm đường thẳng và có tâm tại vô tận. Định lý 1.5. [1] Một chùm bốn đường thẳng cắt một đường thẳng theo hàng điểm có tỉ số kép không thay đổi. Chứng minh. * Trường hợp chùm đồng quy tại điểm S: Gọi l là đường thẳng cắt các đường thẳng a, b, c, d lần lượt tại A, B, C, D và l’ là đường thẳng cắt các đường thẳng a, b,
  • 9. Tải tài liệu tại sividoc.com Viết đề tài giá sinh viên – ZALO:0973.287.149-TEAMLUANVAN.COM 6 c, d lần lượt tại A’, B’, C’, D’. Ta cần chứng minh đẳng thức (ABCD) = (A’B’C’D’) (Hình 1.4). S N’ l’ M’ N B’ M C’ D’ A’ A B C D l a b c d Hình 1.4 Qua điểm B kẻ đường thẳng song song với đường thẳng a và cắt đường thẳng c tại N, cắt đường thẳng d tại M. Ta có: CA SA và DA SA   CB MB DB NB Từ đó suy ra: CA DA SA SA NB (1) ABCD :  :  CB AB MB NB MB Tương tự, từ điểm B’ kẻ đường thẳng song song với đường thẳng a và cắt đường thẳng c, d lần lượt tại M’, N’. Ta có A' B 'C ' D ' N 'B' (2) M 'B' Mặt khác, ta có: NB N 'B' (3)  MB M 'B' Từ (1), (2) và (3) ta có (ABCD) = (A’B’C’D’). * Trường hợp chùm song song: Nếu a // b // c // d thì ta luôn có đẳng thức (ABCD) = (A’B’C’D’).
  • 10. Tải tài liệu tại sividoc.com Viết đề tài giá sinh viên – ZALO:0973.287.149-TEAMLUANVAN.COM 7 Định nghĩa 1.6. [1] Trong mặt phẳng cho chùm bốn đường thẳng a, b, c, d. Một đường thẳng l bất kì cắt chùm đó tại A, B, C, D thì (ABCD) được gọi là tỉ số kép của chùm bốn đường thẳng a, b, c, d. Kí hiệu: (abcd) = (ABCD). Nếu chùm đồng quy tại S thì ta kí hiệu: S(abcd) = (ABCD). S l Nếu (abcd) = - 1 thì ta có một chùm điều N hoà, hay a, b liên hợp điều hoà với c, d hay a, b B chia điều hoà c, d. Định lý 1.6. [1] Trong mặt phẳng cho M d chùm bốn đường thẳng đồng quy. Điều kiện cần a c b và đủ để chùm đó lập thành một chùm điều hoà Hình 1.5 là: Một đường thẳng bất kì song song với một trong bốn đường thẳng đó bị ba đường thẳng còn lại chia thành hai đoạn thẳng bằng nhau. Chứng minh. Kẻ đường thẳng l song song với a và cắt b, c, d lần lượt tại M, B, N. Theo định lý trên, ta có: ABCD(abcd) = MB NB và (abcd) = -1  MB NB 1 NBMB   B là trung điểm của đoạn thẳng MN hay MB = NB (Hình 1.5). Hệ quả 1. Trong một chùm điều hoà nếu có hai đường liên hợp vuông góc với nhau thì hai đường đó là các đường phân giác của các góc tạo bởi hai đường còn lại (Hình 1.6a). Hệ quả 2. Hai đường phân giác của hai góc kề bù chia điều hoà hai cạnh của góc đó (Hình 1.6b). Chùm đường thẳng gồm hai cạnh của một góc và hai đường phân giác của góc đó được gọi là chùm phân giác.
  • 11. Tải tài liệu tại sividoc.com Viết đề tài giá sinh viên – ZALO:0973.287.149-TEAMLUANVAN.COM 8 S S b C B D A c a d a) b) Hình 1.6 Trong mặt phẳng, tập hợp các đường thẳng đồng quy tại một điểm S, được gọi là một chùm đường thẳng tâm S. Cho chùm bốn đường thẳng a, b, c, d. Một đường thẳng bất kỳ cắt a, b, c, d thứ tự tại A, B, C, D. Khi đó (ABCD) không phụ thuộc vào vị trí của .Giá trị không đổi của tỉ số kép (ABCD) được gọi là tỉ số kép của chùm bốn đường thẳng a, b, c, d, ký hiệu (abcd) hay S(abcd) khi cần quan tâm đến tâm của chùm. 1.2.2. Tứ giác toàn phần Định nghĩa 1.7. [1] Trong mặt phẳng, cho bốn đường thẳng cắt nhau từng đôi một và không có ba đường nào đồng quy thì chúng lập thành một tứ giác toàn phần. - Các đường thẳng là các cạnh (có bốn cạnh). - Giao của hai cạnh là đỉnh (có sáu đỉnh). - Hai đỉnh không thuộc một cạnh là hai đỉnh đối diện (có ba cặp đỉnh đối diện). - Đường thẳng nối hai đỉnh đối diện là đường chéo (có ba đường chéo). Cho tứ giác toàn phần ABCA’B’C’. Khi đó, ta có cặp đỉnh đối diện là (A, A’), (B, B’), (C, C’); ba đường chéo là AA’, BB’, CC’. Định lý 1.7. [1] Trong một tứ giác toàn phần, cặp đỉnh đối diện chia điều hoà hai giao điểm của đường chéo nối cặp đỉnh đối diện đó với hai đường chéo còn lại. Chứng minh. Gọi P = AA’BB’, Q = AA’CC’, R = BB’CC’. Ta chứng minh (AA’PQ) = (BB’PR) = (CC’QR) = - 1. Ta có: B(AA’PQ) = B’(AA’PQ) = B’(CC’RQ) = B(CC’RQ) = B(A’APQ).  (AA’PQ) = (A’APQ)AA' PQ 1 AA' PQ 2 1. AA' PQ Nếu (AA’PQ) = 1 thì ta có (AA’P) = (AA’Q) hay PQ (vô lý).
  • 12. Tải tài liệu tại sividoc.com Viết đề tài giá sinh viên – ZALO:0973.287.149-TEAMLUANVAN.COM 9 Vậy (AA’PQ) = - 1. Các tỉ số kép khác được chứng minh một cách tương tự. A B P B’ A’ C Q C’ R Hình 1.7 1.3. Đường tròn trực giao Định nghĩa 1.8. [3] Hai đường tròn gọi là trực giao với nhau tại một điểm chung của chúng nếu tại điểm đó hai tiếp tuyến của hai đường tròn vuông góc với nhau. Từ định nghĩa, ta dễ dàng suy ra được các kết quả sau: Định lý 1.8. [3] Điều kiện cần và đủ để hai đường tròn trực giao với nhau là bình phương khoảng cách giữa hai tâm bằng tổng bình phương các bán kính của chúng. Định lý 1.9. [3] Điều kiện cần và đủ để hai đường tròn trực giao với nhau là phương tích của tâm của một trong hai đường tròn đó đối với đường tròn thứ hai bằng bình phương bán kính của đường tròn thứ nhất. Định lý 1.10. [3] Điều kiện cần và đủ để hai đường tròn trực giao với nhau là có một đường kính nào đó của một trong hai đường tròn bị đường tròn kia chia điều hoà. Định nghĩa 1.9. [3] Người ta gọi chùm đường tròn là một tập hợp các đường tròn kể từng đôi một, nhận một đường thẳng duy nhất làm trục đẳng phương. Đường thẳng đó gọi là trục đẳng phương của chùm. Từ định nghĩa trên ta thấy rằng, tâm các đường tròn của một chùm phải nằm trên một đường thẳng gọi là đường chứa tâm của chùm và đường thẳng này vuông góc với trục đẳng phương của chùm.
  • 13. Tải tài liệu tại sividoc.com Viết đề tài giá sinh viên – ZALO:0973.287.149-TEAMLUANVAN.COM 10 Từ định nghĩa của chùm đường tròn, ta suy ra hai định lý sau đây: Định lý 1.11. [3] Điều kiện cần và đủ để một tập hợp các đường tròn lập thành một chùm là có hai điểm mà mỗi điểm đều có cùng phương tích đối với tất cả các đường tròn của tập hợp đó. Trục đẳng phương của chùm là đường thẳng nối hai điểm nói trên. Định lý 1.12. [3] Điều kiện cần và đủ để một tập hợp các đường tròn có tâm thẳng hàng lập thành một chùm là có một điểm có cùng phương tích đối với tất cả các đường tròn của tập hợp đó. Trục đẳng phương của chùm là đường thẳng đi qua điểm nói trên và vuông góc với đường chứa tâm. 1.4. Cực và đường đối cực 1.4.1. Đường đối cực của một điểm đối với hai đường thẳng cắt nhau Định nghĩa 1.10. [3] Hai điểm M và N gọi là liên hợp với nhau đối với hai đường thẳng đồng quy Ox, Oy nếu đường thẳng MN cắt hai đường thẳng đó tại hai điểm A, B sao cho (MNAB) = -1. Nếu (MNAB) = -1 thì ta cũng suy ra (ABMN) = -1 và khi đó hai điểm A và B cũng liên hợp với nhau đối với hai đường thẳng đồng quy OM, ON. Bài toán. Cho một điểm M không thuộc hai đường thẳng Ox, Oy. Hãy tìm tập hợp các điểm N liên hợp với M đối với hai đường thẳng đã cho. Lời giải. Qua M ta kẻ một đường thẳng lần lượt cắt Ox, Oy tại A, B. Ta lấy trên đường thẳng đó một điểm N sao cho (MNAB) = -1 (Hình 1.8). P O N’ B’ A’ N1 M B A N x y Q z Hình 1.8
  • 14. Tải tài liệu tại sividoc.com Viết đề tài giá sinh viên – ZALO:0973.287.149-TEAMLUANVAN.COM 11 Nếu kẻ đường thẳng Oz đi qua O và N thì ta có chùm (OM, Oz, Ox, Oy) là một chùm điều hoà. Do đó, mọi điểm của đường thẳng Oz (trừ hai điểm P và Q) đều liên hợp với điểm M đối với hai đường thẳng đồng quy Ox, Oy (do hai điểm P và Q thuộc Oz mà MP // Ox và MQ // Oy ta phải loại ra vì lúc đó các đường thẳng MP và MQ đều không cắt cả hai đường thẳng Ox và Oy). Ngược lại, nếu N1 là một điểm không thuộc đường thẳng Oz nói trên thì không liên hợp với M vì khi đó nếu đường thẳng MN1 cắt Ox, Oy, Oz lần lượt tại A’, B’, N’ thì ta có: (MN’A’B’) = -1 còn (MN1A’B’) (MN’A’B’) nên (MN1A’B’) -1. Do đó, điểm N1 không liên hợp với M đối với hai đường thẳng Ox và Oy. Vậy tập hợp các điểm N liên hợp với điểm M đối với hai đường thẳng Ox, Oy là đường thẳng Oz loại trừ hai điểm P, Q nói trên. Định nghĩa 1.11. [3] Đường thẳng Oz trong bài toán trên gọi là đường đối cực của điểm M đối với hai đường thẳng Ox, Oy. Điểm M gọi là cực của đường thẳng Oz đối với hai đường thẳng đó. Nhận xét. Muốn dựng đường đối cực của một điểm M đối với hai đường thẳng Ox, Oy cho trước, dựa vào tính chất của hình tứ giác toàn phần ta tìm hai điểm P và Q phân biệt đều cùng liên hợp với M đối với Ox, Oy nói trên. Ta có PQ là đường đối cực của điểm M đối với Ox, Oy và PQ luôn đi qua điểm O (Hình 1.9a). O M P A O N M Q B x y a) b) Hình 1.9
  • 15. Tải tài liệu tại sividoc.com Viết đề tài giá sinh viên – ZALO:0973.287.149-TEAMLUANVAN.COM 12 1.4.2. Đường đối cực của một điểm đối với một đường tròn Định nghĩa 1.12. [3] Hai điểm M và N gọi là liên hợp với nhau đối với đường tròn (O), nếu đường tròn đường kính MN trực giao với đường tròn (O) (Hình 1.9b). Nếu đường thẳng MN cắt đường tròn (O) tại hai điểm A và B thì điều kiện cần và đủ để M và N liên hợp với nhau đối với đường tròn (O) cho trước là tỉ số kép (MNAB) = -1. Hai điểm M, N có thể liên hợp với nhau đối với đường tròn (O) mà đường thẳng MN không cắt đường tròn này. Bài toán. Cho đường tròn (O) và một điểm M không trùng với tâm O của đường tròn đó. Hãy tìm tập hợp những điểm N liên hợp của M đối với đường tròn (O) đã cho. Lời giải. Nếu N là điểm liên hợp của M đối với đường tròn (O) thì theo định nghĩa, đường tròn đường kính MN trực giao với đường tròn (O). Khi đó, đường kính AB đi qua M của đường tròn (O) bị đường tròn đường kính MN chia điều hoà. Gọi H là giao điểm thứ hai của đường tròn đường kính MN với đường thẳng AB. Ta có (ABMH) = -1 (Hình 1.10). Trong hàng điểm điều hoà A, B, M và H, điểm H hoàn toàn được xác định vì ba điểm A, B, M đã được xác định. Mặt khác, do MN là đường kính nên MH HN. Nói cách khác, điểm N nằm trên đường thẳng m vuông góc với đường thẳng MO tại H. N Ngược lại, nếu N’ là điểm bất kì của đường thẳng m thì đường tròn đường kính MN’ đi qua H và do (ABMH) = -1 nên đường M A HO B tròn đường kính MN’ trực giao với đường tròn (O). Vậy điểm N’ liên hợp với M đối với đường tròn (O). Hình 1.10 Vậy tập hợp điểm N liên hợp với điểm M đối với một đường tròn (O) cho trước là một đường thẳng m vuông góc với đường thẳng MO tại H với (MHAB) = -1, trong đó A, B là giao điểm của đường thẳng MO với đường tròn tâm O.
  • 16. Tải tài liệu tại sividoc.com Viết đề tài giá sinh viên – ZALO:0973.287.149-TEAMLUANVAN.COM 13 Định nghĩa 1.13. [3] Đường thẳng m trong bài toán trên gọi là đường đối cực của điểm M đối với đường tròn (O). Điểm M gọi là cực của đường thẳng m đối với đường tròn (O) nói trên. Như vậy, mỗi điểm M không trùng với điểm O của đường tròn tâm O có một đường đối cực xác định và ngược lại, mỗi đường thẳng không đi qua O có một điểm cực xác định đối với một đường tròn tâm O cho trước. Vì (ABMH) = -1 nên đường đối cực m của điểm M đối với đường tròn (O) sẽ cắt, không cắt hay tiếp xúc với đường tròn tâm O (Hình 1.11a,b,c). Muốn dựng đường đối cực của một điểm M đối với một đường tròn tâm O cho trước, ta vẽ qua M hai cát tuyến MAB, MCD (Hình 1.12). Gọi P và Q lần lượt là các điểm liên hợp với M nghĩa là (ABMP) = -1 và (CDMQ) = -1. m m m I R M AHOB H AMOBHMO A B K S a) b) c) Hình 1.11 Ta suy ra PQ là đường đối cực của điểm M. Ta có thể dựa vào tính chất của hình tứ giác toàn phần để tìm các điểm P và Q liên hợp với M đối với A, B và C, D. Đặc biệt, khi các cát tuyến đó trở H thành tiếp tuyến thì ba điểm P, A, B trùng nhau và ba điểm C, Q, D cũng trùng nhau. Do đó, muốn dựng đường đối cực của một điểm M ta thường làm như sau: M B A P O C Q D Hình 1.12
  • 17. Tải tài liệu tại sividoc.com Viết đề tài giá sinh viên – ZALO:0973.287.149-TEAMLUANVAN.COM 14 - Nếu điểm M nằm ngoài đường tròn (O) thì từ M ta vẽ hai đường tiếp tuyến MI, MK với đường tròn, trong đó I và K là hai tiếp điểm. Khi đó, đường thẳng IK là đường đối cực của điểm M cho trước (Hình 1.11a). - Nếu điểm M nằm trong đường tròn thì ta vẽ đường thẳng vuông góc với MO tại M. Đường thẳng này cắt đường tròn tại hai điểm R và S (Hình 1.11b). Các tiếp tuyến của đường tròn tại R và S cắt nhau tại H. Đường thẳng m vuông góc với đường thẳng MO tại H là đường đối cực của điểm M cho trước. - Nếu điểm M nằm trên đường tròn thì tiếp tuyến tại M của đường tròn chính là đường đối cực của điểm M cho trước (Hình 1.11c). 1.4.3. Các tính chất của cực và đường đối cực đối với một đường tròn 1) Đối với một đường tròn cho trước, nếu đường đối cực của điểm A đi qua điểm B thì đường đối cực của điểm B đi qua điểm A. Chứng minh. Nếu điểm B nằm trên đường đối cực a của điểm A thì A và B là hai điểm liên hợp đối với đường tròn cho trước. Mặt khác ta biết rằng, tập hợp các điểm liên hợp của điểm B là đường đối cực b của điểm B đó (Hình 1.13). Vậy điểm A B phải nằm trên đường đối cực b của điểm B (vai trò của A và B là bình đẳng). Ta có: B a A b. Định nghĩa 1.14. [3] Hai đường b A thẳng a, b được gọi là liên hợp với nhau đối với một đường tròn cho trước nếu đường này đi qua cực của đường kia. a 2) Đối với một đường tròn cho trước, các đường đối cực của các điểm thẳng hàng Hình 1.13 thì đồng quy và các cực của các đường thẳng đồng quy thì thẳng hàng. Chứng minh. Theo tính chất 1, giả sử các điểm A1, A2…, An nằm trên đường thẳng b nghĩa là các điểm Ai b với i = 1, 2…, n thì điểm B thuộc các đường thẳng b và ai là các đường đối cực của các điểm Ai. Vậy các đường đối cực của các điểm Ai đều đồng quy tại B. Phần còn lại chứng minh tương tự.
  • 18. Tải tài liệu tại sividoc.com Viết đề tài giá sinh viên – ZALO:0973.287.149-TEAMLUANVAN.COM 15 1.4.4. Phép đối cực Trên mặt phẳng cho một đường tròn cơ sở (C). Giả sử có một hình H gồm các điểm và các đường thẳng. Với mỗi điểm của hình H đều có các đường đối cực của nó đối với đường tròn (C), với mỗi đường thẳng của hình H có các điểm là cực của nó. Hình H' là tập hợp các đường thẳng (gồm các đường đối cực của các điểm thuộc hình H) và các điểm (gồm các cực của các đường thẳng thuộc hình H). Khi đó, ta nói có một phép đối cực với đường tròn cơ sở (C) biến hình H thành hình H'. Rõ ràng muốn chứng minh tính thẳng hàng của các điểm trên hình H ta chỉ việc chứng minh tính đồng quy của các đường thẳng tương ứng trên hình H'. Ví dụ 1.1. [3] (Định lý Bri-ăng-xông) Ba đường thẳng nối các cặp đỉnh đối diện của một lục giác ngoại tiếp một đường tròn đồng quy tại một điểm. Lời giải. Giả sử ABCDEF là lục giác ngoại tiếp đường tròn (O). Gọi M, N, P, Q, K, I lần lượt là các tiếp điểm của các cạnh AB, BC, CD, DE, EF, FA với đường tròn (C). Khi đó, theo định lý Pát-xcan: B MN KQ= M N QP IM= , , thẳng hàng. A C PN IK= P Hiển nhiên,là cực của BE,là cực của I AD, là cực của CF. Vì , , thẳng hàng nên BE, F D Q K AD, CF đồng quy tại một điểm. Ta có phép đối cực E biến ba điểm , , thành ba đường thẳng BE, AD, Hình 1.14 CF (Hình 1.14). Định lý 1.13. [3] Phép đối cực bảo tồn tỉ số kép, nghĩa là qua phép đối cực, một chùm bốn đường thẳng (đồng quy) biến thành bốn điểm và tỉ số kép của bốn điểm này bằng tỉ số kép của bốn đường thẳng đó. Hệ quả. Phép đối cực biến một chùm đường thẳng điều hoà thành một hàng điểm điều hoà và ngược lại.
  • 19. Tải tài liệu tại sividoc.com Viết đề tài giá sinh viên – ZALO:0973.287.149-TEAMLUANVAN.COM 16 Như vậy, phép đối cực là một công cụ tương đối hiệu quả trong việc chuyển đổi hai dạng bài toán chứng minh đồng quy và chứng minh thẳng hàng, chuyển từ chùm đường thẳng điều hòa sang hàng điểm điều hòa và ngược lại. 1.5. Cách xác định cực và đường đối cực * Trường hợp 1: Khi cực S ở ngoài đường tròn (O). Ta có 2 cách dựng sau: - Cách 1: Từ S kẻ tới (O) hai tiếp tuyến SA, SB (A, B là tiếp điểm). Khi đó đường đối cực của S đối với (O) là AB. A S O . B Hình 1.15 - Cách 2: Từ S kẻ tới (O) hai cát tuyến SAB, SCD. Giả sử AD cắt BC ở F, AC cắt BD ở E. Khi đó đường đối cực của điểm S đối với (O) là đường thẳng EF. S F B A E D C Hình 1.16 * Trường hợp 2: Khi điểm S nằm trong đường tròn (O). Ta có 2 cách dựng sau đây:
  • 20. Tải tài liệu tại sividoc.com Viết đề tài giá sinh viên – ZALO:0973.287.149-TEAMLUANVAN.COM 17 - Cách 1: Qua điểm S dựng đường vuông góc với OS, đường này cắt (O) tại hai điểm A, B. Tiếp tuyến của (O) tại A, B cắt nhau ở điểm P. Khi đó đường đối cực của điểm S đối với đường tròn (O) là đường thẳng qua P vuông góc với OS. A P S O B Hình 1.17 - Cách 2: Qua điểm S dựng hai dây cung AB và CD. Giả sử AD cắt BC ở E, AC cắt BD ở F. Khi đó đường đối cực của điểm S đối với (O) là EF. E C A S .O F D B Hình 1.18 * Trường hợp 3: Điểm S nằm trên đường tròn (O). Khi đó, tiếp tuyến của (O) tại S chính là đường đối cực của S đối với (O).
  • 21. Tải tài liệu tại sividoc.com Viết đề tài giá sinh viên – ZALO:0973.287.149-TEAMLUANVAN.COM 18 O. S Hình 1.19 Chương 1 của luận văn trình bày các khái niệm cơ bản như hàng điểm điều hòa, chùm đường thẳng, chùm đường thẳng điều hòa và tứ giác toàn phần. Đây là những nội dung có liên quan đến hàng điểm điều hòa. Chúng ta có thể chứng minh hàng điểm điều hòa dựa trên các tính chất của chùm đường thẳng điều hòa và tứ giác toàn phần. Kiến thức về đường tròn trực giao, cực và đường đối cực đối với hai đường thẳng đồng quy và đối với đường tròn cũng như cách dựng đường đối cực của một điểm cho trước. Với cực và đường đối cực ta có thể đưa ra cách nhìn xuyên suốt, nhất quán đối với một số dạng toán như chứng minh quan hệ vuông góc, chứng minh các điểm thẳng hàng, chứng minh quan hệ đồng quy,... Các bài toán về cực và đường đối cực thường gặp ở bậc trung học phổ thông là cực và đường đối cực của một điểm đối với đường tròn hoặc đối với cặp đường thẳng cắt nhau. Đặc biệt, phép đối cực được trình bày cho chúng ta một công cụ trong việc chuyển đổi bài toán chứng minh thẳng hàng và bài toán chứng minh đồng quy. Trong chương 2 luận văn sẽ khai thác một số lớp bài toán sử dụng đến khái niệm cực và đường đối cực để chứng minh các điểm thẳng hàng, chứng minh các đường thẳng đồng quy và giải bài toán tìm điểm cố định.
  • 22. Tải tài liệu tại sividoc.com Viết đề tài giá sinh viên – ZALO:0973.287.149-TEAMLUANVAN.COM 19 Chương 2 SỬ DỤNG HÀNG ĐIỂM ĐIỀU HÒA TRONG GIẢI TOÁN HÌNH HỌC PHẲNG Để có thể sử dụng hàng điểm điều hòa trong giải toán, chúng ta cần phải nhận ra các hàng điểm điều hòa trong bài toán, đặc biệt là vận dụng linh hoạt các tính chất trong tứ giác toàn phần, tứ giác điều hòa, chùm phân giác,… Dưới đây là một số minh họa cách tìm các hàng điểm điều hòa trong một bài toán cụ thể. 2.1. Chứng minh hàng điểm điều hòa Để chứng minh bốn điểm lập thành hàng điểm điều hòa chúng ta có thể sử dụng định nghĩa, nghĩa là chứng minh tỉ số kép của bốn điểm bằng -1. Các định lý thường được áp dụng trong giải dạng toán này là định lý Xêva, định lý Mênêlauýt, hệ thức Niu-tơn và hệ thức Đề-các về hàng điểm điều hòa. Ví dụ 2.1. [4] Cho tam giác ABC. Lấy E trên BC, điểm F trên AC và điểm K trên AB sao cho AE, BF, CK đồng quy tại một điểm. Gọi T là giao điểm của FK với BC. Chứng minh rằng (TEBC) = -1. Giải. Bài toán có giả thiết về các đường thẳng đồng quy trong tam giác, vì vậy định lý Xêva, định lý Mênêlauýt được sử dụng trong bài toán này. Thật vậy, trongABC, áp dụng định lý Xêva với ba đường đồng quy AE, BF, CK ta có: EB . FC . KA 1 (1) EC FA KB Mặt khác, áp định lý Mênêlauýt với ba điểm thẳng hàng T, K, F ta lại có: TC . KB . FA 1 (2) TB KA FC Hình 2.1
  • 23. Tải tài liệu tại sividoc.com Viết đề tài giá sinh viên – ZALO:0973.287.149-TEAMLUANVAN.COM 20 Nhân (1) và (2) vế theo vế suy ra: TC TB  EC EB hay (TEBC) = -1. * Nhận xét: Nếu gọi I là điểm đồng quy của AE, BF, CK thì AIBC là một tứ giác toàn phần với các đường chéo AI, FK và BC mà lời giải là một trong những cách chứng minh cho định lý rất đẹp về hình tứ giác toàn phần: “Trong một hình tứ giác toàn phần, một đường chéo bị hai đường chéo còn lại chia điều hòa”. Bài toán đơn giản này cho ta sử dụng tính chất một hình tứ giác toàn phần hay hàng điểm điều hòa cho một tam giác có ba đường thẳng đồng quy. Ví dụ 2.2. [4] Cho tứ giác ABCD ngoại tiếp đường tròn tâm (O). Gọi M, N, P, Q lần lượt là các tiếp điểm trên các cạnh AB, BC, CD, DA với đường tròn. Gọi K là giao điểm của đường thẳng MQ với NP và I là giao điểm của đường thẳng MP với QN. Chứng minh rằng (DBIK) = -1. Giải. Bài toán có giả thiết về các tiếp điểm của đường tròn với các cạnh của tam giác, vì vậy định lý Mênêlauýt được sử dụng, từ đó xuất hiện các tỉ số giữa các đoạn thẳng và có thể được sử dụng để chứng minh hàng điểm điều hòa theo định nghĩa. Áp dụng định lý Mênêlauýt cho tam giác ABD với 3 điểm thẳng hàng K, M, KB QD MA 1 KB  MB Q ta có: . . hay (vì QA = MA) (1) KD QA MB KD QD Mặt khác, ta có thể chứng minh được: MB  IB (2) QD ID A K M Q B O . I N D P C Hình 2.2
  • 24. Tải tài liệu tại sividoc.com Viết đề tài giá sinh viên – ZALO:0973.287.149-TEAMLUANVAN.COM 21 Từ (1) và (2) suy ra KD KB  ID IB (Hình 2.2). Vì I nằm trong đoạn thẳng BD và K nằm ngoài đoạn thẳng BD nên ta suy ra KD KB  ID IB . Vậy (DBIK) = -1. Ví dụ 2.3. [2] ChoABC không cân tại A, phân giác trong AD, đường cao AH. Gọi E, F là hình chiếu của D trên AB, AC. Kẻ đường thẳng EF cắt đường thẳng BC tại điểm L. Chứng minh rằng (HLBC) = -1. Giải. Tương tự ví dụ 2.2, bài toán này sử dụng định lý Mênêlauýt như sau: A E F B D H C L Hình 2.3 XétABC, ta cần chứng minh: FC  EA  HB 1. FA EB HC Các tứ giác EAHD, FADH nội tiếp đường tròn BE.BA BH.BD (1)  CD.CH CF.CA (2) Từ (1) và (2) suy ra BH . BD BE . BA  BH  BE  HB .CF 1 (3) CH CH CD CF CA CF HC BE Mà AD là phân giácBAC nên AE = AFAE 1 (4) AF Từ (3) và (4) ta có HB . CF . AE  1HB . FC . EA 1 (5) HC BE AF HC FA EB FC EA LB XétABC với cát tuyến EFL, ta có . . 1 (6) FA EB LC (áp dụng định lý Mênêlauýt trong mặt phẳng).
  • 25. Tải tài liệu tại sividoc.com Viết đề tài giá sinh viên – ZALO:0973.287.149-TEAMLUANVAN.COM 22 Từ5 và6 ta có HC HB . LC LB 1 hay (HLBC) = -1. Ví dụ 2.4. [4] Cho điểm A nằm ngoài đường tròn (O). Từ điểm A kẻ hai tiếp tuyến AB, AC đến đường tròn (O) (với B, C là hai tiếp điểm). Đường thẳng AO cắt đường tròn (O) tại E, F và cắt cạnh BC tại điểm K. Chứng minh rằng (AKEF) = -1. Giải. Trong bài toán này, chúng ta nhận thấy xuất hiện các tam giác vuông. Do đó, ta có thể sử dụng hệ thức lượng trong tam giác vuông. Các hệ thức này có quan hệ với hệ thức Niu-tơn về hàng điểm điều hòa. Đó cũng là một ý tưởng để chứng minh hàng điểm điều hòa. Ta có OB2 = OK.OA (hệ thức lượng tam giác vuông) (1) Mặt khác ta lại có: OB2 = OE2 = OF2 (2) Từ (1) và (2) ta suy ra: OE2 = OF2 = OK.OA. Từ đó suy ra điều phải chứng minh (Hình 2.4) . B F O K E A C Hình 2.4 Ví dụ 2.5. [2] Cho hình vuông và một đường tròn tâm O nội tiếp hình vuông. Một tiếp tuyến bất kỳ của đường tròn cắt các cặp cạnh đối của hình vuông tại A, B và C, D. Chứng minh rằng (ABCD) = - 1. Giải. Bài toán xuất hiện các đường phân giác của một góc. Điều này gợi ý cho việc sử dụng các chùm phân giác trong chứng minh hàng điểm điều hòa.
  • 26. Tải tài liệu tại sividoc.com Viết đề tài giá sinh viên – ZALO:0973.287.149-TEAMLUANVAN.COM 23 * Cách 1: Ta có OD là phân giác củaGOF, OC là phân giác củaFOI mà ta lại cóGOF +FOI = 180o nên OD OC (1) Ta có OA là phân giác củaEOF, OD là phân giác củaFOG. Từ đó suy ra rằngAOD =AOF +FOD = 1 2EOF + 1 2FOG = 450 . Từ điều trên và (1) suy ra OA là phân giác củaCOD. Tương tự, ta chứng minh được OA OB (Hình 2.5). Như vậy: OA, OB, OC, OD là một chùm đường thẳng điều hòa. Từ đó suy ra (ABCD) = - 1. C M A E F D G O B Q H N I P Hình 2.5 * Cách 2: Xét chùm đường thẳng FE, FI, FH, FG. Ta có: số đo cung EI = số đo cung IH FI là phân giácEFH số đo cung IH = số đo cung HG FH là phân giácIFG Suy ra FE, FI, FH, FG là chùm đường thẳng điều hòa. Mặt khác: FE OA, FI OC, FH OB, FG OD. Từ đó suy ra các đường thẳng OA, OB, OC, OD cũng là chùm phân giác nên nó là chùm đường thẳng điều hòa, suy ra (ABCD) = - 1.
  • 27. Tải tài liệu tại sividoc.com Viết đề tài giá sinh viên – ZALO:0973.287.149-TEAMLUANVAN.COM 24 Ví dụ 2.6. [2] Cho đường tròn tâm O, điểm M nằm ngoài đường tròn. Gọi MA, MB là hai tiếp tuyến với đường tròn (A, B là các tiếp điểm) và cát truyến MCD với đường tròn (C, D thuộc đường tròn tâm O). Chứng minh rằng AM, AB, AC, AD là chùm đường thẳng điều hòa. Giải. GọiI AB MO , OM cắt (O) tại C’, D’ (Hình 2.6). Ta có MO AB và cung AC’ = cung BC’. GọiH ABCD . Ta có (NIC’D’) = - 1 D(NIC’D’) = - 1. Từ đó suy ra DC’ là phân giác củaMDI (vì C’D DD’). Vậy cung CC’ = cung C’K (với K DI(O) ). Như vậy, các điểm C, A đối xứng với K, B qua đường thẳng MO. Vì thếCIA =KIB, màKIB =AID (hai góc đối đỉnh) nênCIA =AID hay IA là phân giác củaCID. Mặt khác IM IA. Vậy IM, IA, IC, ID là chùm phân giác nên nó là chùm đường thẳng điều hòa. Từ đó suy ra (MHCD) = -1 hay AM, AB, AC, AD là chùm đường thẳng điều hòa. A H D C I . O M C’ D’ K B Hình 2.6 * Nhận xét: Từ bài toán trên, ta có kết quả sau đây: “Với mỗi cát tuyến MCD cắt đường nối hai tiếp điểm A, B của đường tròn tại điểm H thì ta có hàng điểm điều hòa: (MHCD) = - 1”. Ví dụ 2.7. [2] ChoABCnội tiếp đường tròn (O), tiếp tuyến tại A và B của (O) cắt nhau tại I. Một đường thẳng d đi qua I cắt AC, BC lần lượt tại M và N, cắt đường tròn (O) tại P, Q. Chứng minh rằng (MNPQ) = -1.
  • 28. Tải tài liệu tại sividoc.com Viết đề tài giá sinh viên – ZALO:0973.287.149-TEAMLUANVAN.COM 25 Giải. Bài toán xuất hiện các tiếp tuyến từ một điểm đến đường tròn, do đó sẽ xuất hiện các đường đối cực của một điểm nào đó đối với đường tròn. Chúng ta có thể khai thác yếu tố “cực” và “đường đối cực” trong bài toán này. A D I Q O . C T N P M B E Hình 2.7 Giải. Dựng các tiếp tuyến MD, ME. Giả sử N ' DE BC . Áp dụng định lý Briăng-xông cho lục giác AEDDBC có: AE DB T , ED BC N ', DD CA M . Khi đó M, N’, T thẳng hàng, từ đó suy ra PQ, BC, DE đồng quy. Mà PQ BC N N N ' . Do đó N ED suy ra (MNPQ) = -1 (tính chất “cát tuyến cắt đường nối hai tiếp điểm” với MD, ME là hai tiếp tuyến). Từ đó ta có điều phải chứng minh. 2.2. Chứng minh vuông góc Cực và đường đối cực là một công cụ hiệu quả trong chứng minh quan hệ vuông góc trong mặt phẳng. Dưới đây chúng tôi minh họa một số ví dụ về khai thác các tính chất của cực và đường đối cực trong giải bài toán chứng minh vuông góc. Ví dụ 2.8. [4] Giả sử đường tròn (O) với tâm O và bán kính R. Qua điểm M nằm trong đường tròn (M khác điểm O) vẽ hai dây cung CD và EF không đi qua tâm O. Hai tiếp tuyến tại C, D của (O) cắt nhau tại điểm A, hai tiếp tuyến tại E, F của (O) cắt nhau tại điểm B. Chứng minh rằng OM và AB vuông góc với nhau.
  • 29. Tải tài liệu tại sividoc.com Viết đề tài giá sinh viên – ZALO:0973.287.149-TEAMLUANVAN.COM 26 Giải. Bài toán có hai tiếp tuyến với đường tròn với yêu cầu chứng minh vuông góc. Điều này giúp ta liên tưởng đến đường đối cực của một điểm đối với một đường tròn. Ta thấy đường đối cực của điểm A là đườngthẳng CD đi qua M nên đường đối cực của điểm M sẽ đi qua điểm A (Hình 2.8). Tương Hình 2.8 tự, đường đối cực của điểm M đi qua điểm B. Vậy, đường thẳng AB chính là đường đối cực của điểm M. Do đó, AB vuông góc với OM. Ví dụ 2.9. [4] Cho tam giác ABC cân tại A. Hai đường thẳng d1, d2 bất kì qua điểm A. Các đường thẳng đi qua B, C tương ứng vuông góc với d1, d2 cắt nhau tại D. Đường thẳng đi qua B vuông góc với AB cắt d1 tại E, đường thẳng đi qua C vuông góc với AC cắt d2 tại F. Chứng minh rằng AD vuông góc với EF. Giải. Bài toán này không xuất hiện đường tròn nhưng ta để ý thấy yếu tố “cân” trong tam giác ABC. Vậy, có đường tròn tâm A, bán kính AB đi qua B và C (Hình 2.9). Dễ nhận thấy BE, CF lần lượt là các tiếp tuyến của đường tròn (A; AB). Đường đối cực của điểm E sẽ đi qua Hình 2.9 điểm B và vuông góc với AE hay d3. Tương tự, đường đối cực của điểm F sẽ đi qua điểm C và vuông góc với CF hay d4. Vậy, cực của đường thẳng EF đối với đường tròn (A; AB) chính là điểm D. từ đó suy ra AD vuông góc với EF.
  • 30. Tải tài liệu tại sividoc.com Viết đề tài giá sinh viên – ZALO:0973.287.149-TEAMLUANVAN.COM 27 Ví dụ 2.10. [4] Cho tam giác ABC với các đường cao BB’, CC’. Gọi E, F lần lượt là trung điểm của AC, AB. Đường thẳng EF cắt đường thẳng B’C’ tại điểm K. Chứng minh rằng AK vuông góc với đường thẳng Ơle của tam giác ABC. Giải. Ta xét cực và đường đối cực đối với đường tròn Ơle của tam giác ABC (đường tròn tâm O9). Gọi I là giao điểm của FB’ và EC’, G là giao điểm của CF và BE, H là giao điểm của BB’ và CC’. Áp dụng định lý Pa-puýt cho hai bộ ba điểm (F, C’, B) và (E, B’, C) ta suy ra ba điểm H, G, I thẳng hàng (Hình 2.10). Do đó, O9I là đường thẳng Ơle của tam giác ABC (1) Hình 2.10 Mặt khác, chú ý E, F, B’, C’ cùng nằm trên đường tròn (O9) nên suy ra AK chính là đường đối cực của điểm I. Vậy, O9I vuông góc với AK (2) Từ (1) và (2) ta suy ra điều phải chứng minh. Ví dụ 2.11. [4] Cho tam giác ABC nội tiếp đường tròn (O; R). Các đường phân giác trong BE, CF của các góc B, góc C cắt lại (O) lần lượt tại M, N. Đường thẳng qua điểm M vuông góc với BM cắt đường thẳng đi qua N vuông góc với CN tại điểm S. Chứng minh rằng SO vuông góc với EF. Giải. Trước hết ta tìm đường đối cực của điểm S đối với đường tròn (O) và chứng minh rằng nó song song với EF. Các đường thẳng SN, SM cắt lại (O) lần lượt tại L, G. Khi đó, ta dễ thấy C, O, G thẳng hàng và B, O, L thẳng hàng. Tiếp tuyến tại
  • 31. Tải tài liệu tại sividoc.com Viết đề tài giá sinh viên – ZALO:0973.287.149-TEAMLUANVAN.COM 28 G và N của (O) cắt nhau tại điểm Q, tiếp tuyến của tại L và M của (O) cắt nhau tại điểm P. Đường thẳng OP cắt LM tại điểm H, đường thẳng OQ cắt NG tại điểm K. Ta thấy, đường đối cực của điểm Q là đường thẳng GN đi qua S nên đường đối cực của điểm S đi qua điểm Q. Tương tự, đường đối cực của điểm S cũng đi qua điểm P. Do đó, đường đối cực của điểm S là PQ (Hình 2.11). Ta chứng minh PQ // EF. Thật vậy, ta thấy IE // OP, IF // OQ nên để chứng minh PQ // EF ta chỉ ra góc lượng giácFI , FE (QO, QP ) k 2 . Hình 2.11 Mặt khác, ta nhận thấy OK .OQ OG 2  OL2  OH .OP .Từ đó suy ra 5 điểm Q, K, H, P cùng nằm trên đường tròn vàQO, QP ( HK , HO) k 2 .Từ đó suy ra ta cần chỉ raFI , FE ( HK , HO) k 2 (1) Kẻ ID, IV lần lượt vuông góc với AC, AB và chú ý rằng: ID sinIFV IE  sinIED (vì ID = IV) IF sin IED IV sinIFV
  • 32. Tải tài liệu tại sividoc.com Viết đề tài giá sinh viên – ZALO:0973.287.149-TEAMLUANVAN.COM 29  C sin A  CM OK  2 sinNAC   (định lý hàm số sin) (2)  Bsin MAB BM OH sin A   2 Ta lại có IE // OH, IF // OK nênFI , FE ( HK , HO) k 2 (3) Từ (2) và (3) suy ra tam giác IEF đồng dạng với tam giác OKH. Do đó, (1) đúng nên suy ra điều phải chứng minh. Ví dụ 2.12. [4] Cho tứ giác ABCD ngoại tiếp đường tròn (I) và nội tiếp đường tròn (O). Tiếp điểm của đường tròn (I) trên các cạnh AB, BC, CD, DA lần lượt là M, N, P, Q. Chứng minh rằng MP vuông góc với NQ. Giải. Trường hợp tứ giác ABCD có ít nhất một cặp cạnh song song thì đơn giản. Ta sẽ giải bài toán trong trường hợp còn lại. Hình 2.12 Xét cực và đường đối cực đối với đường tròn (I) (Hình 2.12). Đường thẳng AB cắt đường thẳng CD tại điểm E, đường thẳng AD cắt đường thẳng BC tại điểm F. Ta thấy cực của đường thẳng MP là điểm E, cực của đường thẳng NQ là điểm F. Để giải bài toán ta chỉ cần chứng minh IE và IF vuông góc với nhau. Thật vậy, IE, IF lần lượt là phân giác của các gócAED,AFB. Gọi giao điểm của IF với AB và CD lần lượt là S, V thì ta cần chứng minh tam giác ESV cân tại điểm E.
  • 33. Tải tài liệu tại sividoc.com Viết đề tài giá sinh viên – ZALO:0973.287.149-TEAMLUANVAN.COM 30 Ví dụ 2.13. [4] Cho tứ giác ABCD ngoại tiếp đường tròn tâm (O). Gọi M, N, P, Q lần lượt là các tiếp điểm trên các cạnh AB, BC, CD, DA với đường tròn. Gọi K là giao điểm của MQ với NP. Chứng minh rằng OK vuông góc với AC. Giải. Bài toán xuất hiện các tiếp tuyến từ một điểm đến đường tròn, từ đó ta dễ dàng nhận thấy đường thẳng AC là đường đối cực của điểm K và đường thẳng QK là đường đối cực của điểm A. Do đó, gọi E và F là hai giao điểm của AC với đường tròn (O). Hai tiếp tuyến qua E và F với đường tròn (O) cắt nhau tại K’. Dễ dàng chứng minh được rằng các điểm K’, N, P thẳng hàng và K’, M, Q thẳng hàng (Hình 2.13). Từ đó suy ra K’ là giao điểm của MQ với NP hay K’ K. Vậy KE, KF là hai tiếp tuyến kẻ từ K với đường tròn (O). Từ đó suy ra KO EF hay KO AC. K A E Q M B O . N F D P C Hình 2.13 Ví dụ 2.14. [2] Cho tứ giác MNPQ nội tiếp đường tròn (O). Đặt K = QM PN, L = MN QP, I = MP QN. Chứng minh rằng I là trực tâm của tam giác KOL. Giải. Phân tích tương tự ví dụ 2.13 ta thấy xuất hiện các cực và đường đối cực trong bài toán này, do đó sẽ có các điểm cùng nằm trên một đường thẳng.
  • 34. Tải tài liệu tại sividoc.com Viết đề tài giá sinh viên – ZALO:0973.287.149-TEAMLUANVAN.COM 31 K A M B N O.I C Q P L D Hình 2.14 Kẻ bốn tiếp tuyến đi qua M, N, P, Q với đường tròn (O). Các tiếp tuyến này cắt nhau tại bốn điểm là A, B, C, D. Dễ thấy I là giao điểm của AC với BD. Mặt khác, ta thấy BD OL nên suy ra D, B, K thẳng hàng. Suy ra KI OL và LI KO hay I là trực tâm củaKOL (Hình 2.14). 2.3. Chứng minh song song Ví dụ 2.15. [4] Cho tam giác ABC có đường tròn nội tiếp là (I). Tiếp điểm của (I) trên các cạnh BC, CA, AB lần lượt là D, E, F. Đường thẳng AD cắt lại đường tròn (I) tại điểm M. Đường thẳng đi qua M vuông góc với đường thẳng AD cắt EF tại điểm N. Chứng minh rằng AN song song với BC. Giải. Xét cực và đường đối cực đối với đường tròn (I). Gọi P là giao điểm thứ hai của MN với (I). Dễ thấy D, P, I thẳng hàng. Đường thẳng EF cắt IP, IA lần lượt tại điểm J, G. Ta thấy AM . AD AE 2  AG. AI (Hình 2.15). Ta suy ra các điểm M, G, I, D cùng nằm trên đường tròn. Do đó: góc lượng giác
  • 35. Tải tài liệu tại sividoc.com Viết đề tài giá sinh viên – ZALO:0973.287.149-TEAMLUANVAN.COM 32 (GM , GF ) (GA, GF ) (GA, GM ) k 2 ( DI , DM )  ( MD, MP) ( DI , DM ) k ( PM , PD) k Từ đó, suy ra tứ giác MGJP nội tiếp. Ta có: NJ . NG NP.NM NE.NF . Hình 2.15 Chú ý rằng G là trung điểm của FE nên ta suy ra (NJEF) = -1 hay N thuộc đường đối cực của điểm J (1) Mặt khác, đường đối cực của điểm A là EF đi qua J nên đường đối cực của điểm J đi qua A (2) Từ (1) và (2) suy ra đường đối cực của điểm J là đường thẳng AN. Vậy IJ vuông góc với AN, mà IJ vuông góc với BC từ đó suy ra điều phải chứng minh. Ví dụ 2.16. [4] Cho hai đường thẳng a và a’ cắt nhau tại A và giả sử trên a ta có bốn điểm A, B, C, D sao cho (ABCD) = -1 và trên a’ có bốn điểm A, B’, C’, D’ sao cho (AB’C’D’) = -1. Chứng minh rằng các đường thẳng BB’, CC’, DD’ hoặc song song với nhau hoặc đồng quy. Giải. Bài toán sử dụng đến các chùm đường thẳng song song hoặc chùm các đường thẳng đồng quy. Tỉ số kép không đổi của một chùm đường thẳng được khai thác trong ví dụ này.
  • 36. Tải tài liệu tại sividoc.com Viết đề tài giá sinh viên – ZALO:0973.287.149-TEAMLUANVAN.COM 33 * Nếu BB’ và CC’ cắt nhau tại O, giả sử tia OD cắt đường thẳng a’ tại D’’. Vì (ABCD) = -1 O(ABCD) = -1 O(AB’C’D’) = -1 (AB’C’D’’) = -1. O a’ D’ C’ B’ A C B D a Hình 2.16 Mặt khác, theo giả thiết (AB’C’D’) = -1 nên D’ trùng với D’’. Vậy các đường thẳng BB’, CC’, DD’ đồng quy tại O. * Nếu BB’ và CC’ song song, từ A và D ta vẽ các đường thẳng song song với CC’ và BB’. Đường thẳng song song đi qua D cắt AB’ tại D’’, ta chứng minh D’’ trùng với D’. Từ đó, ta suy ra các đường thẳng BB’, CC’, DD’ song song. 2.4. Chứng minh thẳng hàng Cực và đường đối cực là công cụ hữu hiệu trong chứng minh thẳng hàng. Thật vậy, chúng ta có thể sử dụng tính chất “cực của các đường thẳng đồng quy thì thẳng hàng” hoặc quỹ tích các điểm liên hợp với một điểm cho trước để chỉ ra chúng cùng nằm trên đường đối cực của điểm đó. Ví dụ 2.17. [4] Cho một điểm A cố định và một đường thẳng d cố định không đi qua A. Gọi O là hình chiếu vuông góc của A trên d và I là trung điểm của đoạn thẳng AO. Trên đường thẳng d ta lấy hai điểm thay đổi P và Q không trùng với O. Dựng các đường thẳng Px và Qy vuông góc với d. Đường thẳng QI cắt AP và Px lần lượt tại M và N. Đường thẳng PI cắt AQ và Qy lần lượt tại M’ và N’. a) Chứng minh (QMIN) = -1, (PM’IN’) = -1. b) Chứng minh ba điểm N, A, N’ thẳng hàng.
  • 37. Tải tài liệu tại sividoc.com Viết đề tài giá sinh viên – ZALO:0973.287.149-TEAMLUANVAN.COM 34 Giải. Trong bài toán này, chúng ta cần chỉ ra các điểm N, A, N’ cùng nằm trên đường đối cực của điểm I đối với hai đường thẳng đồng quy AP, AQ. a) Ta có chùm (PQ, PM, PI, PN) là một chùm điều hoà vì có cát tuyến AIO song song với PN và AI = IO. Do đó (QMIN) = -1. Tương tự ta có chùm (QP, QM’, QI, QN’) là chùm điều hoà. Suy ra, ta có (PM’IN’) = -1 (Hình 2.17). y N’ d A M’ I M x N Q O P Hình 2.17 b) Vì (QMIN) = -1 nên AN là đường đối cực của điểm I đối với hai đường thẳng cắt nhau AP và AQ.Tương tự (PM’IN’) = -1 nên AN’ là đường đối cực của điểm I đối với hai đường thẳng cắt nhau AP và AQ, từ đó suy ra các điểm N, A, N’ cùng thuộc đường đối cực của điểm I đối với hai đường thẳng AP và AQ hay ba điểm N, A, N’ thẳng hàng. Ví dụ 2.18. [4] Cho tam giác ABC ngoại tiếp đường tròn (I). Tiếp điểm của đường tròn (I) trên các cạnh BC, CA, AB lần lượt là D, E, F. Gọi M, N, P lần lượt là điểm chung của các cặp đường thẳng (EF; BC), (DF; CA), (DE; AB). Chứng minh rằng các điểm M, N, P thẳng hàng. Giải. Bài toán xuất hiện các đường tròn và các tiếp tuyến đến đường tròn này. Do vậy, tính chất của cực và các đường đối cực của một điểm đối với một đường tròn sẽ được sử dụng trong chứng minh thẳng hàng. Thật vậy, đường đối cực
  • 38. Tải tài liệu tại sividoc.com Viết đề tài giá sinh viên – ZALO:0973.287.149-TEAMLUANVAN.COM 35 của điểm A đối với đường tròn (I) là EF đi qua điểm M nên đường đối cực của điểm M đi qua điểm A (Hình 2.18). Dễ thấy, đường đối cực của điểm M đi qua điểm D nên suy ra đường đối cực của điểm M đối với đường tròn (I) là đường thẳng AD. Tương tự, ta có đường đối cực của điểm N là đường thẳng BE, đường đối cực của điểm P là đường thẳng CF. Áp dụng định lý Xêva ta chứng minh được các đường thẳng AD, BE, CF đồng quy nên ba điểm M, N, P thẳng hàng. Hình 2.18 Ví dụ 2.19. [4] Cho tam giác ABC và một điểm O. Các đường thẳng đi qua O và vuông góc với OA, OB, OC theo thứ tự cắt BC, CA, AB tại M, N, P. Chứng minh rằng M, N, P thẳng hàng. Giải. Phân tích tương tự ví dụ 3.18, gọi A’, B’, C’ lần lượt là cực của các đường thẳng BC, CA, AB đối với đường tròn (O; R) với R > 0. Do BC, CA, AB không đồng quy nên A’, B’, C’ không thẳng hàng.
  • 39. Tải tài liệu tại sividoc.com Viết đề tài giá sinh viên – ZALO:0973.287.149-TEAMLUANVAN.COM 36 Hình 2.19 Vì đường đối cực của B’ đi qua điểm A nên B’ thuộc đường đối cực của điểm A đối với (O). Tương tự đường đối cực của C’ đi qua A nên đường đối cực của A đi qua C’. Từ đó, suy ra đường đối cực của điểm A đối với đường tròn (O) chính là B’C’. Tương tự ta có C’A’, A’B’ tương ứng là đường đối cực của hai điểm B, C đối với đường tròn (O) (Hình 2.19). Vì đường đối cực của M vuông góc với OM, OA OM nên đường đối cực của M song song với AO. Mà AO vuông góc với đường đối cực của điểm A nên đường đối cực của điểm M vuông góc với B’C’ (1). Vì M BC là đường đối cực của điểm A’ nên điểm A’ thuộc đường đối cực của điểm M (2). Từ (1) và (2) suy ra đường đối cực của điểm M là đường cao trong tam giác A’B’C’. Tương tự, các đường đối cực của điểm N, P đối với (O) cũng là đường cao trong tam giác A’B’C’ suy ra chúng đồng quy (Hình 2.19). Vậy các điểm M, N, P thẳng hàng. Ví dụ 2.20. [4] Cho tam giác ABC có (I) là đường tròn nội tiếp. Gọi D, E, F lần lượt là các tiếp điểm của (I) trên các cạnh BC, CA, AB. Gọi D’, E’, F’ lần lượt là các giao điểm của các đường thẳng EF với BC, FD với CA, DE với AB. Chứng minh rằng D’, E’, F’ thẳng hàng. Giải. Ta thấy EF là đường đối cực của A đối với (I) mà D’ EF nên điểm A thuộc đường đối cực của điểm D’ đối với (I). Do D’D là tiếp tuyến với (I) nên AD là
  • 40. Tải tài liệu tại sividoc.com Viết đề tài giá sinh viên – ZALO:0973.287.149-TEAMLUANVAN.COM 37 đường đối cực của điểm D’ đối với (I). Tương tự, ta có BE, CF cũng là đường đối cực của các điểm E’, F’ đối với (I). Hình 2.20 Ta biết AD, BE, CF đồng quy tại điểm Giéc-gôn, gọi là K. Khi đó, D’, E’, F’ phải nằm trên đường đối cực của điểm K đối với (I) (Hình 2.20). Từ đó suy ra D’, E’, F’ thẳng hàng và đường thẳng D’E’F’ vuông góc với IK. Ví dụ 2.21. [4] Cho tam giác ABC không cân. Các đường phân giác ngoài của các góc A, B, C cắt các cạnh đối diện lần lượt tại A', B', C'. Gọi O, I lần lượt là tâm đường tròn ngoại tiếp và nội tiếpABC. Chứng minh rằng các điểm A', B', C' thẳng hàng và đường thẳng A'B'C' vuông góc với OI. Giải. Bài toán không cho các đường vuông góc, nhưng với các giả thiết về đường tròn nội tiếp, đường tròn ngoại tiếp tam giác gợi ý cho chúng ta có thể sử dụng khái niệm cực và đường đối cực trong chứng minh bằng toán vuông góc. Gọi tiếp điểm của đường tròn (I) nội tiếp tam giác trên BC, CA, AB lần lượt là D, E, F. Gọi M, N, P lần lượt là trung điểm của các cạnh FE, FD, DE. Xét cực và đường đối cực đối với đường tròn (I). Ta thấy AA' là đường đối cực của M nên A' thuộc đường đối cực của M. Mà A' thuộc BC là đường đối cực của D nên ta có đường đối cực của A' chính là đường thẳng DM (1) Tương tự, đường đối cực của B', C' lần lượt là các đường thẳng EN, FP (2) (Hình 2.21).
  • 41. Tải tài liệu tại sividoc.com Viết đề tài giá sinh viên – ZALO:0973.287.149-TEAMLUANVAN.COM 38 Chú ý rằng các đường thẳng DM, EN, FP đồng quy tại trọng tâm G của tam giác DEF (3) Từ (1), (2), (3) ta có A', B', C' thẳng hàng và đường thẳng A'B'C' vuông góc với IG (đường thẳng Ơle của tam giác DEF). Hình 2.21 Ví dụ 2.22. [4] Cho tứ giác ABCD ngoại tiếp đường tròn (O) và M, N, P, Q lần lượt là các tiếp điểm trên các cạnh AB, BC, CD, DA của tứ giác. Đặt K = ADBC,L=ABDC,E=QMPN,F=QPMN. Chứng minh bốn điểm K, L, E, F thẳng hàng. Giải. Bài toán xuất hiện các cực và đường đối cực của điểm đối với đường tròn. Do đó, gọi I là giao điểm của BD với AC, E’ là giao điểm của DB với KL, T là giao điểm của CE’ với DK (Hình 2.22). Dễ thấy (TAKD) = -1 suy ra (CT, CA, CK, CD) = -1. Do đó (E’IBD) = -1. Mặt khác, (EIBD) = -1 nên suy ra E’ E. Từ đó suy ra E, K, L thẳng hàng (1) Lập luận tương tự cũng có F, K, L thẳng hàng (2) Từ (1) và (2) ta suy ra điều phải chứng minh.
  • 42. Tải tài liệu tại sividoc.com Viết đề tài giá sinh viên – ZALO:0973.287.149-TEAMLUANVAN.COM 39 F T K A E M B Q D O. I P N L C Hình 2.22 Ví dụ 2.23.[2] Cho tứ giác ABCD nội tiếp đường tròn (O). Gọi  S   ABCD,  F   ADBC,  E ACBD . Kẻ tiếp tuyến SM, SN với (O).  Chứng minh rằng bốn điểm E, F, M, N thẳng hàng. Giải. Bài toán có các tiếp tuyến đối với đường tròn, do vậy nó có liên quan đến bài toán dựng đường đối cực của một điểm đối với đường tròn và từ đó làm xuất hiện các tứ giác toàn phần. Giả sử K  CDEF , K ' CDMN. Khi đó, theo tính chất của tứ giác toàn  phần FEAB ta có (SKDC) = -1 (1) Mặt khác theo tính chất “cát tuyến”, ta có (SK’DC) = -1 (2)
  • 43. Tải tài liệu tại sividoc.com Viết đề tài giá sinh viên – ZALO:0973.287.149-TEAMLUANVAN.COM 40 F . . M B A E . O S D K C N Hình 2.23 Từ (1) và (2) ta có K K’. Tương tự, ta cũng có L L’ với LEFAB , L'MNAB .   Từ đó EF và MN có hai điểm chung nên hai đường thẳng này phải trùng nhau. Vậy ta có M, N, E, F thẳng hàng. 2.5. Chứng minh đồng quy Bài toán chứng minh đồng quy có thể coi là bài toán “đối ngẫu” của bài toán chứng minh thẳng hàng. Phép đối cực chính là phương tiện để chuyển đổi hai dạng bài toán. Do vậy, cực và đường đối cực sẽ được khai thác triệt để trong giải bài toán dạng này. Ví dụ 2.24. [4] Cho tam giác ABC. Đường tròn nội tiếp tam giác ABC tiếp xúc với các cạnh BC, CA, AB lần lượt tại D, E, F. Đường tròn nội tiếp tam giác DEF tiếp xúc với EF, FD, DE lần lượt tại M, P, N. Chứng minh rằng các đường thẳng AM, BP, CN đồng quy. Giải. Gọi I, O lần lượt là tâm của đường tròn nội tiếp tam giác DEF và tam giác ABC. Gọi H, K, L lần lượt là giao điểm của các cặp đường thẳng (MP; EF), (MN; FD), (MP; DE). Ta dễ thấy H, K, L thẳng hàng (1)
  • 44. Tải tài liệu tại sividoc.com Viết đề tài giá sinh viên – ZALO:0973.287.149-TEAMLUANVAN.COM 41 Hình 2.24 Chú ý rằng DM, FN, EP đồng quy nên (HMFE) = -1. Do đó, M thuộc đường đối cực của điểm H đối với đường tròn (O). Mặt khác, điểm A thuộc đường đối cực của điểm H đối với (O) nên ta có AM là đường đối cực của điểm H đối với (O) (2) Tương tự, ta có BP là đường đối cực của điểm K đối với (O) và Cn là đường đối cực của điểm L đối với (O) (3) Từ (1), (2) và (3) suy ra điều phải chứng minh. Ví dụ 2.25. [4] Cho tứ giác ABCD nội tiếp đường tròn (O). Gọi M, N lần lượt là trung điểm của AB, CD. Đường tròn (ABN) cắt lại cạnh CD tại điểm P, đường tròn (CDM) cắt lại cạnh AB tại điểm Q. Chứng minh rằng các đường thẳng AC, PQ, BD đồng quy. Giải. Trong bài toán này, chúng ta sẽ tìm các đường đối cực của các điểm đối với đường tròn (O). Từ đó khai thác tính chất của các đường đối cực trong chứng minh đồng quy. Thật vậy, khi AB // CD thì bài toán đơn giản. Ta đi xét trường hợp còn lại.
  • 45. Tải tài liệu tại sividoc.com Viết đề tài giá sinh viên – ZALO:0973.287.149-TEAMLUANVAN.COM 42 Gọi S là giao điểm của đường thẳng AB và CD (Hình 2.25). Gọi d là đường đối cực của điểm S đối với (O). Gọi I là giao điểm của AC và BD thì dễ thấy điểm I thuộc đường thẳng d. Ta thấy SM .SQ SC.SD SA.SB . Chú ý rằng M là trung điểm của AB nên ta có (SQAB) = -1. Do đó, điểm Q thuộc đường thẳng d. Tương tự ta có điểm P cũng thuộc đường thẳng d. Từ đó suy ra điều phải chứng minh. Hình 2.25 Ví dụ 2.26. [4] Trong tam giác ABC kẻ các đường cao AA’, BB’, CC’. Gọi H là trực tâm của tam giác ABC. Gọi J là một giao điểm của AA’ với đường tròn (I) đường kính BC. Chứng minh rằng BC, B’C’ và tiếp tuyến tại điểm J của đường tròn (I) đồng quy. Giải. Tương tự ví dụ 2.25, trong bài toán này, chúng ta cũng sẽ đi tìm các đường đối cực của một điểm nào đó đối với đường tròn (I). Gọi giao điểm của AH với đường tròn (I) là J1, J2. Vậy điểm J sẽ là J2 hoặc J1. Ta chứng minh BC, B’C’ và tiếp tuyến tại J1 của đường tròn (I) đồng quy. Xét cực và đường đối cực đối với đường tròn (I). Gọi giao điểm của BC và B’C’ là điểm S (Hình 2.26). Ta thấy AH là đường đối cực của điểm S, AH đi qua điểm J1 nên đường đối cực của J1 sẽ đi qua điểm S hay tiếp tuyến tại J1 đi qua điểm S. Vậy ta có điều phải chứng minh.
  • 46. Tải tài liệu tại sividoc.com Viết đề tài giá sinh viên – ZALO:0973.287.149-TEAMLUANVAN.COM 43 Hình 2.26 Ví dụ 2.27. [4] Gọi O là tâm đường tròn nội tiếp tứ giác ABCD. Qua A, B, C, D lần lượt vẽ các đường thẳng dA, dB, dC và dD tương ứng vuông góc với OA, OB, OC và OD. Các cặp đường thẳng dA và dB, dB và dC, dC và dD, dD và dA tương ứng cắt nhau tại K, L, M, N. Chứng minh rằng KM và LN cắt nhau tại O. Giải. Gọi I, J, P, Q lần lượt là tiếp điểm của đường tròn (O) trên AB, BC, CD, DA (Hình 2.27). Hình 2.27
  • 47. Tải tài liệu tại sividoc.com Viết đề tài giá sinh viên – ZALO:0973.287.149-TEAMLUANVAN.COM 44 Gọi E, F, G, H lần lượt là giao điểm của các cặp đường thẳng (OA; IQ), (OB; IJ), (OC; JP), (OD; PQ). Ta sẽ chứng minh K, O, M thẳng hàng (Hình 2.27). Theo giả thiết ta sẽ có dA là đường đối cực của điểm E đối với đường tròn (O). Tương tự, dB là đường đối cực của điểm F. Từ đó suy ra EF là đường đối cực của điểm K, GH là đường đối cực của điểm M đối với đường tròn (O). Mặt khác, dễ thấy EF // GH. Từ đó suy ra điều phải chứng minh. Ví dụ 2.28. [4] Cho tứ giác ABCD ngoại tiếp đường tròn (O). Tiếp điểm thuộc các cạnh AB, BC, CD, DA lần lượt là M, N, P, Q. Các đường thẳng AN, AP cắt đường tròn (O) tại E, F. Chứng minh rằng: a) Các đường thẳng MP, NQ, AC, BD đồng quy. b) Các đường thẳng ME, QF, AC đồng quy. Giải. .I . J Hình 2.28 a) Hạ CJ MP. Ta có:OMPOPMBMPCPM CJ CP . GọiI AC MP IA  AM  AM (1) IC JC PC Tương tự gọi I' ACNQ I ' A AQ (2)  I ' C NC
  • 48. Tải tài liệu tại sividoc.com Viết đề tài giá sinh viên – ZALO:0973.287.149-TEAMLUANVAN.COM 45 Vì AM = AQ và PC = PN nên từ (1) và (2) suy ra I I ' . Ta suy ra các đường thẳng MP, NQ, AC đồng quy tại I (3) Tương tự, ta cũng có MP, NQ, BD đồng quy tại I (4) Kết hợp (3) và (4) ta có điều phải chứng minh. b) Gọi K là cực của đường thẳng AC đối với đường tròn (O). Xét tứ giác nội tiếp MNPQ. Theo tính chất cực và đường đối cực của tứ giác nội tiếp ta có MQ và NP cắt nhau tại điểm K. Tương tự, tứ giác EFPN nội tiếp cũng có EF và NP cắt nhau tại điểm K suy ra MQ và EF cắt nhau tại điểm K . Vì ME và QF cắt nhau tại một điểm thuộc đường đối cực của K đối với (O) tức thuộc đường thẳng AC. Do đó, ba đường thẳng ME, QF, AC đồng quy. Ví dụ 2.29.(Định lý Đờ-giác) [3] Cho ΔABC và ΔA’B’C’. Ta gọi giao điểm của cạnh BC và B’C’, cạnh CA và C’A’, cạnh AB và A’B’ lần lượt là X, Y, Z. Chứng minh rằng các điểm X, Y, Z thẳng hàng khi và chỉ khi các đường thẳng AA’, BB’, CC’ đồng quy. Giải. Định lý này có thể chứng minh dựa vào định lý Xêva kết hợp với định lý Mênêlauýt. Tuy nhiên, các cách chứng minh đó không thể cho ta ngay chiều ngược lại của bài toán. Phương pháp sử dụng tỉ số kép của chùm đường thẳng hay phép chiếu xuyên tâm dưới đây sẽ khắc phục điều này. B D C A Y Z X A’ C’ B’ D’ Hình 2.29
  • 49. Tải tài liệu tại sividoc.com Viết đề tài giá sinh viên – ZALO:0973.287.149-TEAMLUANVAN.COM 46 Thật vậy, gọi AA’BY = D, AA’B’Y = D’. Ta có ba điểm X, Y, Z thẳng hàng A(XYZA’) = A’(XYZA) (YCDB) = (C’YB’D’) (YCDB) = (C’YB’D’) = (YC’D’B) AA’, BB’, CC’ đồng quy (Hình 2.29). 2.6. Chứng minh điểm cố định Đường đối cực của một điểm đối với hai đường thẳng cắt nhau hoặc đối với đường tròn cho trước là tập hợp các điểm liên hợp điều hòa với điểm đó. Do vậy, chúng ta có thể khai thác tính chất của cực và đường đối cực để giải quyết các bài toán tìm điểm cố định qua các ví dụ cụ thể dưới đây. Ví dụ 2.30. [4] Cho tam giác ABC nội tiếp trong đường tròn (O). Gọi D và D’ là chân hai đường phân giác trong và ngoài của góc A. Gọi P là giao điểm của hai tiếp tuyến của (O) tại B và C. Chứng minh rằng cực của đường thẳng AP đối với (O) là trung điểm của DD’. Giải. Gọi E là trung điểm của DD’. Ta có đường đối cực của điểm P là đường thẳng BC đi qua E (Hình 2.30). Do vậy E và P là hai điểm liên hợp với nhau đối với đường tròn (O) (1) Mặt khác, do AD là đường phân giác của gócBAC và AD A D’ nên ta có (D’DBC) = -1 (chùm tâm A). Vì E là trung điểm của DD’ nên theo hệ thức Niu-tơn ta có: ED '2  ED 2 EB .EC (2) Hình 2.30
  • 50. Tải tài liệu tại sividoc.com Viết đề tài giá sinh viên – ZALO:0973.287.149-TEAMLUANVAN.COM 47 XétADD’ với AE là đường trung tuyến. Ta có AE = ED = ED’ (3) Từ (2) và (3) ta suy ra EA2  . . Ta suy ra AE là tiếp tuyến của (O) tại EB EC điểm A. Do đó, E và A liên hợp với nhau đối với đường tròn (O) (4) Từ (1) và (4) suy ra đường đối cực của điểm E đối với đường tròn (O) chính là đường thẳng AP. Ví dụ 2.31. [4] Cho đường tròn (O) đường kính AB và đường thẳng d vuông góc với AB tại điểm I ở ngoài đường tròn. Điểm M thay đổi trên (O), các đường thẳng MA, MB cắt d lần lượt tại P và Q. Đường thẳng QA cắt đường tròn (O) tại điểm N. Chứng minh rằng đường thẳng MN đi qua một điểm cố định. Giải. Giả thiết bài toán có đường tròn và đường thẳng d cố định, điều đó làm xuất hiện suy nghĩ rằng điểm cố định cần tìm có thể là cực của một đường thẳng cố định đối với một đường tròn cho trước. Do đó, khai thác tính chất của cực và đường đối cực đối với đường tròn cũng là một hướng tiếp cận trong bài toán này. Gọi E = AO MN. XétBQP ta có AM QB, AI PQ nên suy ra A là trực tâm của tam giác BQP. Từ đó ta có QA BP hay QN BP (Hình 2.31). Mà BN QN nên suy ra P, N, B thẳng hàng. Mặt khác, đường thẳng QE là đường đối cực của điểm P đối với đường tròn (O) nên hai điểm E và P liên hợp với nhau đối với (O). Mà PQ OE nên ta suy ra E là cực của đường thẳng PQ đối với đường tròn (O). Vậy E là điểm cố định (vì PQ cố định). Hình 2.31
  • 51. Tải tài liệu tại sividoc.com Viết đề tài giá sinh viên – ZALO:0973.287.149-TEAMLUANVAN.COM 48 Ví dụ 2.32. [4] Từ điểm P nằm ngoài đường tròn (O) ta vẽ các tiếp tuyến PA và PB tới đường tròn (O). Từ điểm B hạ đường vuông góc BD với đường kính AC. Chứng minh rằng PC đi qua trung điểm BD. Giải. Bài toán xuất hiện các đường tiếp tuyến đối với đường tròn, do đó chúng ta có thể xác định các cặp điểm liên hợp điều hòa trong bài toán này. Hình 2.32 Gọi I là giao điểm của PC và BD (Hình 2.32). Kéo dài PB cắt AC tại điểm E. Ta có hai điểm B và E liên hợp với nhau đối với đường tròn (O). Mà BD CE nên đường đối cực của điểm E đối với đường tròn (O) là đường thẳng BD. Vậy hai điểm E và D liên hợp với nhau đối với (O) nên suy ra (EDCA) = -1 hay P(EDCA) = -1. Mà ta có PA // BD suy ra IB = ID. Vậy PC đi qua trung điểm của BD. Ví dụ 2.33. [4] Cho đường tròn (O) và dây cung AB. Từ trung điểm I của dây cung AB kẻ hai dây cung MN và PQ. Các đường thẳng MP và NQ cắt dây cung AB lần lượt tại J và K. Chứng minh rằng I cũng là trung điểm của JK. Giải. Đây là bài toán con bướm quen thuộc, chúng ta có thể chứng minh thông qua các tam giác đồng dạng và các góc bằng nhau. Tuy nhiên, sử dụng khái niệm đường đối cực giúp lời giải trở nên thú vị hơn. Gọi D là giao điểm của hai tiếp tuyến tại A và B của đường tròn (O). Khi đó, ta có AB chính là đường đối cực của điểm D đối với đường tròn (O) suy ra I và D là hai điểm liên hợp với nhau đối với đường tròn (O) (Hình 2.33). Kẻ Dx OI suy ra Dx chính là đường đối cực của điểm I đối với (O). Mặt khác, I và C là hai điểm liên hợp với nhau đối với (O) nên C Dx. Gọi E là giao điểm của PQ và Dx. Khi đó hai
  • 52. Tải tài liệu tại sividoc.com Viết đề tài giá sinh viên – ZALO:0973.287.149-TEAMLUANVAN.COM 49 điểm E và I liên hợp với nhau đối với (O), ta suy ra (PQIE) = -1 hay C(PQIE) = -1. Mà ta có JK // Cx suy ra IJ = JK. Vậy I là trung điểm của đoạn thẳng JK. Hình 2.33 Ví dụ 2.34.[4] Cho đường tròn (O), điểm M nằm ngoài (O) và điểm I nằm trong (O). Một đường thẳng thay đổi qua I cắt (O) tại A, A’. Các đường thẳng MA, MA’ lần lượt cắt (O) tại các điểm thứ hai B, B’. Chứng minh rằng đường thẳng BB’ đi qua một điểm cố định. Giải. * Trường hợp 1: Gọi N = AB’ A’B và P = AA’ BB’. Ta có NP chính là đường đối cực của điểm M đối với (O). Hình 2.34
  • 53. Tải tài liệu tại sividoc.com Viết đề tài giá sinh viên – ZALO:0973.287.149-TEAMLUANVAN.COM 50 Gọi Q = NP MI, R = BB’ MI, S = NP MA’ suy ra P(MSB’A’) = -1. Từ đó ta có P(MQRI) = -1 suy ra (MQRI) = -1. Do M, Q, I cố định nên R cũng cố định. Vậy BB’ đi qua điểm cố định R MI thỏa mãn (MORI) = -1 với Q là giao điểm của đường đối cực của điểm M đối với (O) với đường thẳng MI (Hình 2.34). * Trường hợp 2: AB’ // A’B. Gọi J = AA’ BB’. Ta có MO chính là đường trung trực của A’B nên cũng đi qua J. Gọi C, D lần lượt là giao điểm của MO với (O). Khi đó, do CD là đường kính nên gócDAC = 900 . Mặt khácA’AC =CAB nên suy ra chùm A(DCJM) là chùm phân giác suy ra (DCJM) = -1. Vậy điểm J thuộc đường đối cực của điểm M đối với (O) suy ra J(B’A’SM) = -1 hay (MQRI) = -1. Vậy điểm R cố định (Hình 2.35). Hình 2.35 * Trường hợp 3: AA’ // BB’. Ta chứng minh giao của DR và CI nằm trên đường đối cực của điểm M đối với đường tròn (O). Khi đó (MQRI) = (MJCD) = -1 (Hình 2.36). Vậy R là điểm cố định. Hình 2.36
  • 54. Tải tài liệu tại sividoc.com Viết đề tài giá sinh viên – ZALO:0973.287.149-TEAMLUANVAN.COM 51 Ví dụ 2.35. [4] Cho đường tròn (O) và một đường thẳng d nằm ngoài (O). Một điểm S di chuyển trên đường thẳng d. Từ S kẻ hai tiếp tuyến SA, SB tới (O). Chứng minh rằng khi S di chuyển trên d thì AB luôn đi qua một điểm cố định. Giải. Gọi I là cực của đường thẳng d đối với đường tròn (O). Vì d cố định nên điểm I cố định (Hình 2.37). Vì điểm S thuộc đường thẳng d nên suy ra I Hình 2.37 đường đối cực của điểm S sẽ đi qua cực của đường thẳng d hay đường thẳng AB luôn đi qua điểm I cố định. Ví dụ 2.36. [2] Cho đường tròn (O), trên (O) có hai điểm B, C cố định và điểm A thay đổi. Đường kính DE vuông góc với BC và cắt AB, AC lần lượt tại M, N. Lấy P, Q sao cho (ABMP) = (ACNQ) = -1. Chứng minh rằng PQ đi qua một điểm cố định. Giải. * Cách 1: Gọi I là trung điểm của đoạn thẳng BC, ta chứng minh PQ đi qua I GọiF BQ DE. Vì (ACNQ) = -1 (QM, BI, BN, BF) = -1 (MINF) = -1 (1) Gọi   (Hình 2.38). I ' PQ BC . Ta chứng minh I ' I Do (ABMP) = -1 (QN, QF, QM, QI’) = -1  (NFMI’) = -1 (2) Từ (1) và (2) suy ra I ' I . Vậy đường thẳng PQ đi qua điểm I cố định. * Nhận xét: Nếu PQ đi qua điểm I thì từ (ABMP) = -1 Q(ABMP) = -1. Ta có BC cắt QA, QB, QP lần lượt tại B, I, C mà IB = IC nên ta nghĩ đến MQ // BC. Từ đó ta có cách giải khác như sau: * Cách 2: Giả sử IQ ABP ', ta sẽ chứng minh P ' P . Qua điểm A kẻ đường thẳng vuông góc với DE và cắt DE, IQ lần lượt tại H, K.
  • 55. Tải tài liệu tại sividoc.com Viết đề tài giá sinh viên – ZALO:0973.287.149-TEAMLUANVAN.COM 52 MA  HA  HA  NA  QA (do AH//BC,  và tỉ số kép Ta có IB IC MB IB IC NC QC (ACNQ) = -1)) MQ //BC. N P D A M Q F B I C E Hình 2.38 P ' A KA Khi đó, ta có  (do AK//IB) P 'A KA QA MA . P ' B IB P ' B IC QC MB Hay MA :P ' A 1, từ đó suy ra (ABMP’) = -1. MB P ' B Mặt khác, theo giả thiết (ABMP) = -1. Do đó suy ra P ' P . * Nhận xét: Nếu sử dụng định lý Mênêlauýt, ta có thể suy ra cách giải thứ ba dưới đây: * Cách 3: Gọi I là trung điểm đoạn thẳng BC. Áp dụng định lý Mênêlauýt choABC , ta cần chứng minh IB . QC .PA 1. IC PB QA MA : PA 1 PA  MA Vì (ABMP) = -1 nên hay . MB PB PB MB
  • 56. Tải tài liệu tại sividoc.com Viết đề tài giá sinh viên – ZALO:0973.287.149-TEAMLUANVAN.COM 53 Tương tự, ta có (ACNQ) = -1 nên NA : QA 1 hay QC NC . NC QC QA NA    IB QC PA  IB  NC  MA 1 (áp dụng định lý Mênêlauýt Từ đó . . .   .   IC QA PB IC NA  MB với MNI là cát tuyến trongABC). Ví dụ 2.37. [2] Cho hai đường thẳng cố định Ox, Oy và điểm A không nằm trên Ox, Oy và phân giácxOy. Hai đường thẳng di động qua A, đối xứng qua OA, một đường cắt Ox tại M, đường kia cắt Oy tại N. Chứng minh rằng đường thẳng MN đi qua một điểm cố định. Giải. Theo đề bài A và O là hai điểm cố định. Các đường thẳng AM, AN lại đối xứng qua OA, nghĩa là phân giác củaMAN, từ đó ta nghĩ đến chùm phân giác. Do đó, ta kẻ đường thẳng d qua điểm A và vuông góc OA để tạo ra chùm phân giác. d K M O I A N Hình 2.39 Gọi d là đường thẳng qua A và vuông góc với OA, K là giao điểm của MN và d, I là giao điểm của OA và MK. Do AM và AN đối xứng với nhau qua OA nên AO là phân giác củaMAN. Mà AI AK, do đó AK, AI, AM, AN là chùm phân giác nên là chùm đường thẳng điều hòa A(KIMN) = -1 (KIMN) = -1 O(KIMN) = -1.
  • 57. Tải tài liệu tại sividoc.com Viết đề tài giá sinh viên – ZALO:0973.287.149-TEAMLUANVAN.COM 54 Mà các đường thẳng OM, OI, ON cố định nên đường thẳng OK cố định. Mặt khác,K OK d điểm K cố định. Vậy đường thẳng MN luôn đi qua điểm K cố định (Hình 2.39). Ví dụ 2.38. [2] Cho tam giác ABC không cân (B, C cố định, A thay đổi), với ba đường cao AD, BE, CF. Đường thẳng d đi qua D, song song với EF, cắt các đường thẳng AB, AC lần lượt tại M, N, các đường thẳng EF, BC cắt nhau tại P. Chứng minh rằng đường tròn ngoại tiếp tam giác MNP đi qua một điểm cố định. Giải. Giả thiết của bài toán có các đường phân giác nên ta nghĩ đến chùm phân giác và I là trung điểm của DE nên nghĩ đến hệ thức Mácloranh trong bài toán. A d E F N P B . D Q C M Hình 2.40 Theo tính chất của tứ giác toàn phần, ta có (BCDP) = -1. Khi đó, nếu gọi Q là trung điểm của cạnh BC, theo hệ thức Mácloranh ta có: DQ .DP DB . DC (1) Mặt khác, do tứ giác BCEF nội tiếp (đường tròn đường kính BC) và MN // EF nên (NC, NM) = (EC, EF) = (BC, BF) = (BC, BM). Từ đó, suy ra bốn điểm B, C, M, N cùng nằm trên một đường tròn. Do đó: DB .DC DM .DN (2) Từ (1) và (2) suy ra DQ.DP DM .DN , tức là bốn điểm P, Q, M, N cùng nằm trên đường tròn. Vậy đường tròn ngoại tiếp tam giác MNP đi qua điểm Q cố định. Ví dụ 2.39. [2] Cho đường tròn (O). Hai điểm B, C cố định trên đường tròn, BC không phải đường kính. Lấy A là điểm trên đường tròn không trùng với B, C. Các đường thẳng AD, AE lần lượt là đường phân giác trong và ngoài của
  • 58. Tải tài liệu tại sividoc.com Viết đề tài giá sinh viên – ZALO:0973.287.149-TEAMLUANVAN.COM 55 BAC. Điểm I là trung điểm của DE. Qua trực tâm của tam giác ABC kẻ đường thẳng vuông góc với AI cắt AD, AE lần lượt tại M, N. Chứng minh rằng MN luôn đi qua một điểm cố định. Giải. Gọi 2 là độ lớn cung nhỏ BC. Khi đóBAC bằng hoặc 180o - . Gọi J là điểm đối xứng của O qua BC, suy ra J cố định. Ta có OJ = 2d(O,BC) = 2Rcos = AH, OJ // AH (vì cùng vuông góc với BC) nên AOJH là hình bình hành. Ta suy ra AO // HJ (1) A N O. H B D I E C M J Hình 2.41 Lại có (CBDE) = -1 nên theo hệ thức Niutơn ta có ID 2  . , mà IA = ID IB IC (tam giác ADE vuông tại A), suy ra IA2  . . IB IC Do đó, IA tiếp xúc (O) hay IA OA (2) Từ (1) và (2) suy ra JH AI, mà MN đi qua H và vuông góc với AI nên M, N, J thẳng hàng. Vậy MN đi qua điểm J cố định. 2.7. Chứng minh đẳng thức Hệ thức Đề-các, hệ thức Niutơn và hệ thức Mácloranh đối với hàng điểm điều hòa là những công cụ hiệu quả khi giải bài toán chứng minh đẳng thức, đặc biệt là các bài toán đẳng thức chứa các tỷ lệ thức. Chúng ta cần nhận dạng những đẳng thức có liên quan đến hàng điểm điều hòa như các ví dụ dưới đây.
  • 59. Tải tài liệu tại sividoc.com Viết đề tài giá sinh viên – ZALO:0973.287.149-TEAMLUANVAN.COM 56 Ví dụ 2.40. [4] ChoABC và AH = h là chiều cao xuất phát từ A. Gọi r và r’ lần lượt là bán kính của đường tròn nội tiếp và bàng tiếp đối với góc A của tam giác ABC. Chứng minh rằng: h 2  1 r r 1 ' . Giải. Đẳng thức cần chứng minh chính là hệ thức Đề-các đối với hàng điểm điều hòa, vì vậy ta phải tìm cách xác định được hàng điểm điều hòa đó. Bài toán cho h là chiều cao xuất phát từ A, còn r, r’ là bán kính của đường tròn nội tiếp và bàng tiếp đối với góc A của tam giác ABC, điều này làm ta nghĩ đến sử dụng chùm phân giác để suy ra hàng điểm điều hòa trong bài toán này. Gọi I là tâm đường tròn nội tiếp tam giác ABC và K là tâm đường tròn bàng tiếp góc A của tam giác đó. Các đường thẳng BI, BK là phân giác của góc B. Ta suy ra các tia BA, BC, BI, BK tạo nên một chùm điều hoà. Gọi D là giao điểm của AI và BC ta có (ADIK) = -1. Chiếu vuông góc các điểm A, I, D, K xuống đường cao AH ta được lần lượt các điểm A, I', H, K'. Trên tia HA chọn H làm gốc toạ độ, ta có: HA h, HI ' r , HK 'r ' Ta có (AHI'K') = -1  HA 2  HI 1 'HK 1 ' (theo hệ thức Đề-các đối với hàng điểm điều hòa). 2 1 1 Hình 2.42 hay h r r ' . Ví dụ 2.41. [4] Cho tam giác ABC cóBCA = 900 . Gọi D là chân đường cao hạ từ C, X là điểm nằm trên đoạn thẳng CD. Gọi K là điểm thuộc đoạn thẳng AX sao cho BK = BC. Tương tự L là điểm trên đoạn thẳng BX sao cho AL = AC. Gọi M là giao điểm của AL và BK. Chứng minh rằng MK = ML.
  • 60. Tải tài liệu tại sividoc.com Viết đề tài giá sinh viên – ZALO:0973.287.149-TEAMLUANVAN.COM 57 Giải. Dễ thấy các đường tròn (A, AC) và (B, BC) cắt nhau tại điểm E khác C thì E đối xứng với C qua đường thẳng AB. Khi đó, dễ thấy AC, AE cùng tiếp xúc với đường tròn (B, BC). Gọi Q = AK (B, BC) (Q khác K). Do AC, AE cùng tiếp xúc với đường tròn (B, BC) nên tứ giác CQEK là tứ giác điều hòa. Do đó, tiếp tuyến tại K và Q của (B, BC) cắt nhau tại điểm P thuộc CE. Hơn nữa ta có (PXCE) = -1. Vậy tương tự, nếu gọi N = BL (A, AC) thì tiếp tuyến tại L và N cắt nhau tại P’ thuộc CE và (P’XCE) = -1. Do đó P P’. Từ đó, chú ý CE là trục đẳng phương của (A, AC) và (B, BC) nên PL = PK. Từ đó ta dễ thấy hai tam giác vuôngPML =PMK suy ra MK = ML (Hình 2.43). Hình 2.43 Ví dụ 2.42. [4] Cho hai đường tròn (O1) và (O2) cắt nhau tại A, B. Gọi C, D thuộc đường thẳng O1O2 sao cho AC vuông góc với O1A và AD vuông góc với O2A. Gọi P là điểm thuộc đoạn thẳng AB. Đường thẳng CP giao với (O1) tại L sao cho C, L khác phía với AB. Đường thẳng DP giao với (O2) tại K sao cho D, K khác phía với AB, LO1 cắt KO2 tại M. Chứng minh rằng MK = ML.
  • 61. Tải tài liệu tại sividoc.com Viết đề tài giá sinh viên – ZALO:0973.287.149-TEAMLUANVAN.COM 58 Giải. Gọi DK giao (O2) tại R khác K. Ta dễ thấy DA, DB tiếp xúc với (O2) do đó tứ giác ARBK điều hòa. Vậy tiếp tuyến tại K và R của (O2) cắt nhau tại Q thuộc AB và (ABPQ) = -1 (Hình 2.44). Hình 2.44 Tương tự, gọi CL giao với (O1) tại S khác L thì tiếp tuyến tại S và L của (O1) cắt nhau tại Q’ thuộc AB và (ABPQ’) = -1 do đó Q Q’. Từ đó QL, QK lần lượt tiếp xúc (O1), (O2) mà AB là trục đẳng phương của (O1), (O2) do đó QL = QK. Từ đó, ta dễ thấy hai tam giác vuôngQML =QMK suy ra MK = ML. Ví dụ 2.43. [2] Cho hình bình hành ABCD và d là đường thẳng thay đổi qua A và cắt BD, BC, CD lần lượt tại E, F, G. Chứng minh rằng AE 1  AF 1  AG 1 . Giải. Đẳng thức cần chứng minh có dạng giống như hệ thức Đề-các, do đó ta cần dựng một điểm K sao cho AK 2  AF 1  AG 1 , nghĩa là AK 2AE , cần chứng minh (AKFG) = -1.
  • 62. Tải tài liệu tại sividoc.com Viết đề tài giá sinh viên – ZALO:0973.287.149-TEAMLUANVAN.COM 59 A D O E B F C K L G Hình 2.45 * Cách 1: Gọi K là điểm thuộc d sao cho AK 2AE hay E là trung điểm của AK. Khi đó EO // KC (tính chất đường trung bình tam giác), suy ra BD//KC. Xét chùm CA, CK,CF, CG. Ta có BD // CK chắn CA, CF, CG lần lượt tại O, B, D; mà OB = OD (do ABCD là hình bình hành) nên theo định lý cát tuyến song song thì C(AKFG) = -1 suy ra (AKFG) = -1. Theo hệ thức Đề-các: AK 2  AF 1  AG 1  AE 1  AF 1  AG 1 (do AK 2AE ). * Cách 2: Gọi L là trung điểm của CG và K là giao điểm của BL và AG. Do AB // LG nên KB KL  LG BA (định lý Ta-lét) mà LG BA CD CL , suy ra KB KL  CD CL Theo định lý Ta-lét đảo, ta có CK // BD. Xét tam giác ACK, ta có O là trung điểm AC và OE // CK, suy ra E là trung điểm của AK. Xét chùm BA, BK, BF, BG, ta có GD là cát tuyến song song với BA, chắn trên ba tia còn lại hai đoạn bằng nhau CL = LG. Theo định lý cát tuyến song song, BA, BK, BF, BG là chùm điều hòa nên suy ra (AKFG) = -1. Theo hệ thức Đề-các: AK 2  AF 1  AG 1  AE 1  AF 1  AG 1 . Ví dụ 2.44. [2] Cho tam giác ABC có trọng tâm G. Một đường thẳng d thay đổi đi qua G cắt BC, CA, AB lần lượt tại M, N, P. Chứng minh rằng:
  • 63. Tải tài liệu tại sividoc.com Viết đề tài giá sinh viên – ZALO:0973.287.149-TEAMLUANVAN.COM 60 GM 1 GN 1 GP 1 0 Giải. A Q x N G B I C M d P Hình 2.46 Dựng Ax // BC. Gọi I là là trung điểm của BC. Xét chùm Ax, AG, AB, AC có BC là cát tuyến song song với Ax, chắn trên ba tia còn lại hai đoạn bằng nhau IB = IC. Theo định lý cát tuyến song song, Ax, AG, AB, AC là chùm điều hòa suy ra (Ax, AG, AB,AC) = -1. Gọi Q là giao điểm của Ax và d thì (QGPN) = -1 (Hình 2.46). Theo hệ thức Đề-các: 2  1  1 2GM  GM  GM (1) GQ GQ GP GN GP GN MàGIMGAQ ( g g) GI  GM 1 GA GQ 2 Vậy từ (1) ta suy ra1 GM GP GM GN GM 1  GP 1  GN 1  0 . * Nhận xét: Từ bài toán này ta có thể mở rộng thêm: Qua A kẻ các đường thẳng song song với GB, GC và cắt d tại X, Y. Chứng minh 1  1  1  0 . GM GX GY Giải.
  • 64. Tải tài liệu tại sividoc.com Viết đề tài giá sinh viên – ZALO:0973.287.149-TEAMLUANVAN.COM 61 X A Y x G R B M I C S d Hình 2.47 Giả sử các đường thẳng đi qua A, song song GB, GC và cắt BC lần lượt tại R, S. Ta chứng minh được G là trọng tâmARS (vì IB = IC và IR IB  IC IS IG IA 1 3 nên IR = IS, mặt khác lại có IG IA 1 3 ) rồi áp dụng bài toán trên cho ba điểm M, X, Y lần lượt nằm trên RS, AR, SA ta có được điều phải chứng minh. Ví dụ 2.45. [2] Cho đường tròn đường kính CD tâm O. Trên CD lấy điểm A1, A2 sao cho ( A1A2CD ) = -1. Qua lần lượt kẻ các đường t hẳng d1, d2 vuông góc với CD. Một tiếp tuyến thay đổi của (O) cắt d1, d2 lần lượt tại M1, M2. Chứng minh rằng OM 1 const . OM 2 Giải. Việc chứng minh tỉ số OM1  const làm ta nghĩ đến tính chất về tỷ lệ OM 2 đường phân giác trong tam giác. Vậy ta cần tạo ra chùm phân giác có hai tia OM1, OM2. Theo gải thiết, lại xuất hiện tiếp tuyến làm ta liên tưởng đến việc tạo ra sự vuông góc (chùm phân giác). Như vậy ta cần tạo ra các tiếp tuyến nữa để sử dụng tính chất hai tiếp tuyến cắt nhau. Ở đây, tiếp tuyến dựng từ C và D là điều tối ưu nhất ta có thể dùng. Bằng cách cộng các góc, ta có đượcC ' OD ' 90o từ đó chùm tia cần tìm là O(M1M2C’D’). A1, A2